Pathology_Reviewer_UTAH

You might also like

Download as pdf or txt
Download as pdf or txt
You are on page 1of 72

University of Utah

College of Medicine

GENERAL PATHOLOGY REVIEWER


ATHEROSCLEROSIS AND THROMBOSIS SECTION

A 56-year-old reports reduced exercise tolerance over B. Protein C deficiency


the past 5 years. In the past year he has noted chest Incorrect. Protein C deficiency is
pain after ascending a flight of stairs. He smokes 2 associated with venous thrombosis, but
packs of cigarettes per day. On examination he has a the onset of the disease is typically much
blood pressure of 155/95 mm Hg. His body mass index earlier in life.
is 30. Laboratory findings include a total serum C. Immobilization
cholesterol of 245 mg/dL with an HDL cholesterol of 22 CORRECT. The immobilization while in
mg/dL. Which of the following vascular abnormalities is hopsital would predispose to thrombosis
he most likely to have? of leg veins. This is the most common
A. Hyperplastic arteriosclerosis cause for deep venous thrombosis.
Incorrect. Hyperplastic arteriolosclerosis D. Pregnancy
can accompany hypertensive emergency Incorrect. The enlarged uterus
when systolic pressure is >=180 and/or obstructing venous return would
diastolic pressure >=120 mmHg along predispose to thrombosis. However, her
with signs of acute or ongoing end-organ age is not in the childbearing years, and
damage, which he does not have. prior pregnancies do not significantly
B. Lymphedema increase the risk for thrombosis.
Incorrect. Lymphedema is not related to E. Chronic alcohol abuse
the risk factors that he has, which are Incorrect. The liver disease of chronic
factors affecting muscular arteries. alcoholism predisposes to coagulopathy
C. Medial calcific sclerosis with hemorrhage, not thrombosis.
Incorrect. Medical calcific sclerosis is a F. Hypertension
benign form of arteriosclerosis that does Incorrect. Systemic hypertension
not often contribute to significant disease. increases the risk for atherosclerosis,
It does not involve coronary arteries. which increases the risk for thrombosis in
D. Atherosclerosis the arterial system, such as the coronary
CORRECT. He has multiple risk factors arteries or carotid arteries.
for atherosclerosis, including his weight,
smoking, hypertension, and high total
cholesterol with low 'good' HDL A 63-year-old man has had insulin dependent diabetes
cholesterol. His findings suggest mellitus for over two decades. The degree of control of
coronary artery disease with risk for an his disease is indicated by the laboratory finding of
acute coronary syndrome. hemoglobin A1C 10.1%. He has noted episodes of
E. Deep venous thrombosis abdominal pain following meals. These episodes have
Incorrect. He does have an increased worsened over the past year. On physical examination,
risk for thrombosis, but atherosclerosis is there are no masses and no organomegaly of the
the worst risk, and his symptoms suggest abdomen, and he has no tenderness to palpation.
coronary artery disease. Which of the following pathologic findings is most likely
F. Plexiform arteriopathy to be present in this man?
Incorrect. Plexiform arteriopathy is seen A. Ruptured aortic aneurysm
in peripheral pulmonary arteries with Incorrect. Atherosclerotic aneurysms of
pulmonary hypertension. the abdominal aorta can occur with the
advanced atherosclerosis of diabetes
A 54-year-old previously healthy woman is hospitalized mellitus, but there can be a pulsatile
for pneumonia. On the 10th hospital day she is found mass, and rupture is a medical
to have swelling and tenderness of her right leg, which emergency.
has developed over the past 48 hours. Raising the leg B. Hepatic infarction
elicits pain. An ultrasound examination reveals findings Incorrect. The liver has a dual blood
suggestive of femoral vein thrombosis. Which of the supply (hepatic arterial plus portal
following conditions is most likely to have contributed venous circulations) and is difficult to
the most to the appearance of these findings? infarct.
A. Trousseau syndrome C. Mesenteric artery occlusion
Incorrect. The hypercoagulable state of CORRECT. He has 'abdominal angina'
Trousseau syndrome with an underlying from diminished blood flow to the bowel
malignancy such as an adenocarcinoma as a consequence of severe
predisposes to venous thrombosis, but atherosclerosis. Persons with diabetes
her prior healthy condition makes mellitus may have this finding, because
malignancy less likely. all branches of major arteries to the
bowel are affected by atherosclerosis.

Page 1 of 72
Atherosclerosis and Thrombosis | Pathology Reviewer (Utah) | 2 / 72
D. Acute pancreatitis make the infarct hemorrhagic. His
Incorrect. Pancreatitis is a disease of the neurodegenerative disorder with reduced
pancreatic acinar parenchyma, not the movement is a risk for phlebothrombosis.
islets. Even in persons with pancreatitis, B. Atherosclerosis
diabetes mellitus is a rare outcome. Incorrect. Pulmonary atherosclerosis
Diabetes is not a risk for pancreatitis. occurs with pulmonary hypertension, but
E. Chronic renal failure is typically limited in extent, without
Incorrect. Nephrosclerosis can lead to significant arterial narrowing.
renal failure. However, there is typically C. Vasculitis
no pain associated with this process. Incorrect. Vasculitis involving the lungs is
likely to be diffuse.
A 62-year-old man has experienced substernal chest D. Arteriosclerosis
pain upon exertion with increasing frequency over the Incorrect. Arteriolosclerosis does not
past 6 months. An electrocardiogram shows features involve pulmonary circulation.
consistent with ischemic heart disease. He has a total E. Thrombosis
serum cholesterol of 262 mg/dL. By angiography, there Incorrect. Thrombosis within the lung
is 75% narrowing of coronary arteries. In which of the likely occurs in areas of inflammation. In
following locations is a mural thrombus most likely to this case the localized lesion is unlikely to
occur in this man? represent in situ vascular thrombosis.
A. Left atrium
Incorrect. The left atrium is unlikely to A 44-year-old woman has a family history of heart
undergo infarction; it is most often disease. Her father and mother both developed
affected by enlargement from valvular myocardial infarction and congestive heart failure as a
disease. result of occlusive coronary atherosclerosis. A dietary
B. Left ventricle modification to include consumption of which of the
CORRECT. The left ventricle is the major following is most likely to reduce her risk for ischemic
user of oxygen and nutrients and heart disease?
requires a good blood supply. A A. 40% of total caloric intake as fat
complication of ischemic heart disease Incorrect. This is far too much fat. No
with infarction is development of more than 30% of the diet should consist
overlying mural thrombus. Such mural of fat.
thrombi are likely to result from damage B. Increased saturated fat
to the left ventricle from ischemic heart Incorrect. Saturated fats are worse than
disease, either acutely with an underlying unsaturated fats or monosaturated fats.
myocardial infarction, or with a left C. Food with cholesterol
ventricular aneurysm formed following Incorrect. Increased dietary cholesterol is
resolution of a large myocardial infarction not good.
C. Right atrium D. Cold water fish oil
Incorrect. The right atrium is unlikely to CORRECT. Fish oils diminish
be involved with ischemic heart disease. arachidonic acid metabolites and reduce
D. Right ventricle platelet aggregation.
Incorrect. The right ventricle is unlikely to E. Fat found in beef products
be involved with ischemic heart disease. Incorrect. Animal fats tend to be
E. Aorta saturated fats.
Incorrect. The aorta may undergo F. Hydrogenated oils with trans-fats
atherosclerotic changes with aneurysm Incorrect. Trans-fats are partially
formation, in which mural thrombus may hydrogenated fats made commercially.
form. In this case, the likelihood of an Trans-fats tend to increase LDL
acute coronary event is more likely. cholesterol, decrease HDL cholesterol,
F. Vena cava and increase triglycerides - all of which
Incorrect. The systemic venous system is are bad effects.
not directly affected by atherosclerosis.
A study of atherogenesis is performed. There is a
A 66-year-old man with Parkinson disease develops propensity for atheromas to form at muscular arterial
pleuritic chest pain. On exam, he has guarding over the branch points, such as the carotid and aortic
right lower lung with dullness to percussion. A chest CT bifurcations. Which of the following events in the
scan shows a focal, wedge-shaped pleural baesed arteries at these locations is most likely to initiate
area of hemorrhage in the right lower lobe. Which of atherogenesis?
the following is the most likely cause for his pulmonary A. Collagen synthesis
lesion? Incorrect. There is collagen deposition as
A. Embolism atheromatous plaques develop, but this
CORRECT. A pulmonary infarction is is not the initiating event.
described. A thromboembolus from the B. Endothelial dysfunction
venous circulation, usually arising in CORRECT. A variety of stressors may
large leg veins, has lodged in a peripheral affect endothelial cells, including the
pulmonary arterial branch. The bronchial shear stress from turbulent flow at arterial
arterial supply to the lung does not branch points. Endothelial dysfunction is
provide enough oxygenation to prevent the initiating event that allows insudation
infarction, but does provide blood to of lipids to promote atherogenesis.
Atherosclerosis and Thrombosis | Pathology Reviewer (Utah) | 3 / 72
C. Lactic acidosis A. Thrombus
Incorrect. Lactic acid may build up in Incorrect. Thrombosis is a complication
tissues with oxygen debt, such as muscle. of an atheroma.
The metabolic rate of arterial cells is not B. Fatty streak
high, and direct diffusion provides CORRECT. A fatty streak on the arterial
oxygen and nutrients to endothelium. intimal surface is the first grossly visible
D. Cholesterol breakdown sign of atheroma formation. It is benign
Incorrect. Once endothelial injury has and reversible, but it may be the
occurred, lipids begin to collect, but lipids precursor to more severe plaques.
are not the inciting factor at arterial C. Calcification
branch points. Incorrect. Calcification is a complication
E. Hypoxemia of an atheroma.
Incorrect. Hypoxemia does not affect D. Hemorrhage
muscular arteries directly, as the arterial Incorrect. Hemorrhage is a complication
cells do not have a high metabolic rate. of an atheroma.
E. Exudate
A 66-year-old woman has the sudden loss of Incorrect. Lymphocytes and
movement on part of the left side of her body. She has macrophages play a role in
smoked a pack of cigarettes a day for the past 45 years. atherogenesis, but their numbers are
She has vital signs including T 37.1°C, P 80/minute, relatively small.
RR 16/minute, and BP 160/100 mm Hg. A cerebral F. Ulceration
angiogram reveals occlusion of a branch of her middle Incorrect. Ulceration of an atheromatous
cerebral artery. Laboratory findings include a plaque is an advanced lesion, not an
hemoglobin A1C of 9%. Which of the following early lesion. Ulceration or fissuring of the
components of blood lipids is most important in plaque increases the likelihood for
contributing to her disease? overlying thrombosis.
A. Chylomicrons
Incorrect. Chylomicrons are formed by A 63-year-old man has had increasing exercise
complexing lipids absorbed from the intolerance for the past 6 years so that he now
small intestine to transport apoproteins. becomes short of breath upon climbing a single flight
They then travel to the liver. They do not of stairs. Laboratory studies have shown fasting blood
directly produce atheromatous plaques. glucose measurements from 145 to 210 mg/dL for the
B. Lipoprotein lipase past 25 years, but he has not sought medical treatment.
Incorrect. Endothelial lipoprotein lipase If he dies suddenly, which of the following is most likely
splits off fatty acids that are then used as to be the immediate cause of death?
a substrate in other metabolic pathways A. Myocardial infarction
and not in atheroma formation. CORRECT. This is the most common
C. Oxidized LDL cause of death in persons with diabetes
CORRECT. She has had a 'stroke' which mellitus, because of the high prevalence
is most often a consequence of cerebral of advanced coronary atherosclerosis.
atherosclerosis or embolic disease from B. Nodular glomerulosclerosis
the heart as a consequence of ischemic Incorrect. Although renal failure is a
heart disease from atherosclerosis. LDL common complication of diabetes, it is
brings cholesterol to arterial walls, and not the major cause of death.
when increased LDL is present or when C. Cerebral hemorrhage
there is hypertension, smoking, and Incorrect. Cerebrovascular disease in
diabetes, there is more degradation of diabetes is common but is not the major
LDL to oxidized LDL which is taken up cause of death.
into arterial walls via scavenger receptors D. Hyperosmolar coma
in macrophages to help form atheromas. Incorrect. This complication may be life-
D. VLDL threatening, but it is not frequent.
Incorrect. Both adipose tissue and E. Right lower leg gangrene
muscle are areas where VLDL is Incorrect. This complication is usually
transformed to LDL, and this does not treated before secondary sepsis
directly promote atherogenesis. becomes life-threatening.
E. HDL cholesterol
Incorrect. HDL cholesterol helps to A 45-year-old man dies suddenly and unexpectedly.
transport lipid away from arterial walls to The immediate cause of death is found to be a
the liver, thus protecting against hemorrhage in the right basal ganglia region. On
atheroma formation. microscopic examination his renal artery branches
have concentric endothelial cell proliferation which
An autopsy study reveals that evidence for atheroma markedly narrows the lumen, resulting in focal
formation can begin even in children. The gross ischemia and hemorrhage of the renal parenchyma. An
appearances of the aortas are recorded and compared elevation in which of the following substances in his
with microscopic findings of atheroma formation. blood is most likely to be associated with these findings?
Which of the following is most likely to be the first visible
gross evidence for the formation of an atheroma?
Atherosclerosis and Thrombosis | Pathology Reviewer (Utah) | 4 / 72
A. Ammonia F. Decreased plasma cortisol
Incorrect. An elevated blood ammonia is Incorrect. She is unlikely to have an acute
indicative of hepatocyte dysfunction and Addison disease which would be
increasing hepatic failure. characterized by hypotension and
B. Calcium hypoglycemia, and she is unlikely to
Incorrect. Hypercalcemia can be present at this age with congenital
associated with hyperparathyroidism. adrenal hyperplasia.
C. Cholesterol
Incorrect. Increased cholesterol leads to A 73-year-old woman who exercises regularly falls
atherosclerotic vascular changes. down the stairs and injures her right hip. A radiograph
D. Renin is taken of the pelvis. There is no fracture but the
CORRECT. The findings suggest radiograph reveals calcification of the small muscular
hyperplastic arteriolosclerosis, which arteries lateral to her uterus. What is the probable
accompanies hypertensive emergency vascular lesion which accounts for this calcification?
when systolic pressure is >=180 and/or A. Ulcerative atherosclerosis
diastolic pressure >=120 mmHg along Incorrect. Although calcification may
with signs of acute or ongoing end-organ complicate atheromas, this is typically
damage. Reduced renal blood flow seen in larger vessels.
increases renin, driving hypertension. B. Calcific medial sclerosis
E. Troponin I CORRECT. Monckeberg calcific medial
Incorrect. Myocardial injury increases the sclerosis is a benign, incidental finding
troponin I. most often seen in the elderly. Small
F. Triglyceride arteries in pelvis, breast, neck, and
Incorrect. An elevated triglyceride extremities are typically involved. The
increases the risk for atherosclerosis with vascular lumen is not compromised.
atheroma formation, but not concentric C. Metastatic calcification
endothelial proliferation. Incorrect. Metastatic calcification from
G. C-reactive protein hypercalcemia would involve other areas
Incorrect. An increased CRP can be a in lung, kidney and GI tract first.
marker for an increased risk for D. Trauma
atherosclerosis with atheroma formation, Incorrect. Dystrophic calcification
but not concentric endothelial following injury from trauma would be
proliferation. localized and would take weeks to form.
E. Dystrophic calcification
A 10-year-old previously healthy child has been noted Incorrect. Dystrophic calcification from
by her parents to be constantly thirsty. She is tissue damage would probably not be so
consuming large amounts of soft drinks. She is localized to arteries.
urinating often. Her diet and exercise patterns have not
changed, except for an increased appetite, yet she A 57-year-old woman has had episodes of abdominal
appears cachectic and has lost 7 kg over the past 4 pain following meals for the past year. Her BMI is 31.
months. On physical examination there are no She has hypertension. Her total serum cholesterol is
abnormal findings, other than peripheral muscle increased. Which of the following types of blood vessel
wasting and weakness. Which of the following is the most likely location for the pathologic abnormality
laboratory findings would you most strongly suspect is causing her pain?
present in this girl? A. Artery
A. Increased blood insulin CORRECT. She has abdominal angina
Incorrect. The insulin is probably from vascular insufficiency. She has
decreased. multiple risk factors for atherosclerosis.
B. Decreased blood glucagon Atherosclerosis occurs in muscular
Incorrect. The glucagon is probably arteries, because the level of blood
increased. pressure in arteries predisposes to
C. Ketonuria endothelial dysfunction.
CORRECT. Ketonuria is typical for type I B. Arteriole
diabetes mellitus. The findings of Incorrect. The blood pressure is
polydipsia, polyuria, and polyphagia are considerably lowered at the level of
present in this case. The absolute arterioles. Instead of atherogenesis,
decrease in insulin leads to a catabolic arteriolosclerosis occurs in arterioles.
state, with underutilization of C. Capillary
carbohydrates, and lipolysis with Incorrect. Capillaries are not involved
metabolism of fatty acids to produce with atherogenesis.
ketone bodies. D. Venule
D. Increased serum osmolality Incorrect. Venules are not involved with
Incorrect. This finding is more typical of atherogenesis.
type II diabetes mellitus. E. Vein
E. Decreased plasma H+ (alkalosis) Incorrect. Veins are not involved with
Incorrect. The child probably has atherogenesis.
ketoacidosis.
Atherosclerosis and Thrombosis | Pathology Reviewer (Utah) | 5 / 72
A 29-year-old woman is involved in a motor vehicle D. Increased synthesis of creatine kinase
accident that results in severe lacerations to her lower Incorrect. Creatine kinase is released,
extremities, along with blunt abdominal trauma. In the but synthesis does not (cannot) increase
emergency room she is noted to have cool, pallid skin. from the injured cells.
She has vital signs showing T 36.9°C, P 110/minute, E. Myofiber atrophy
RR 26/minute, and BP 70/30 mm Hg. She has Incorrect. The elevation in creatine
decreased urine output. Which of the following kinase is indicative of myocardial cell
laboratory findings on a blood sample from this patient necrosis, not atrophy. Atrophic myofibers
is most likely to be present? have less cellular contents - and less
A. Hematocrit of 54% enzymes - to release.
Incorrect. She has signs of marked blood
loss, so her hematocrit should be low-- A 52-year-old woman has experienced marked
probably in the 20's or lower. substernal, crushing chest pain for the past 3 hours.
B. Glucose of 181 mg/dL Her vital signs show T 36.9°C, P 90/minute, RR
Incorrect. Hyperglycemia is not a feature 18/minute, and BP 100/60 mm Hg. Laboratory studies
of blood loss with shock. include a serum creatine kinase MB fraction of 10
C. PaO2 of 20 mmHg microgm/L as well as a serum troponin I of 4.5 ng/mL.
Incorrect. She has had blood loss, but the Which of the following findings is the best evidence for
oxygen carrying capacity of the the presence of a coronary arterial thrombus as the
remaining RBCs is not affected, and her etiology for her chest pain?
ability to oxygenate is not completely lost. A. Total serum cholesterol of 300 mg/dL
She is compensating via increased Incorrect. Though an increased total
cardiac output. serum cholesterol suggests a risk for
D. Lactic acid of 4.8 mmol atherosclerosis and its complications, it
CORRECT. She has marked blood loss does not predict acute events.
with shock. There will be vasoconstriction B. Large size of the infarction by
in skin and soft tissue in response to the scintigraphic scanning
hypovolemia. Decreased renal blood flow Incorrect. The size of the infarct gives no
from shock may lead to acute tubular clue as to whether a thrombus was
necrosis. The lack of tissue perfusion present or not.
with shock leads to increased anaerobic C. 80% coronary occlusion by angiography
glycolysis and lactic acidosis. Incorrect. The severity of the
E. Troponin I of 4 ng/mL atherosclerosis does not necessarily
Incorrect. An elevated troponin I predict whether a thrombus will have
suggests myocardial ischemia. With formed. Narrowing may be produced by
good coronary blood flow in a young the atheroma formation alone.
person, the myocardium is not seriously D. Response to thrombolytic therapy
affected until the patient is near death. CORRECT. The purpose of thrombolytic
therapy is to lyse the thrombus and
A 61-year-old man has the sudden onset of severe restore blood flow. Many acute coronary
chest pain. Vital signs include T 37°C, P 102/minute, events are the result of thrombus
RR 20/minute, and BP 80/40 mm Hg. An formation on pre-existing atheromas.
electrocardiogram demonstrates changes that are Low dose aspirin therapy helps prevent
consistent with myocardial ischemia involving the left such events.
lateral ventricular free wall. He is given thrombolytic E. Hemoglobin A1C of 10.1%
therapy with tissue plasminogen activator (tPA) 3 hours Incorrect. Atherosclerosis is accelerated
after the onset of chest pain. However his serum and complications are more likely with
creatinine kinase is found to be 450 U/L 3 hours after diabetes mellitus, but this does not
this therapy. Which of the following cellular events has indicate that a thrombus is present.
most likely occurred?
A. Cellular regeneration In an experiment, a glass bead is embolized to a
Incorrect. The elevation in creatine branch of the renal artery. A day later there is a focal
kinase is indicative of myocardial cell area in which the renal parenchymal cells in the
necrosis, not regeneration. distribution of the occluded artery show karyolysis and
B. Drug-induced necrosis karyorrhexis. The outlines of the cells are still visible,
Incorrect. The tPA does not produce a but the nuclei have lost basophilic staining and the
chemical injury, but induces thrombolysis cytoplasm is eosinophilic but pale. Which of the
to help restore blood flow. following types of cellular necrosis is most likely
C. Reperfusion injury present?
CORRECT. The restoration of blood flow A. Caseous
is helpful if the existing cell damage is not Incorrect. Caseous necrosis is a mixture
great, so further damage can be of liquefactive and coagulative necrosis,
prevented. However, the reperfusion of and is typical for granulomatous
damaged cells results in generation of inflammation caused by mycobacterial or
oxygen free radicals to produce a fungal infections.
reperfusion injury. B. Coagulative
CORRECT. A typical ischemic infarction
with coagulative necrosis is described.
Coagulative necrosis is most likely to
Atherosclerosis and Thrombosis | Pathology Reviewer (Utah) | 6 / 72
occur in solid internal organs with high circulation is through a patent foramen
metabolic demands. ovale - a rare event.
C. Fatty
Incorrect. Fat necrosis most often occurs A 25-year-old previously healthy primigravida is in the
in breast (with trauma) and in pancreas first trimester of pregnancy. During two successive
(with acute pancreatitis). prenatal visits, she has fasting serum glucose levels of
D. Gangrenous 127 and 131 mg/dL. Prior to this pregnancy, her fasting
Incorrect. Gangrene involves a whole serum glucose was 80 mg/dL. A hemoglobin A1C level
organ or body part. There can be is 8.1% at the last visit, at 18 weeks gestation. She
elements of coagulative necrosis ('dry' feels well and has no major health problems. Which of
gangrene) and liquefactive necrosis the following problems is most likely to become
('wet' gangrene). apparent in the latter part of her pregnancy?
E. Liquefactive A. Intrauterine fetal growth retardation
Incorrect. Liquefactive necrosis is most Incorrect. The babies of diabetic mothers
typical for brain infarction and for tend to have macrosomia (increased
abscesses with many neutrophils. size).
B. Ketoacidosis
A 53-year-old man has the sudden onset of chest pain. Incorrect. This patient has gestational
He is found to have a serum troponin I of 5 ng/mL. A diabetes, not type 1 diabetes mellitus.
year later he has reduced exercise tolerance. An C. Hyperosmolar coma
echocardiogram reveals an akinetic segment of left Incorrect. This unlikely, given that
ventricle, and he has reduced cardiac output, with an hyperosmolar coma is more typical of
ejection fraction of 25%. He then experiences a older type 2 diabetics.
transient ischemic attack (TIA). His serum troponin I is D. Congenital anomalies
now <0.5 ng/mL. Thrombus formation involving which Incorrect. The rate of fetal malformation
of the following locations is most likely to have put him is slightly increased in mothers with
at greatest risk for the TIA? diabetes mellitus type 1 or 2, but not
A. Cerebral vein gestational diabetes.
Incorrect. The cerebral bridging veins E. Placental insufficiency
cross the subdural space and rupture of CORRECT. The big problem in
them can produce a subdural hematoma. gestational diabetes is eventual placental
Cerebral venous thrombosis is far less malfunction in later pregnancy (third
common than cerebral arterial trimester) with potential fetal demise.
thrombosis or embolism.
B. Vertebral artery A 54-year-old man with diabetes mellitus has had 3
Incorrect. Vertebral artery thrombi are urinary tract infections during the past year. He now
rare and may be the result of local trauma. sees the physician for an ulceration on his right big toe
C. Superior vena cava which has not healed in 2 months. Laboratory studies
Incorrect. The superior vena cava may on each of his doctor visits over the past year show
become occluded by thrombus when blood glucose levels below 110 mg/dL. This situation
involved by obstruction in the chest, such could be best explained by which of the following
as mass effect from a neoplasm in the laboratory findings?
lung or medastinum. A thromboembolus A. Elevated hemoglobin A1C
from a leg vein may pass through the CORRECT. Out-of-control diabetes over
inferior vena cava on the way to the time can be better detected because the
pulmonary arterial tree. glycosylated RBCs will persist for months
D. Left ventricle and the Hgb A1C increased. Single
CORRECT. Mural thrombi can form over glucose values reflect the past few hours.
the damaged area of ventricular wall His problems developed over years.
following myocardial infarction. This can B. Increased urine ketones
happen acutely overlying the necrotic Incorrect. The ketones suggest
myocardium; it can occur remotely in a ketoacidosis, but this could have been a
ventricular aneurysm. Portions of the recent development.
mural thrombus can break off and C. Elevated serum osmolarity
embolize via the systemic arterial Incorrect. Hyperosmolar coma is
circulation to places such as the cerebral possible, but would be relatively recent in
circulation. onset.
E. Coronary artery D. Decreased plasma insulin
Incorrect. His original myocardial Incorrect. This would be a likely finding
infarction may have been due to coronary with diabetes mellitus, but would tell you
thrombosis, but his last event was not, nothing about the course or the degree of
since the troponin I was not elevated. control of hyperglycemia.
F. Saphenous vein E. Elevated titer of anti-insulin antibodies
Incorrect. A thrombus in the saphenous Incorrect. In type I diabetes mellitus,
vein may become a thromboembolus that elevated anti-insulin antibodies are often
travels to the pulmonary arterial tree of present, but tell you nothing about the
the lungs. The only way such a course or the control of the
thromboembolus may reach the arterial hyperglycemia.
Atherosclerosis and Thrombosis | Pathology Reviewer (Utah) | 7 / 72
A 25-year-old man experiences chest pain on exercise Incorrect. The major damage to the heart
when he attempts to climb three flights of stairs. This with diabetes mellitus is done via
pain is relieved by sublingual nitroglycerin. He is 178 atherosclerotic coronary artery disease.
cm tall and weighs 101 kg. His blood pressure is Atherosclerosis affects muscular arteries
130/85 mm Hg. Laboratory studies show a total serum by formation of intimal atheromatous
cholesterol of 550 mg/dL with an HDL cholesterol plaques which occlude the lumen and
component of 25 mg/dL. The blood glucose is 120 lead to ischemic injury.
mg/dL. He is worried about these findings because his D. Impotence
brother died of a myocardial infarction at age 34. Which CORRECT. The hyperglycemia leads to
of the following conditions is this man most likely to formation of sorbitol in tissues not
have? requiring insulin for glucose uptake,
A. Diabetes mellitus, type II resulting in osmotic cellular injury. This
Incorrect. Type II diabetes mellitus is can occur in small vessels, retina, kidney,
most often a disease of middle aged to and nerves. The failure of erection and
older adults, taking years for ejaculation results from diabetic
complications to develop. His obesity is a neuropathy.
risk for DM type II. E. Pyelonephritis
B. Hypertensive emergency Incorrect. Pyelonephritis with diabetes is
Incorrect. A hypertensive emergency related to diminished neutrophil function
may, on occasion, complicate with hyperglycemia and risk for infection.
longstanding hypertension that is
associated with development of A 49-year-old woman has experienced marked pain in
atherosclerosis, but this is usually a her lower extremities on ambulation more than 300
complication seen in middle aged to older meters for the past 5 months. On physical examination,
adults. her lower extremities are cool and pale, without
C. Familial hypercholesterolemia swelling or erythema. No dorsalis pedis or posterior
CORRECT. The very high cholesterol tibial pulses are palpable. Her body mass index is 32.
with symptoms of coronary artery She is a smoker. Which of the following abnormalities
disease at such a young age, coupled of the vasculature is most likely to account for these
with the family history, are all consistent findings?
with familial hypercholesterolemia. In A. Lymphatic obstruction
most persons with hypercholesterolemia Incorrect. The result of lymphatic
from multifactorial causes, including diet obstruction is lymphedema, with marked
and exercise patterns, the cholesterol swelling from collection of interstitial fluid.
rarely exceeds 400 mg/dL. Lymphatics normally scavenge the fluid
D. Cushing syndrome and protein that escapes at the venule
Incorrect. The hypercortisolism of and capillary level and returns it to
Cushing syndrome tends to promote circulation.
secondary diabetes mellitus, but the B. Arteriosclerosis
underlying conditions that lead to Incorrect. Arteriolosclerosis is more
Cushing syndrome are unlikely to allow a typically found in the kidneys and brain,
long enough survival to develop severe not the extremities.
atherosclerosis. C. Atherosclerosis
E. Morbid obesity CORRECT. She has claudication from
Incorrect. Morbid obesity is a risk for type severe peripheral arterial atherosclerosis,
II diabetes mellitus, and decades are most likely from the iliac arteries down.
required to develop complications of Her major risk factor is obesity that
atherosclerosis. It is unlikely that the promotes insulin resistance and diabetes
cholesterol would be this high. mellitus that leads to atherosclerosis.
D. Medial calcific sclerosis
In a clinical study of patients with diabetes mellitus, a Incorrect. Medial calcific sclerosis of
group of patients is found who have had blood glucose Monckeberg is not typically a serious
measurements ranging from 140 to 180 mg/dL for at problem, because there is minimal
least 10 years. Rectal biopsies from these patients now arterial vascular luminal narrowing.
show that there is a form of osmotic cellular injury E. Venous thrombosis
present in arterioles. This form of injury is most likely to Incorrect. Venous thrombosis should
be manifested elsewhere in the body by which of the lead to a warm, tender, swollen extremity,
following complications? typical for phlebothrombosis
A. Cerebral infarction (thrombophlebitis).
Incorrect. Atherosclerotic
cerebrovascular disease is more An autopsy study is conducted involving the gross
frequent with diabetes mellitus, but is appearance of the aorta of adults ranging in age from
usually large vessel disease not directly 60 to 90. In some of these patients, the aorta
related to osmotic vascular injury. demonstrates atheromatous plaques covering from 70
B. Gastric adenocarcinoma to 95% of the intimal surface area, mainly in the
Incorrect. The risk for cancer is not abdominal portion, with ulceration and calcification.
increased with diabetes mellitus. Which of the following contributing causes of death are
C. Congestive heart failure these patients most likely to have?
Atherosclerosis and Thrombosis | Pathology Reviewer (Utah) | 8 / 72
A. Hyperparathyroidism E. Rheumatoid arthritis
Incorrect. Hypercalcemia is not a risk Incorrect. Persons with autoimmune
factor for advanced atherosclerosis. diseases generally do not have major
B. Hypertension problems with thrombosis, though some
CORRECT. Hypertension is a risk factor can develop an antiphospholipid
that accelerates atheromatous plaque syndrome with thrombosis.
formation.
C. Marfan syndrome At autopsy, the kidneys of a 44-year-old woman who
Incorrect. In Marfan syndrome, there is died suddenly are found to be of normal size. Their
cystic medial necrosis of the aorta, but surfaces appear finely granular. There are small
not atherosclerosis. hemorrhages noted. The cortices appear pale.
D. Thrombophlebitis Microscopically, many small renal arteries and
Incorrect. Thrombophlebitis involves arterioles demonstrate concentric intimal thickening
veins. Atherosclerosis does not involve with marked luminal narrowing. These findings are
veins. most likely to be present as a result of which of the
E. Vasculitis following underlying diseases?
Incorrect. Various forms of vasculitis may A. Amyloidosis
produce vascular occlusion, but do not Incorrect. Amyloid deposits can be seen
accelerate atherosclerosis. in small arteries and glomeruli in the
F. Colonic adenocarcinoma kidney, but do not produce a hyperplastic
Incorrect. Cancer is a debilitating arteriolosclerosis.
condition likely to result in reversal of B. Systemic lupus erythematosus
atheroma formation. Hypercoagulable Incorrect. Lupus nephritis may be
states from cancers are most likely to accompanied by a vasculitis and by
result in venous thrombosis, though glomerular basement membrane
arterial thrombi are possible, but not thickening, but not hyperplastic
dependent upon prior arterial atheroma arteriolosclerosis.
formation. C. Scleroderma
G. Systemic lupus erythematosus CORRECT. Hyperplastic
Incorrect. SLE and autoimmune diseases arteriolosclerosis is typically seen in
in general are debilitating conditions in association with severe hypertension,
which existing atheromas may diminish which can complicate prior 'essential'
in size. The antiphospholipid syndrome hypertension or occur with progressive
that can sometimes occur with SLE can systemic sclerosis (scleroderma).
lead to thrombosis, not atheroma D. Rheumatoid arthritis
formation. Incorrect. There are no prominent
vascular changes with rheumatoid
A 58-year-old woman has the sudden onset of severe arthritis.
dyspnea with chest pain and diaphoresis. A chest CT E. Viral hepatitis
scan shows areas of decreased attenuation within the Incorrect. Cryoglobulinemic vasculitis
main pulmonary arteries. Her D-dimer assay is may complicate some cases of hepatitis
elevated. Her acute event is most likely to be a B or C infection. In general viral infections
consequence of which of the following? do not involve the vasculature.
A. Placement of a hip prosthesis F. Diabetes mellitus
CORRECT. Immobilization is a major risk Incorrect. Diabetes mellitus leads to
for the development of venous accelerated atherosclerosis of medium to
thrombosis and subsequent pulmonary large arteries. There may also be hyaline
embolization (the areas of diminished arteriolosclerosis, but not hyperplastic
attenuation). Immobilization occurs arteriolosclerosis.
following surgery. The postoperative goal:
get the patient mobile as soon as A 34-year-old previously healthy woman notes that she
possible, and meanwhile employ has bruises form on her arms and legs with just minor
methods to reduce the risk for thrombosis. trauma. Physical examination reveals areas of purpura
B. Marked thrombocytopenia from 1 to 3 cm in size over her trunk and extremities,
Incorrect. Thrombocytopenia leads to but no swelling, warmth, or erythema. Peripheral
bleeding, not to thrombosis. pulses are all palpable and full. Her blood pressure is
C. Chronic alcoholism 110/70 mm Hg. An ultrasound examination of her lower
Incorrect. Chronic alcoholics tend to have extremities with Doppler flow measurement reveals no
a coagulopathy due to their underlying evidence for thrombosis. Laboratory findings include
liver disease which predisposes them to serum urea nitrogen of 16 mg/dL, LDH 300 U/L, total
bleeding, not thrombosis. protein 6.9 g/dL, albumin 5.3 g/dL, alkaline
D. Infection with the human phosphatase 50 U/L, AST 40 U/L, and ALT 20 U/L.
immunodeficiency virus Which of the following additional laboratory findings is
Incorrect. Some AIDS patients get an most likely to be present in this patient?
acquired protein S deficiency that A. Hyperglycemia
predisposes to thrombosis, but this is not Incorrect. Diabetes mellitus is a disease
common. marked by hyperglycemia that leads to
early and advanced atherosclerosis with
peripheral vascular disease involving
Atherosclerosis and Thrombosis | Pathology Reviewer (Utah) | 9 / 72
arteries, with reduced blood flow and A 53-year-old woman is found on a routine physical
reduced pulses. examination to have vital signs with T 37°C, P
B. Hypercholesterolemia 78/minute, RR 16/minute, and BP 165/110 mm Hg.
Incorrect. Patients with There are no other significant findings. She has an
hypercholesterolemia are at increased abdominal ultrasound examination that shows the right
risk for accelerated atherosclerosis and kidney to be atrophic. Angiography reveals markedly
peripheral vascular disease involving reduced blood flow to the right renal artery from an
arteries. occlusion at the orifice in the abdominal aorta. Which
C. Lactic acidosis of the following laboratory findings is she most likely to
Incorrect. Poor tissue perfusion leads to have?
increased anaerobic glycolysis and lactic A. Serum sodium of 161 mmol/L
acidosis. However, her pulses are not Incorrect. Marked dehydration or free
reduced. water loss could lead to this degree of
D. Thrombocytopenia hypernatremia. Many persons with
CORRECT. The platelets are essential hypertension may have
responsible for dealing with small leaks in increased retention of sodium, but it is not
small vessels. Thrombocytopenia is marked by significant hypernatremia.
marked by petechiae and purpura. She B. Serologic evidence of anti-cardiolipin
does not have evidence for peripheral antibody
vascular disease, since her circulation is Incorrect. Anti-cardiolipin antibody is
good. Venous thrombosis should lead to seen with the antiphospholipid syndrome,
swelling and tenderness. which is a cause for thrombosis. It is
E. Hypoprothrombinemia unlikely that the selective renal artery
Incorrect. She has no evidence for liver occlusion represents a hypercoagulable
disease, which would reduce state.
prothrombin production. The pattern of C. Prothrombin time of 25 seconds
her bleeding fits best with lack of platelets, Incorrect. An elevated prothrombin time
not coagulation factors. predisposes to bleeding.
F. Anemia D. Plasma renin activity of 4.8 ng/mL/hr
Incorrect. The amount of bleeding into supine
soft tissues is unlikely to lower the CORRECT. She has renal artery
hematocrit significantly. Bone marrow stenosis that is reducing renal blood flow
injuries that lead to reduced platelet on the right and simulating the JG cells to
production, however, may also lead to secrete renin to try and raise blood
anemia. With anemia, tissue appear pale. pressure to increase renal blood flow.

A longitudinal study is conducted to detect serum E. Serum lactic acid of 5.5 mmol/L
markers that predict risk for death from acute coronary Incorrect. The decreased perfusion of the
syndromes. A marker is identified that is synthesized right kidney occurs over a long period of
and released from the liver in response to formation of time, without a significant rise in lactate
cytokines in atheromatous plaques. This marker that would accompany more global
increases endothelial adhesiveness to platelets. What hypoperfusion.
is this marker most likely to be?
A. Prostacyclin A study of pathologic findings in the islets of
Incorrect. Prostacyclin is produced in the Langerhans is performed. It is observed that insulitis
arachidonic acid pathway in endothelium may occur in the islets. The inflammatory infiltrate is
to reduce platelet activation. predominantly composed of T lymphocytes. Which of
B. Immunoglobulin G the following is most likely to occur as a consequence
Incorrect. Immunoglobulins are not of insulitis?
directly implicated in atherogenesis. A. Neoplasia
C. C-reactive protein Incorrect. An insulitis is typical for type I
CORRECT. An increased CRP predicts a diabetes mellitus; there is no increased
greater likelihood for adverse events risk for neoplasia.
from atherogenesis. One benefit of B. Malabsorption
'statin' drugs is their CRP lowering effect. Incorrect. The insulitis involves just the
D. Alpha-1-antitrypsin endocrine portion of the pancreas, not
Incorrect. AAT is the major circulating the exocrine portion, so there is no loss
anti-protease. It mitigates neutrophil of pancreatic enzymes for digestion.
proteases. However, atherogenesis is C. Ketoacidosis
not related to neutrophil actions. CORRECT. An insulitis is seen with type
E. Acetoacetate I diabetes mellitus. Actually, it is rare to
Incorrect. Free fatty acids, including see it because by the time the patients
acetoacetate, are released with lipolysis, present with overt diabetes mellitus, the
but do not directly participate in islets are long gone.
atherogenesis. D. Obesity
Incorrect. Obesity is typically associated
with type II diabetes mellitus; with type I
diabetes mellitus, there is a catabolic
state with weight loss.
Atherosclerosis and Thrombosis | Pathology Reviewer (Utah) | 10 / 72
E. Sepsis creatine kinase (CK) reflects changes in
Incorrect. The lymphocytes in the islets any striated muscle, and is not specific
are part of an autoimmune phenomenon, for cardiac muscle. The CK-MB fraction
not an infectious process. is most specific to myocardium, but
remains elevated for up to 3 days.
A 57-year-old man has had blood pressure E. Lipase
measurements in the range of 160/95 to 180/110 mm Incorrect. Lipase is increased with
Hg for many years. He has taken no medications. A pancreatitis, which is not related to
renal scan reveals kidneys of normal size for age. cardiovascular disease.
These findings with benign nephrosclerosis are most
likely to occur with which of the following vascular A 68-year-old woman has survived multiple episodes
changes? of pulmonary thromboembolism during the past three
A. Hyaline arteriosclerosis months. On physical examination there are no
CORRECT. Hyaline arteriolosclerosis is abnormal findings. Which of the following is the most
a feature of long-standing hypertension likely underlying condition leading to this patient's
and is part of benign nephrosclerosis recurrent pulmonary thromboembolism?
which may go on for many years with no A. Micronodular cirrhosis of the liver
complications. The small arterioles are Incorrect. Liver disease leads to a
thickened with pink (hyaline) coagulopathy marked by a tendency to
proteinaceous deposits. bleeding, not thrombosis.
B. Monckeberg’s medial calcific sclerosis B. Adenocarcinoma of the pancreas
Incorrect. Medial calcific sclerosis is a CORRECT. The recurrent episodes
form of arteriolosclerosis without suggest a hypercoagulable state, and
significant complications. The vascular carcinomas can do this (Trousseau
lumen is not significantly compromised syndrome) by releasing substances that
by the medial calcification. act as thromboplastins.
C. Complex calcified atherosclerosis C. Thrombocytopenia
Incorrect. Coronary artery disease is Incorrect. A decreased platelet count is a
caused by atherosclerosis that can be risk for hemorrhage, not thrombosis.
accelerated by hypertension. D. Familial hypercholesterolemia
Atheromatous plaques occur in medium Incorrect. Hypercholesterolemia could
to large sized arteries and can be accelerate atherosclerosis and
complicated by calcification of the thrombosis, but in arteries, not veins.
plaques. E. Mitral valve endocarditis
D. Arterial mural thrombosis Incorrect. An endocarditis could lead to
Incorrect. Mural thrombosis typically embolization, but into the systemic
occurs in the aorta and its branches when circulation from the mitral valve.
there is severe atherosclerosis. F. Type II diabetes mellitus
E. Hyperplastic arteriosclerosis Incorrect. Diabetes mellitus increases the
Incorrect. Hyperplastic arteriolosclerosis risk for atherosclerosis, but
is usually seen in the kidneys when there atherosclerosis does not involve veins.
is severe hypertension.
A 57-year-old woman has experienced episodes of
A 45-year-old man feels some crushing chest pain altered mental status for the past month. On physical
along with numbness in his left arm after shoveling 15 examination, she has bilateral carotid artery bruits. She
cm of snow off his driveway. Three hours later he is given folic acid supplementation. Which of the
collapses and is taken to the emergency room. Which following laboratory test findings is she most likely to
of the following laboratory tests run on a blood have?
specimen is most useful in this situation? A. Factor V Leiden mutation
A. Total white blood cell count Incorrect. The factor V Leiden mutation is
Incorrect. An elevation in WBC count is a risk for a hypercoagulable state with
not a specific indicator for myocardial venous thrombosis, typically manifested
infarction. at a young age.
B. Glucose B. Factor VIII deficiency
Incorrect. Though he may have diabetes Incorrect. Factor VIII deficiency is one
mellitus as an underlying cause for the cause for hemophilia with risk for
atherosclerosis leading to myocardial bleeding into soft tissues, manifested
infarction, serum glucose will not help to early in life.
diagnose the infarction. C. Increased carcinoembryonic antigen
C. Platelet count Incorrect. An increased CEA can be seen
Incorrect. Platelet counts are of no value with some malignancies, such as
in diagnosing infarctions, even though adenocarcinoma of the colon. In this
platelets play a role in formation of case, the mass is an aortic aneurysm
thrombi that can lead to infarction. caused by atherosclerosis.
D. Troponin D. Homocysteinemia
CORRECT. Elevations in troponins are CORRECT. An elevated plasma
going to be seen acutely with myocardial homocysteine level is a risk for
infarction beginning within 3 hours and atherosclerosis as well as thrombosis.
remaining for up to 14 days. Total Her findings point to transient ischemic
Atherosclerosis and Thrombosis | Pathology Reviewer (Utah) | 11 / 72
attacks (TIAs) with risk for stroke. The diuresis, and vasodilation, lessening the
carotid bruits represent arterial narrowing strain on the myocardium.
as the result of atherosclerosis. D. Hematocrit
E. Increased HDL cholesterol Incorrect. The hematocrit suggests the
Incorrect. HDL is the 'good' cholesterol ability of the blood to carry oxygen and
that tends to protect against prevent ischemia, but does not predict
atherosclerosis. whether there is poor perfusion with
ischemia.
A 70-year-old man has noted coldness and numbness E. Sedimentation rate
of his lower left leg, increasing over the past 4 months. Incorrect. The sed rate gives an
He also experiences pain in this extremity when he indication of the presence of
tries walking more than the distance of half a city block. inflammation, a consequence of an
On physical examination, his dorsalis pedis, posterior ischemic event.
tibial, and popliteal artery pulses are not palpable.
Which of the following laboratory test findings is he A 44-year-old African-American man has had elevated
most likely to have? blood pressure for years which has not been treated.
A. Protein S deficiency He now has severe headaches. On physical
Incorrect. Coagulopathies with examination his blood pressure is 275/150 mm Hg.
thrombosis may be due to a congenital Laboratory studies show Hgb 13.8 g/dL, serum glucose
anticoagulant factor deficiency, but 76 mg/dL, and creatinine 3.5 mg/dL. These findings are
thrombotic events are usually sudden. most likely to be associated with which of the following
B. Blood culture with Staphylococcus pathologic lesions involving his kidneys?
aureus A. Hyperplastic arteriosclerosis
Incorrect. Septic thrombosis and mycotic CORRECT. Hyperplastic
aneurysms may be the result of arteriolosclerosis is seen in the setting of
septicemia, but this is an acute illness. hypertensive emergency when systolic
C. Decreased arterial oxygen saturation pressure is >=180 and/or diastolic
Incorrect. Oxygen saturation is typically pressure >=120 mmHg along with signs
dependent more upon pulmonary of acute or ongoing end-organ damage,
function and is not related to focal events, and renal failure is common.
as in the lower extremities. B. Hyaline arteriosclerosis
D. Hyperglycemia Incorrect. These are complications
CORRECT. This is peripheral arterial typically seen in association with
vascular disease from severe diabetes mellitus.
atherosclerosis, which is promoted by C. Monckeberg medial calcific sclerosis
diabetes mellitus. The absence of pulses Incorrect. This is a histologic curiosity
defines this as an arterial process, as with no major clinical importance.
does the claudication (pain with exercise). D. Atherosclerosis
Thrombophlebitis is a venous process Incorrect. Hypertension is a risk for
and leads to swelling and pain in the leg, coronary artery disease, but a
but not loss of pulses. hypertensive emergency is not
E. Hypercalcemia specifically linked to coronary thrombosis.
Incorrect. Calcium levels have little to do E. Thrombophlebitis
with vascular problems in general Incorrect. Venous thrombosis is most
(except for the rare condition in persons often a result of stasis from
with renal failure called calciphylaxis). immobilization.

A 72-year-old man suffered a myocardial infarction A 60-year-old woman has become increasingly
involving half the left ventricular free wall 3 months ago. obtunded over the past day. She was found by her
He now has increasing dyspnea and orthopnea. On daughter in a stuporous condition and brought to the
echocardiography his ejection fraction is 29%. On emergency department. On physical examination, she
examination he has poor capillary filling in hands and has poor skin turgor. She is afebrile. Her vital signs
feet. A chest x-ray shows pulmonary edema. Which of reveal a blood pressure of 90/40 mm Hg, respirations
the following laboratory test analytes is most likely to 15 and shallow, pulse 95, and temperature 36°C.
be increased in this man at this point in time? Laboratory studies show a hemoglobin A1C of 10%.
A. Sodium Her serum electrolytes show sodium 144 mmol/L,
Incorrect. The serum sodium may give an potassium 5 mmol/L, chloride 95 mmol/L, CO2 22
indication of the degree of hydration, not mmol/L, and glucose 940 mg/dL. Which of the following
tissue perfusion. is the most likely diagnosis?
B. Creatine kinase A. Insulin overdose
Incorrect. Poor perfusion leading to Incorrect. An overdose of insulin would
myocardial infarction would increase the lead to hypoglycemia.
creatine kinase. B. Hyperosmolar coma
C. Natriuretic peptide CORRECT. This is an extremely
CORRECT. With congestive heart failure, elevated serum glucose that would
there is volume overload that triggers an increase the serum osmolality markedly.
increase in natriuretic peptide, typically
measured as the b-type (BNP). This
hormone normally leads to natriuresis,
Atherosclerosis and Thrombosis | Pathology Reviewer (Utah) | 12 / 72
C. Hyperlipidemia C. Inflammation with neutrophils
Incorrect. Hyperlipidemia can be due to Incorrect. Neutrophilic inflammation can
either an increased serum cholesterol or be the result of infection and is unusual in
serum triglyceride. arteries. It is not related to
D. Ketoacidosis atherosclerosis.
Incorrect. Diabetics with ketoacidosis
may have a serum glucose around 500 to D. Deposition of immunoglobulins
600 mg/dL, but generally not over 1000 Incorrect. There can be T-cell activation
mg/dL. with atherogenesis, but B-cells do not
E. Overeating play a significant role.
Incorrect. You can't get an extremely high E. Activation of complement
serum glucose just from oral intake alone. Incorrect. The process of atherogenesis
proceeds slowly, without significant
A study is performed involving persons who have a inflammation and activation of
history of diabetes mellitus type 1 or type 2. These complement.
patients are found to have cellular injury that results
from glycosylation end products and from sorbitol A 65-year-old man has had increasing lower leg
accumulation within cells. The same patients are also swelling along with reduced exercise tolerance for the
shown to have ischemic tissue damage from past 5 years. He has chest pain on exertion. He has
accelerated and advanced atherosclerosis. Which of not experienced dyspnea. He has had 4 transient
the following complications is most likely to result from ischemic attacks in the past year. He has experienced
atherosclerosis in these patients? abdominal pain in the past 2 months. Vital signs show
A. Hepatic failure T 37°C, P 82/min, RR 16/min, and BP 130/85 mm Hg.
Incorrect. The liver is not primarily He has pitting edema to the knees bilaterally. The lower
affected by atherosclerosis, as it has a extremities have palpable pulses, no tenderness, and
dual blood supply. no erythema. An abdominal CT scan shows dilation of
B. Impotence the abdominal aorta to 5 cm, filled with mural thrombus.
Incorrect. The neuropathy that occurs Other family members have had similar problems.
with diabetes mellitus can affect penile Which of the following underlying conditions is the most
erection in men. likely risk factor for his findings?
C. Stroke A. Factor V Leiden mutation
CORRECT. Stroke from cerebral Incorrect. This is a cause for
atherosclerosis or from embolization of hypercoagulability, usually manifesting at
mural thrombi from a heart involved with an early age, with a risk mainly for
ischemic injury as a consequence of venous thrombosis, not atherosclerosis.
atherosclerosis. B. Adenocarcinoma of the colon
D. Cataracts Incorrect. Malignancies can be
Incorrect. Cataracts involving the associated with a hypercoagulable state
crystalline lens of the eye are seen more with thrombosis, but not manifestations
frequently in diabetics. of atherosclerosis. In fact, persons with
E. Retinopathy malignancies typically have a regression
Incorrect. Blindness from diabetic of atherosclerotic lesions.
retinopathy is a frequent long term C. Multiple blunt trauma
complication of diabetes mellitus. Incorrect. Trauma can cause
hemorrhage, not atherosclerosis. The
A 30-year-old man goes to his physician for a routine 'trauma' to arterial endothelium that
health checkup. On physical examination there are no promotes atherosclerosis is from altered
abnormal findings. Laboratory test findings include blood flow at a microscopic level, not
serum glucose 80 mg/dL, hemoglobin A1C 4%, total external trauma.
cholesterol 240 mg/dL, LDL cholesterol 180 mg/dL, D. Vasculitis
and HDL cholesterol 20 mg/dL. Through which of the Incorrect. Vasculitis does not typically
following mechanisms is endothelial vascular injury in lead to findings in a distribution
this patient most likely to occur? associated with atherosclerosis. Some
A. Accumulation of sorbitol forms of vasculitis favor thrombosis,
Incorrect. The accumulation of sorbitol in others occlusion.
tissues not requiring insulin for glucose E. Metabolic syndrome
uptake is a feature of diabetes mellitus CORRECT. He has findings associated
with hyperglycemia. with atherosclerosis, including coronary
B. Insudation of lipid and aortic disease at least, with
CORRECT. He has congestive heart failure and evidence for
hypercholesterolemia with more of the an aortic aneurysm. Metabolic syndrome
'bad' LDL cholesterol that can become predisposes to atherosclerosis. Some
oxidized and taken up by modified patients may eventually develop overt
arterial wall LDL receptors. The lipid diabetes mellitus type 2.
collects in macrophages that transform to
foam cells. These foam cells accumulate
and becomes a lipid lesion--the precursor
to more serious atheromatous plaques.
Atherosclerosis and Thrombosis | Pathology Reviewer (Utah) | 13 / 72
A study of persons with essential hypertension is Following a meal, lipids are digested and absorbed.
performed. They have blood pressures in the range of Lipids collect within enterocytes. Which of the following
140/90 to 160/100 mm Hg. Laboratory studies show chemical components of the blood is mainly
normal serum electrolyte values. Which of the following responsible for transporting exogenous (dietary)
physiologic abnormalities is most likely to account for triglyceride from the intestine to the liver?
their hypertension? A. Apoprotein
A. Hyperreninemia Incorrect. There are a number of
Incorrect. Increased renin is likely to apoproteins involved in transport of lipids
occur with renal arterial abnormalities in the blood, but they are not the primary
that reduce renal blood flow. means of transport of dietary lipids to the
B. Hyperaldosteronism liver.
Incorrect. Increased aldosterone may be B. Chylomicron
elaborated by an adrenal adenoma, and CORRECT. Chylomicrons formed in
it is increased in response to increased intestinal epithelial cells contain
renin. apoproteins, triglyceride and cholesterol.
C. Decreased natriuretic peptide C. Lipoprotein lipase
Incorrect. Though natriuretic peptide is Incorrect. Lipoprotein lipase in
likely not to be elevated with endothelium hydrolyzes chylomicrons,
hypertension, this is not the primary liberating fatty acids for use by fat and
mechanism for the hypertension. muscle. The cholesterol-rich chylomicron
D. Decreased angiotensin converting remnants go to the liver.
enzyme D. Oxidized low density lipoprotein
Incorrect. ACE is needed to convert Incorrect. Such LDL is injurious to
angiotensin I to angiotensin II, to endothelium and is more avidly taken up
increase blood pressure. ACE inhibitors by macrophages and smooth muscle,
are used to treat hypertension. thus promoting atherogenesis in vessels.
E. Sodium retention E. High density lipoprotein
CORRECT. The causes for essential Incorrect. HDL is 'good' because it
hypertension are obscure. The 'set point' transports cholesterol back to the liver.
for sodium diuresis is probably increased
at a higher blood pressure in affected Some cells demonstrate glucose uptake from the blood
persons. Diuretics that promotes regardless of the plasma insulin level. In persons who
natriuresis are effective in treating have had persistent hyperglycemia for years, cellular
essential hypertension. injury can occur. Which of the following cell types is
F. Catecholamine excess most likely to show injury from hyperglycemia?
Incorrect. Increased catecholamines may A. Cardiac myocytes
occur with pheochromocytomas. Incorrect. Cardiac myocytes are striated
muscle cells that require insulin for
A 27-year-old woman had a hemoglobin A1C of 7.9% glucose uptake.
noted during a prenatal visit. She gives birth to a 4350 B. Fibroblasts
gm baby at 37 weeks gestation. Just after the delivery, Incorrect. Fibroblasts have GLUT4
the baby becomes irritable and displays seizure activity. receptors responsive to insulin for
Which of the following laboratory findings is most likely glucose uptake, but the effect is less
to be found in the baby: pronounced than for adipocytes.
A. Decreased hemoglobin A1C C. Steatocytes
Incorrect. The hemoglobin A1C is Incorrect. Fat cells require insulin for
typically increased in diabetics with poor glucose uptake. Steatocytes also are part
control. of the endogenous pathway of lipoprotein
B. Increased serum osmolality transport and convert VLDL to LDL.
Incorrect. Hyperosmolar coma with a D. Neurons
markedly elevated serum glucose is a CORRECT. Neurons do not require
feature of type II diabetes mellitus in insulin for glucose uptake. The excess
adults. glucose diffusing into the cells is shunted
C. Hyperlipidemia into the sorbitol pathway and can lead to
Incorrect. Hyperlipidemia in neonates is osmotic injury, resulting in neuropathy.
unlikely to occur. E. Smooth muscle cells
D. Hypoglycemia Incorrect. Smooth muscle requires
CORRECT. Infants of diabetic mothers insulin for glucose uptake.
are at risk for hypoglycemia following
delivery, because they have islet During hospitalization, a 40-year-old woman develops
hypertrophy and hyperplasia, and the thrombophlebitis. She recovers and is discharged. She
increased islet production of insulin returns to her job as an electrician. A couple of months
carries into the immediate postpartum later, which of the following terms would best describe
period. the process seen in a femoral vein after recovery from
E. Ketoacidosis her thrombophlebitis:
Incorrect. Ketoacidosis is typical for type A. Acute inflammation
I diabetes in which there is an absolute Incorrect. Some inflammation, possibly
insulin lack. acute, may have been present at the time
Atherosclerosis and Thrombosis | Pathology Reviewer (Utah) | 14 / 72
of the original episode of C. Heart rate of 100/minute
thrombophlebitis. Incorrect. Tachycardia may be due to
B. Rupture many causes but by itself does not
Incorrect. Thrombophlebitis is a process indicate atherosclerotic disease.
best known for thrombus formation, and D. Leg swelling
rupture of a vein wall would be an Incorrect. Leg swelling is most indicative
extreme rarity. of deep venous thrombosis, not
C. Embolization atherosclerotic vascular narrowing.
Incorrect. After she is mobile again and Wrong end of body.
months pass, any thrombus capable of E. Pulsatile abdominal mass
embolizing is long gone. Incorrect. A pulsatile abdominal mass is
D. Organization most likely to indicate an atherosclerotic
CORRECT. Thrombi may either lyse or aortic aneurysm, which leads to
will organize over time, with much of the complications in the lower half of the
clot eventually removed and the vascular body.
lumen restored. F. Radial pulse 1+
E. Propagation Incorrect. Reduced pulses may be due to
Incorrect. Propagation of a thrombus is atherosclerosis with vascular narrowing.
most likely to occur at the time it originally The radial pulse is in the arm.
forms, or soon thereafter.
A 70-year-old healthy woman has a check of her health
A 52-year-old man has an ulcerated area on the sole status. On examination her blood pressure is 125/80
of his foot that has not healed for 2 months. He is 180 mm Hg and BMI 24 kg/m2. Laboratory studies show her
cm tall, weighs 126 kg, and has continued to gain serum total cholesterol is 180 mg/dL with LDL
weight gradually. He has not had any major illnesses. cholesterol 90 mg/dL. There is a family history of
His blood pressure is normal. Which of the following cancer. Which of the following is a constitutional risk
laboratory tests performed on serum from a blood factor for atherosclerotic vascular disease in this
sample would be most useful in elucidating the woman?
underlying cause for his problem: A. Age
A. Antithrombin III CORRECT. Her age is a risk factor for
Incorrect. ATIII may be diminished in atherosclerosis. However, a healthy
some cases of recurrent thrombosis. lifestyle can negate this risk. Many older
B. Cortisol persons are not pursuing a healthy
Incorrect. Cortisol may be decreased lifestyle.
with adrenal failure. Increased cortisol B. Blood pressure
with Cushing syndrome may result in Incorrect. Her blood pressure is not
weight gain, but signs of severe elevated. If it were, it could be modified
peripheral atherosclerosis are lacking. by lifestyle changes and by
C. Creatine kinase pharmacologic therapy.
Incorrect. CK is a good indicator for injury C. Family history
to muscle, and cardiac ischemia or Incorrect. She does not have a family
infarction in particular, which may be a history of atherosclerotic disease.
consequence of his underlying disease. Cancer does not count.
D. Glucose D. Gender
CORRECT. The history suggests Incorrect. Her female gender reduces the
diabetes mellitus, most likely type II, and risk.
persistent hyperglycemia would confirm E. Hyperlipidemia
the diagnosis. Incorrect. Her cholesterol levels are in
E. Carcinoembryonic antigen the normal range.
Incorrect. CEA can be a marker for some F. Weight
malignancies. Incorrect. Her BMI is in the normal range.

A 71-year-old woman experiences a transient ischemic


attack (TiA). She had an acute myocardial infarction 5
years ago. Which of the following physical examination
findings is most likely to be associated with her TIA?
A. Blood pressure 140/90 mmHg
Incorrect. A blood pressure of 140/90 is
mild hypertension, which is a risk factor
for atherosclerosis, but not directly
related to the TIA.
B. Carotid bruit
CORRECT. Atheromatous plaques tend
to form initially where there is vascular
turbulence, and a bruit is indicative of
arterial narrowing at the carotid
bifurcation. The turbulence drives
endothelial injury that contributes to
atheroma formation.
CELLULAR INJURY SECTION

A 48-year-old woman has a malignant lymphoma E. Blebs form on cell membranes


involving lymph nodes in the para-aortic region. She is Incorrect. Such membrane blebs
treated with a chemotherapeutic agent which results in constitute a reversible form of cellular
the loss of individual neoplastic cells through injury.
fragmentation of individual cell nuclei and cytoplasm.
Over the next 2 months, the lymphoma decreases in While in a home improvement center warehouse
size, as documented on abdominal CT scans. By which buying paint, a 35-year-old man hears 'Look out below!'
of the following mechanisms has her neoplasm and is then struck on the leg by a falling pallet rack,
primarily responded to therapy? which strikes him on his left leg in the region of his thigh.
A. Coagulative necrosis The skin is not broken. Within 2 days there is a 5 x 7
Incorrect. When parts of an organ cm purple color to the site of injury. Which of the
undergo necrosis in a vascular following substances has most likely accumulated at
distribution from loss of blood supply with the site of injury to produce a yellow-brown color at the
resultant ischemia, then an infarct occurs. site of injury 16 days later?
B. Mitochondrial poisoning A. Lipofuscin
Incorrect. The loss of mitochondrial Incorrect. This 'wear and tear' pigment
activity, as can occur with calcium influx builds up in cells of tissues (such as
with hypoxia, does not signal apoptosis. myocardium and liver) over many years'
C. Phagocytosis time.
Incorrect. Phagocytosis is the process of B. Bilirubin
cells or cellular debris being consumed Incorrect. A generalized yellow color,
by macrophages. Apoptotic bodies may jaundice, can be the result of
be consumed in this fashion. hyperbilirubinemia.
D. Acute inflammation C. Melanin
Incorrect. Acute inflammatory cells Incorrect. Melanin pigment darkens the
(neutrophils) can release enzymes that skin as concentrations of it increase from
destroy other cells and intracellular sunlight exposure (tanning).
matrix, but this is a highly non-selective D. Hemosiderin
process. CORRECT. The iron in the heme
E. Apoptosis pigment from the red blood cells in the
CORRECT. The induction of individual hemorrhage beneath the skin is
cell death occurs in the process of incorporated into hemosiderin granules
apoptosis. The drug effect is targeted that impart the yellow to brown color of
primarily at the neoplastic cells, not the healing contusion (bruise) from blunt
normal cells. force trauma.
E. Glycogen
A 53-year-old man has experienced severe chest pain Incorrect. Glycogen is stored in striated
for the past 6 hours. On physical examination he is muscle and liver and imparts no grossly
afebrile, but has tachycardia. Laboratory studies show visible color change.
a serum troponin I of 10 ng/mL. A coronary angiogram
is performed emergently and reveals >90% occlusion A 54-year-old man with a chronic cough has a
of the anterior interventricular (left anterior descending) squamous cell carcinoma diagnosed in his right lung.
artery. In this setting, an irreversible injury to While performing a pneumonectomy, the thoracic
myocardial fibers will have occurred when which of the surgeon notes that the hilar lymph nodes are small, 0.5
following cellular changes occurs? to 1.0 cm in size, and jet black in colour throughout.
A. Glycogen stores are depleted Which of the following is the most likely cause for this
Incorrect. Glycogen depletion within the appearance to the hilar nodes?
cell can be reversed. A. Anthracotic pigment
B. Cytoplasmic sodium increases CORRECT. The black colour comes from
Incorrect. An increase in cellular sodium carbon pigments in dust particles inhaled
(and the water that follows) produces over the years, engulfed by
edema with cellular swelling, but this is a macrophages, and sent via lymphatics to
potentially reversible form of injury. the lymph nodes. It looks bad but does
C. Nuclei undergo karyorrhexis not compromise lung function. Smokers
CORRECT. Chromatin clumping is will have more anthracosis.
reversible, but dissolution of the entire B. Lipochrome deposits
nucleus is not, and when the nucleus is Incorrect. Lipofuscin pigment builds up
lost, the cell will die. An acute coronary slowly within cell cytoplasm as a result of
syndrome with myocardial infarction is internal cellular changes. It is not an
described here. exogenous pigment.
D. Intracellular pH diminishes C. Melanin accumulation
Incorrect. Acidosis by itself is reversible. Incorrect. Melanin is made by cells
(melanocytes) and is not an exogenous
pigment.

Page 15 of 72
Cellular Injury | Pathology Reviewer (Utah) | 16 / 72
D. Hemosiderosis D. Cholesterol crystals
Incorrect. Hemosiderin is a storage form CORRECT. An inflammatory mass
of iron and is not an exogenous pigment. persisted, with cellular necrosis, and the
E. Metastatic carcinoma lipid from the cell membranes is broken
Incorrect. Metastatic lesions are typically down and cholesterol crystals form. The
firm and white. boy has the complication of otitis media
known as a cholesteatoma
A 50-year-old woman with a history of unstable angina E. Anthracotic pigment
suffers an acute myocardial infarction. Thrombolytic Incorrect. Anthracosis results from
therapy with tissue plasminogen activator (tPA) is accumulation within the lung and lymph
administered to restore coronary blood flow. In spite of nodes of exogenous carbon pigment
this therapy, the degree of myocardial fiber injury may from dusts in the air that we all breathe.
increase because of which of the following cellular
abnormalities? A 43-year-old man has complained of mild burning
A. Cytoskeletal intermediate filament loss substernal pain following meals for the past 3 years.
Incorrect. Cytoskeletal abnormalities Upper GI endoscopy is performed and biopsies are
occur with ischemic injury, but do not taken of an erythematous area of the lower esophageal
worsen with reperfusion. mucosa 3 cm above the gastroesophageal junction.
B. Decreased intracellular pH from There is no mass lesion, no ulceration, and no
anaerobic glycolysis hemorrhage noted. The biopsies show the presence of
Incorrect. Acidosis is part of ischemia, columnar epithelium with goblet cells. Which of the
but does not specifically cause a following mucosal alterations is most likely represented
reperfusion injury. by these findings?
C. Increased free radical formation A. Dysplasia
CORRECT. Such toxic oxygen radicals Incorrect. The growth of the epithelial
are released from neutrophils when cells must become disordered in order to
blood flow is restored following ischemia. be dysplastic.
This is a reperfusion injury. Overall, there B. Hyperplasia
is likely to be more good than harm to Incorrect. Hyperplasia may occur with
restoration of blood flow. inflammation, as the number of cells
D. Mitochondrial swelling increases, but hyperplasia does not
Incorrect. Mitochondria swell with explain the presence of the columnar
ischemic injury, but this is reversible. cells.
E. Nuclear chromatin clumping C. Carcinoma
Incorrect. This is a reversible change that Incorrect. A carcinoma has the cellular
may occur with ischemia. atypia with hyperchromatism and
F. Reduced protein synthesis pleomorphism. Goblet cells would not be
Incorrect. This is a reversible change that seen.
may occur with ischemia. D. Ischemia
Incorrect. Ischemia would be unusual at
A 12-year-old boy has had multiple episodes of ear this site and would be marked by
pain accompanied by fever. On examination his right coagulative necrosis.
tympanic membrane is red and bulging with yellow E. Metaplasia
exudate. Laboratory studies of the exudate show CORRECT. Metaplasia is the
culture positive for Hemophilus influenzae. A year later substitution of one tissue normally found
he has conductive hearing loss on the right, and a head at a site for another. The esophageal
CT scan shows a mass in the right middle ear. Which stratified squamous epithelium
of the following materials is most likely to be seen in undergoes metaplasia in response to the
the tissue curetted from his middle ear? ongoing inflammation from reflux of
A. Lipofuscin pigment gastric contents. This is common in the
Incorrect. Accumulation of lipofuscin is lower esophagus with gastroesophageal
not a function of hemorrhage or reflux disease (GERD).
inflammation, but of collection of cellular
breakdown products, often seen with A 71-year-old woman had the loss of consciousness
aging, and most often in hepatocyte or that persisted for over an hour. When she becomes
myocardial fiber cytoplasm. arousable, she cannot speak nor move her right arm.
B. Russell bodies A cerebral angiogram revealed an occlusion to her left
Incorrect. Russell bodies are intracellular middle cerebral artery. Months later, a computed
accumulations of immunoglobulins in tomographic (CT) scan shows a large 5 cm cystic area
plasma cells. in her left parietal lobe cortex. This CT finding is most
C. Neutrophil granules likely the consequence of resolution from which of the
Incorrect. Neutrophils suggest acute following cellular events?
inflammation more typical for acute otitis A. Liquefactive necrosis
media that accompanied each episode of CORRECT. She had a 'stroke' with
acute otitis media, not the complication cerebral infarction and loss of brain
this boy now has. Neutrophil granules are tissue. The brain undergoes liquefactive
released and dissipate readily. necrosis with infarction. As it resolves,
macrophages remove the dead cells and
Cellular Injury | Pathology Reviewer (Utah) | 17 / 72
debris, leaving a cystic area that forms in there are small amounts of it, and it has
the region of infarction. little effect upon cardiac function. The
B. Atrophy 'brown atrophy' of the heart in this case is
Incorrect. Atrophy would be a more a rare finding.
generalized process, whereas a single C. Glycogen from a storage disease
cystic area in brain suggests a remote Incorrect. Glycogen storage diseases are
infarction. inherited conditions that usually appear
C. Coagulative necrosis early in life. Glycogen does not appear
Incorrect. Coagulative necrosis is more pigmented in H&E stained tissue
typical of parenchymal organs such as sections.
kidney or spleen that do not have as high D. Cholesterol from atherosclerosis
a lipid content as brain. Incorrect. The cholesterol accumulates in
D. Caseous necrosis atheromatous plaques in the arteries, not
Incorrect. Caseous necrosis is more in myocardium.
typical of granulomatous inflammation E. Calcium deposition following necrosis
with M. tuberculosis. Incorrect. Calcium deposits appear as
E. Apoptosis irregular dark blue areas and are not
Incorrect. This is single cell necrosis that associated with aging of the myocardium.
does not result in a grossly visible cystic
area. In an experiment, a series of immunohistochemical
stains are employed to identify different cellular
A 19-year-old woman gives birth to her first child. She components. One particular stain identifies the
begins breast feeding the infant. She continues breast presence of intermediate filaments within cells. This
feeding for almost a year with no difficulties and no cytokeratin stain is most likely to be useful for which of
complications. Which of the following cellular the following diagnostic purposes?
processes that began in the breast during pregnancy A. Cytoskeletal alterations indicate
allowed her to nurse the infant for this period of time? impending cell death
A. Stromal hypertrophy Incorrect. Cytoskeletal alterations occur
Incorrect. The stroma of the breast with ischemia, but are not a useful marker
consists of connective tissue that for such an event.
provides structural support, but does not B. A neoplasm is determined to be a
have cells that produce milk. carcinoma
B. Epithelial dysplasia CORRECT. Carcinomas are derived
Incorrect. Dysplasia in epithelia is a pre- from epithelium and contain cytokeratins,
malignant change not part of normal while sarcomas derived from
physiologic events. This would be a mesenchymal cells contain vimentin.
precursor to breast cancer. Pregnancy is C. Contractile properties of the cell can be
a protective factor against breast cancer. assessed
C. Steatocyte atrophy Incorrect. Many cell types contain
Incorrect. With poor nutrition and weight intermediate filaments.
loss, steatocytes can decrease in size. D. A history of chronic alcoholism can be
However, she loves her baby and will confirmed
take care of herself. Incorrect. Mallory's alcoholic hyaline can
D. Ductal epithelial metaplasia be observed by H&E staining. However,
Incorrect. Metaplasia is the exchange of it is not entirely specific for alcoholism.
the normal epithelium for another in E. The degree of metaplasia or dysplasia
response to chronic irritation. Metaplasia can be assessed
is not a normal physiologic process. Incorrect. Metaplasia and dysplasia are
E. Lobular hyperplasia assessed by light microscopic
CORRECT. There is an increase in the appearances with H&E staining.
breast lobules under hormonal influence,
primarily progesterone with pregnancy, A 20-year-old woman had Goodpasture syndrome
to provide for lactation. which progressed to chronic renal failure. She is 165
cm tall and weighs 55 kg. She now has blood pressure
An 84-year-old man dies from complications of measurements in the range of 150/90 to 180/110 mm
Alzheimer disease. At autopsy, his heart is small (250 Hg, but does not regularly take medications.
gm) and dark brown on sectioning. Microscopically, Laboratory studies show her blood urea nitrogen is
there is light brown perinuclear pigment with H&E over 100 mg/dL and she requires chronic dialysis. A
staining of the cardiac muscle fibers. Which of the chest x-ray shows an enlarged heart. The size of her
following substances is most likely increased in the heart is most likely to be the result of which of the
myocardial fibers to produce this appearance of his following processes involving the myocardial fibers?
heart? A. Hypertrophy
A. Hemosiderin from iron overload CORRECT. The ongoing pressure load
Incorrect. Hemosiderosis is not a of the systemic hypertension led to
complication of aging. myocardial fiber hypertrophy and a heart
B. Lipochrome from ‘wear and tear’ that increased in size. Renal failure often
CORRECT. Lipochrome (lipofuscin) leads to hypertension, and vice versa.
deposition in myocardial cell cytoplasm is
a common finding, though ordinarily
Cellular Injury | Pathology Reviewer (Utah) | 18 / 72
B. Fatty infiltration There is no change of one cell type to
Incorrect. Fat in the heart does not another involved.
increase in response to the increase in B. Lactation following pregnancy
work load from hypertension. Incorrect. This is a form of physiologic
C. Hyperplasia hyperplasia of breast lobules from
Incorrect. Myocardial fibers do not hormonal influence.
undergo a significant degree of C. Vitamin A deficiency
hyperplasia. CORRECT. Vitamin A is necessary to
D. Fatty degeneration maintain epithelia, and squamous
Incorrect. Fatty degeneration of metaplasia of the respiratory tract may
myocardium is typically the result of a occur if there is vitamin A deficiency. The
toxic or hypoxic injury. stratified squamous epithelium does not
E. Edema function as well as the normal
Incorrect. Myocardial edema is not a pseudostratified columnar respiratory
characteristic feature of myocardial injury epithelium, and there is an increased risk
or increased work load. However, heart for respiratory infections.
failure could lead to peripheral edema. D. Acute myocardial infection
Incorrect. An acute MI will lead to cardiac
A 29-year-old man goes on a snorkeling trip to Looe muscle fiber necrosis, which will heal with
Key Marine Sanctuary and later spends time on the production of fibrous scar tissue, but this
beach at Bahia Honda State Park. The next day he has is not a reversible metaplasia.
a darker complexion. His skin does not show warmth, E. Urinary obstruction from an enlarged
erythema, or tenderness. His skin tone fades to its prostate
original appearance within a month. Which of the Incorrect. The enlarged prostate
following substances contributes the most to the represents primarily glandular
biochemical process leading to these skin changes? hyperplasia.
A. Iron oxide
Incorrect. Heme containing iron as part of In an experiment, a disease process is found which
hemosiderin from breakdown of red leads to scattered loss of individual cells, with the
blood cells can impart a brownish colour, microscopic appearance of karyorrhexis and cell
but this is typically local from trauma or fragmentation. The overall tissue structure remains
more global as part of an iron storage intact. This process is most typical for which of the
disease such as hemochromatosis. Iron following diseases?
oxide is also used in tattoo pigments. A. Viral hepatitis
B. Lipofuscin CORRECT. Viral infection leads to
Incorrect. Lipochrome (lipofuscin) is a apoptosis with individual hepatocyte
'wear and tear' pigment imparting a necrosis, either from effects of viral
golden brown appearance to granules in replication or from the body's cell
cells (such as myocytes or hepatocytes), mediated immune response with
but this is not a feature of skin. cytotoxic T lymphocytes.
C. Tyrosine B. Brown atrophy of the heart
CORRECT. The tanning process in skin Incorrect. Brown atrophy of the heart
is stimulated by ultraviolet light exposure. results when there is marked lipofuscin
Melanocytes have the enzyme deposition in the myocardium.
tyrosinase to oxidize tyrosine to C. Renal transplant rejection
dihydroxyphenylalanine in the pathway Incorrect. Tissue destruction with
for melanin production. Natural skin tone transplant rejection is more widespread.
is related to the level of tyrosinase D. Chronic alcoholic liver disease
enzyme activity. Incorrect. Single cell necrosis is not
D. Homogentisic acid evident in chronic alcoholic liver disease.
Incorrect. Homogentisic acid can be part E. Barbiturate overdose
of the process of the rare disease Incorrect. Barbiturates produce
alkaptonuria, in which a black pigment is hypertrophy of smooth endoplasmic
deposited in connective tissues. reticulum, not individual cell necrosis.
E. Glycogen 'Barbs' have gone out of style as drugs of
Incorrect. Glycogen in quantity is starch- abuse.
like and imparts a paler colour to organs
in which it is stored in excess. It does not A 60-year-old woman has noted a dark red-black
involve the skin. appearance to her great toe and second and third toes
of her left foot for the past month. On physical
A study is performed to identify predisposing risks for examination, the toes are cold and have no sensation
tissue cellular changes. In some persons epithelial to touch. The dorsalis pedis and posterior tibial pulses
metaplasia occurs. In which of the following situations are not palpable on the left. A transmetatarsal
is the process of epithelial metaplasia most likely to amputation is performed. These findings are most
take place? typical for a patient with which of the following
A. Tanning of the skin following sunlight conditions?
exposure A. Diabetes mellitus
Incorrect. This physiologic event results CORRECT. Gangrenous necrosis is
from accumulation of melanin pigment. described. Occlusive peripheral
Cellular Injury | Pathology Reviewer (Utah) | 19 / 72
atherosclerotic vascular disease is B. Small intestinal epithelium
typical for diabetes mellitus. Many Incorrect. The intestinal mucosa is
arteries are involved, reducing collateral actively proliferating and sensitive to
flow. This is diabetic gangrene. radiation.
B. Gout C. Erythropoietic cells of the bone marrow
Incorrect. Gout can involve joints, and Incorrect. RBC's are constantly being
there can be erosion of bone, but less replaced by precursors that are
likely overlying ulceration. Gout does not proliferating, so they are radiosensitive.
affect circulation. D. Spermatogonia of testicular tubules
C. Blunt force trauma Incorrect. Germ cells are very
Incorrect. Such focal gangrenous radiosensitive.
necrosis is not typical for trauma, unless E. Neurons of the cerebral cortex
blood supply is disrupted, but then the CORRECT. Neurons are terminally
necrosis should proceed quickly. differentiated cells that do not actively
D. AIDS divide or proliferate, so at therapeutic
Incorrect. The complications of AIDS are doses of radiation, they are relatively
unlikely to lead to gangrenous necrosis. radioresistant. However, at higher
E. Rheumatoid arthritis radiation doses, cerebral injury does
Incorrect. Rheumatoid arthritis can cause occur from damage mainly to white
joint destruction, but not overlying skin matter and to the vasculature.
ulceration. An abnormal immune reaction,
such as a type III hypersensitivity A 79-year-old man has a large myocardial infarction
reaction with antigen-antibody complex involving much of the left ventricular free wall. He
deposition, could produce focal vasculitis, develops congestive heart failure (CHF) with
but would not typically result in infarction decreased cardiac output. Now, a year later, his CHF
of a large area or body part. is worsening. By echocardiography there is a large,
bulging akinetic area typical for a left ventricular
A study is performed involving the microscopic analysis aneurysm. Which of the following laboratory tests on
of tissues obtained from surgical procedures. Some of serum would best indicate poor peripheral tissue
these tissues have the microscopic appearance of an perfusion in this patient?
increased cell size of multiple cells within the tissue, A. Elevated troponin I
due to an increase in the amount of cell cytoplasm, with Incorrect. An elevated troponin suggests
nuclei remaining uniform in size. Which of the following acute to intermediate myocardial
conditions is most likely to have resulted in this finding? ischemic injury, but he does not have
A. Uterine myometrium in pregnancy another myocardial infarction at this time.
CORRECT. This cellular hypertrophy B. Increased sodium
with increase in cell size (not hyperplasia Incorrect. Sodium retention occurs with
with increase in cell number) accounts for CHF, but the serum sodium is not
most of the marked increase in size of the significantly increased.
uterus during pregnancy. Following C. Elevated lactate
pregnancy and reduction in hormonal CORRECT. Under conditions of poor
stimulation, the uterus returns back to its tissue perfusion, there will be more
normal size. anaerobic glycolysis and more acidosis
B. Female breast at puberty in cells throughout the body. The blood
Incorrect. This is a form of physiologic lactate rises in this condition.
hyperplasia with proliferation of ducts D. Increased hematocrit
and lobules leading to an increase in Incorrect. Hypoxia can lead to an
breast size. increased red blood cell mass over a long
C. Liver following partial resection period of time, but this is not a typical
Incorrect. This is a form of compensatory adaptation to CHF.
hyperplasia. E. Increased sedimentation rate
D. Ovary following menopause Incorrect. The sedimentation rate
Incorrect. Following menopause, the increases with inflammatory conditions,
ovary undergoes atrophy and decreases not with CHF.
in size.
E. Cervix with chronic inflammation A 22-year-old woman has a congenital anemia. She
Incorrect. The inflamed epithelium has required multiple transfusions of red blood cells for
undergoes metaplasia. many years. She now has no significant findings on
physical examination. Which of the following
A 17-year-old adolescent receives whole body microscopic findings would most likely present in her
radiation as part of a preparatory regimen for bone liver?
marrow transplantation to treat acute lymphocytic A. Steatosis in hepatocytes
leukemia. Which of the following cells and tissues in the Incorrect. This would more likely be a
body is most likely to remain unaltered by the effects of feature seen with ingestion of great
this therapeutic radiation? amounts of alcohol.
A. Primary follicles of the ovary B. Bilirubin in canaliculi
Incorrect. Germ cells are very Incorrect. The accumulation of iron does
radiosensitive. not lead to biliary obstruction or
dysfunction.
Cellular Injury | Pathology Reviewer (Utah) | 20 / 72
C. Hemosiderin in hepatocytes B. Dry gangrene
CORRECT. There is 250 mg of iron in Incorrect. Gangrenous necrosis refers to
each unit of blood. The body has no destruction of an organ or body part. Dry
mechanism for getting rid of excess iron. gangrene typically describes a process
A small amount of iron is lost with normal involving an extremity such as the foot.
desquamation of epithelia, and C. Fat necrosis
menstruating women will lose a bit more. CORRECT. The enzymes released from
The excess iron becomes storage iron, or the pancreas with acute pancreatitis
hemosiderin. Over time, hemosiderosis damage the surrounding fat and form
involves more and more tissues of the soaps -- localized soft tan to yellow areas
body, particularly the liver. of fat necrosis. The damaged pancreatic
D. Glycogen in hepatocytes exocrine cells release lipase as a marker
Incorrect. Glycogen accumulation can be for their injury.
seen in a variety of inherited glycogen D. Apoptosis
storage diseases in which the lack of an Incorrect. Apoptosis describes
enzyme leads to accumulation of a destruction of single cells, not whole
storage product. tissues.
E. Amyloid in portal triads E. Liquefactive necrosis
Incorrect. Amyloid deposition occurs Incorrect. Liquefactive necrosis is more
when there is an excessive production of typical for damage to an organ such as
a protein that is broken down by the brain with abundant lipid. Liquefactive
proteases to an insoluble end product necrosis is seen in abscesses as well.
known as amyloid that can collect in
various tissues. A 26-year-old man has had a high fever for the past 2
days. On exam he has a heart murmur.
A 20-year-old man is involved in a motor vehicle Echocardiography shows destruction of the aortic
accident which results in multiple blunt trauma and valve by large, irregular vegetations. Staphylococcus
lacerations to his lower extremities. The left femoral aureus is cultured from his blood. He develops left
artery is lacerated, and he incurs extensive blood loss upper quadrant pain. Abdominal CT shows a wedge-
and remains hypotensive for hours during transport to shaped 1.5 x 3 cm splenic lesion with base on the
the emergency department. On admission, his capsule. The splenic lesion is most likely to result from
hematocrit is 12%. Which of the following tissues is which of the following cellular abnormalities?
most likely to withstand the impact of these events with A. Coagulative necrosis
the least damage? CORRECT. The description is that of a
A. Skeletal muscle typical infarct with vascular occlusion
CORRECT. The skeletal muscle tissue is following embolization from vegetations
the least metabolically active of the ones of infective endocarditis.
listed; and is also able to function with B. Abscess formation
anaerobic glycolysis. Incorrect. An abscess is a localized
B. Intestinal epithelium collection of pus with a soft yellow
Incorrect. Shock can often lead to purulent center.
ischemic enteritis. C. Metaplasia
C. Retina Incorrect. Metaplasia, the replacement of
Incorrect. The retinal neurons are very one cell type with another, typically
sensitive to hypoxia. occurs on epithelial surfaces.
D. Cerebral cortex D. Caseous necrosis
Incorrect. The large neurons of the Incorrect. Caseous necrosis is typically
cerebral cortex are very active cheesy white and localized to the center
metabolically. of rounded granulomas.
E. Renal tubules E. Liquefactive necrosis
Incorrect. The cells of renal tubules are Incorrect. Liquefaction leads to soft areas
very sensitive to hypoxia, and the of tissue necrosis that resolve leaving a
proximal tubules in particular. space.

A 40-year-old woman has the sudden onset of severe A 35-year-old woman has developed increasing icterus
abdominal pain. On physical examination she has over the last week. On exam her liver span is increased.
diffuse tenderness in all abdominal quadrants, with Laboratory studies show hyperammonemia.
marked guarding and muscular rigidity. She has Abdominal CT scan shows a liver twice normal size
laboratory findings that include serum AST of 43 U/L, with markedly reduced attenuation. These changes in
ALT of 30 U/L, LDH 630 U/L, and lipase 415 U/L. An her liver most likely resulted from which of the following
abdominal CT scan reveals peritoneal fluid collections conditions?
and decreased attenuation along with enlargement of A. Galactosemia
the pancreas. Which of the following cellular changes Incorrect. Although fatty change in liver
is most likely to accompany these findings? can occur with galactosemia, fibrosis is a
A. Coagulative necrosis prominent component, and such findings
Incorrect. Coagulative necrosis is a are typical in infants.
characteristic feature of ischemic injury to B. Hemochromatosis
many organs such as heart, spleen, and Incorrect. Hemochromatosis with
liver. extensive iron deposition would give the
Cellular Injury | Pathology Reviewer (Utah) | 21 / 72
liver a dark brown to orange colour. The pathology laboratory is most likely to demonstrate
attenuation would not be decreased with which of the following pathologic abnormalities?
CT imaging. A. Neoplasia
C. Tuberculosis Incorrect. A neoplasm is a mass lesion.
Incorrect. Tuberculosis leads to B. Gangrene
granuloma formation. Granulomas tend CORRECT. Gangrenous necrosis is a
to be focal areas of inflammation. typical complication of diabetes mellitus
D. Alcoholism with marked peripheral vascular disease
CORRECT. The marked fatty change from atherosclerosis. Gangrene is a form
(steatosis) of the liver with hepatomegaly of coagulative necrosis that involves a
is a typical sequel of chronic alcohol body part, including several tissues. The
abuse. With x-rays, fat density infection adds an element of liquefactive
approaches that of air density (dark). necrosis, best described as 'wet
Grossly, the liver would appear yellowish. gangrene.'
Microscopically, the hepatocyte C. Vasculitis
cytoplasm would contain a large lipid Incorrect. Such a disorder, with either
droplet. thrombosis or hemorrhage, would be
E. Hypoxemia more likely manifested throughout the
Incorrect. The liver is not a major target body. Coagulopathy is not a feature of
of hypoxic injury because of its dual blood diabetes mellitus.
supply. Hypoxia would lead to focal D. Hemosiderosis
necrosis with loss of parenchyma, not Incorrect. Hemosiderin may form locally
enlargement. from remote hemorrhage. With iron
overload, it collects in tissues of the
A 71-year-old man has difficulty with urination. His mononuclear phagocyte system.
urinary retention leads to numerous trips to the E. Caseation
restroom per day. On digital rectal examination is Incorrect. Caseation is a part of
prostate is diffusely enlarged. Which of the following granulomatous inflammation. Caseating
represents a pathologic change leading to this man's granulomas are soft, cheesy, and white.
problem?
A. Dysplasia The lifestyle patterns of healthy persons from 20 to 30
Incorrect. Dysplasia is disordered growth years of age are studied. A subset of these persons
of an epithelium and is a step toward have a lifestyle characterized by consumption of a lot
neoplasia. Dysplasia typically does not of pizza and very little physical exercise. Which of the
increase the size of a tissue. following tissue changes is most likely to develop in this
B. Hypertrophy subset of persons as a consequence of this lifestyle?
Incorrect. Hypertrophy is what many A. Fatty metamorphosis of liver
people call benign prostatic enlargement, Incorrect. Fatty change in the liver is due
but it is the number of cells that increase to toxic and metabolic derangements,
the most, not cell size, contributing to such as those that occur with malnutrition
enlargement. or alcoholism.
C. Hyperplasia B. Pancreatic fat necrosis
CORRECT. Prostatic hyperplasia is an Incorrect. Pancreatic fat necrosis may
example of pathologic hyperplasia in occur from injury from inflammation or
which prostatic cells become more trauma.
sensitive to hormonal stimuli for growth. C. Fatty degeneration of myocardium
Both glands and stroma can increase in Incorrect. Fatty change in the heart is a
amount. Hyperplasia is an increase in the consequence of toxic or hypoxic events.
number of cells. D. Hypertrophy of steatocytes
D. Metaplasia CORRECT. The fat cells (adipocytes)
Incorrect. Metaplasia is trading one kind increase in size (hypertrophy) with
of epithelial cell for another, but in obesity in adults, and this is the
prostatic hyperplasia, the glands are still predominant effect of weight gain.
lined by columnar epithelium E. Metaplasia of muscle to adipose tissue
Incorrect. Muscle does not typically
E. Neoplasia undergo metaplasia in response to
Incorrect. Neoplasia is loss of cell growth weight gain. Adipocytes in fascial planes
control. Pathologic hyperplasias are still and around the muscle can increase in
responding to stimuli and have not size. The muscle may atrophy in
reached the point of autonomous growth response to the sedentary lifestyle.

A 55-year-old man has a 30-year history of poorly


controlled diabetes mellitus. He has had extensive
black discoloration of skin and soft tissue of his right
foot, with areas of yellowish exudate, for the past 2
months. Staphylococcus aureus is cultured from this
exudate. A below-the-knee amputation is performed.
The amputation specimen received in the surgical
Cellular Injury | Pathology Reviewer (Utah) | 22 / 72
In an experiment, a tissue preparation is subjected to exaggerated healing process, not the
oxidant stress. There are increased numbers of free uncontrolled growth of a neoplasm.
radicals generated within the cells. Generation of which
of the following enzymes within these cells is the most A 31-year-old primigravida has a difficult delivery of a
likely protective mechanism to reduce the number of term infant, with loss of 1500 cc of blood. She has
free radicals? hypotension for 6 hours. Over the next month, her
A. Glutathione peroxidase ACTH level decreases. Within the next 3 months, her
CORRECT. The glutathione peroxidase adrenal glands become decreased in size. This
system is designed to help break down alteration of the adrenals is primarily due to which of
free radicals generated from various the following cellular processes?
forms of cell injury. This system works in A. Metaplasia
the background with small numbers of Incorrect. Metaplasia at this site does not
free radicals. Disease processes occur.
generating greater amounts of free
radicals and cell injury can overwhelm B. Gene mutation
this system. Incorrect. Gene transcription may be
B. Catalase downregulated, but the genes do not
Incorrect. Catalases decompose change over such a short time frame. The
hydrogen peroxide, but they are not rate of mutation in normally dividing
specifically designed to scavenge free somatic cells is very low.
radicals. C. Apoptosis
C. Hydrogen peroxide Incorrect. Diminished ACTH from the
Incorrect. Hydrogen peroxide is pituitary does not typically trigger the loss
generated by some cells, such as of adrenal cortical cells.
neutrophils, in order to destroy D. Autophagocytosis
phagocytized microbes. CORRECT. The loss of ACTH leads to
D. NADPH oxidase cortical atrophy from cellular downsizing,
Incorrect. NADPH oxidase helps to with the consequence of decreased
generate free radicals within glucocorticoid and mineralocorticoid
phagolysosomes in cells such as production.
neutrophils. E. Coagulative necrosis
E. Myeloperoxidase Incorrect. Loss of ACTH does not lead to
Incorrect. Myeloperoxidase within infarction of the adrenal. With three
neutrophil granules helps in destruction arterial blood supplies, the adrenal is
of phagocytized microbes. very hard to infarct.

A 38-year-old man incurs a traumatic blow to his upper A 48-year-old man has a history of chronic alcohol
left arm. He continues to have pain and tenderness abuse. He is still able to perform work at his job. He has
even after 3 months have passed. A plain film had no major illnesses. On physical examination, there
radiograph reveals a 4 cm circumscribed mass in the are no significant findings. Laboratory studies show a
soft tissue adjacent to the humerus. The mass contains serum albumin of 4.1 g/dL, ALT 30 U/L, AST 33 U/L,
peripheral areas of brightness on the x-ray. Over the and total bilirubin 1.1 mg/dL. Which of the following
next year this mass gradually resolves. Which of the microscopic findings in his liver is most likely to be
following terms best describes the changes described? present?
A. Dysplasia A. Cholestasis
Incorrect. Dysplasia refers to disordered Incorrect. Bile stasis may occur in the late
epithelial cell growth and does not apply stages of cirrhosis; or may be a function
to soft tissues. of biliary tract obstruction or liver cell
B. Hyperplasia injury.
Incorrect. There are increased numbers B. Fatty change
of cells in this exaggerated healing CORRECT. The toxic effects of the
process, but that does not account for the alcohol culminate in large lipid droplet
bright calcification. accumulation within hepatocytes. Over
C. Hypertrophy time, more hepatocytes are affected and
Incorrect. Though some cells may the liver is less able to compensate. At
increase in size, the process is one of this point, the man retains sufficient
bone formation. hepatocyte function.
D. Metaplasia C. Hemochromatosis
CORRECT. The brightness on x-ray Incorrect. Increased iron in the liver may
suggests calcification from osseous occur in association with alcoholism; but
metaplasia of connective tissues that is not nearly as common as fatty change.
developed in the healing process. This D. Hypertrophy of smooth endoplasmic
condition is known as myositis ossificans, reticulum
because there is bone formation in the Incorrect. This is the result of chronic
injured muscle. barbiturate ingestion.
E. Neoplasia E. Coagulative necrosis
Incorrect. This process is one of Incorrect. Liver infarction is rare because
metaplastic bone formation as part of an of the dual blood supply of oxygen
Cellular Injury | Pathology Reviewer (Utah) | 23 / 72
through the hepatic artery and portal organelles, via its own lysosomes, but
venous system. the cell does not die.

A 44-year-old woman has had episodes of right upper A 73-year-old man suffers a 'stroke.' On physical
quadrant pain during the past 2 weeks. Her stools have examination he cannot move his right arm. A cerebral
become pale in colour over the past 3 days. Laboratory angiogram demonstrates occlusion of the left middle
studies show a serum total bilirubin of 9.7 mg/dL. A cerebral artery. An echocardiogram reveals a
cholangiogram shows that a gallstone has passed into thrombus within a dilated left atrium. Which of the
the common bile duct, resulting in obstruction of the following is the most likely pathologic alteration from
biliary tract. Which of the following tissue this event that has occurred in his brain?
accumulations is most likely to be visualized on her A. Cerebral softening from liquefactive
skin surfaces? necrosis
A. Hemosiderosis CORRECT. Liquefactive necrosis typifies
Incorrect. Excessive iron can be brain infarction. The brain tissue contains
accumulated through increased abundant lipid. After the initial softening,
absorbtion, increased intake, or tissue macrophages will increase in
prolonged transfusion therapy. number and clear the necrotic debris,
B. Calcification leaving a cystic space. Since neurons
Incorrect. Dystrophic calcification can cannot regenerate, the size of the infarct
occur in areas of tissue damage, as in determines the amount of functional loss.
granulomatous diseases. The liver is not The brain has some capacity for rewiring,
a typical spot for metastatic calcification. but this capacity diminishes with age.
C. Lipofuscin deposition B. Pale infarction with coagulative necrosis
Incorrect. Steatosis occurs with direct Incorrect. Infarction of most organs is
injury to hepatocytes, not biliary tract accompanied by coagulative necrosis,
obstruction. but not the brain.
D. Icterus C. Predominantly the loss of glial cells
CORRECT. She has a 'jaundiced' Incorrect. Neurons are far more sensitive
appearance to her sclerae and skin due to hypoxia than glial cells.
to the increased amount of bilirubin. The D. Recovery of damaged neurons if the
bile pigments impart a yellow colour to vascular supply is re-established
the tissues. She has biliary tract Incorrect. It is unlikely that the vascular
obstruction from cholelithiasis and supply can be reestablished in a matter
choledocholithiasis. of minutes.
E. Steatosis E. Wet gangrene with secondary bacterial
Incorrect. Fatty change is a process that infection
occurs in the liver, and biliary tract Incorrect. Gangrenous necrosis is more
obstruction does not typically cause it. typical of a body part, such as a toe or a
foot.
A 45-year-old man has a traumatic injury to his forearm
and incurs extensive blood loss. On physical A 30-year-old woman is claiming in a civil lawsuit that
examination in the emergency department his blood her husband has abused her for the past year. A
pressure is 70/30 mm Hg. Which of the following workup by her physician reveals a 2 cm left breast
cellular changes is most likely to represent irreversible mass. There is no lymphadenopathy. No skin lesions
cellular injury as a result of this injury? are seen, other than a bruise to her upper arm. A
A. Epithelial dysplasia needle biopsy of the breast mass is performed. On
Incorrect. Although dysplasia can be a microscopic examination, the biopsy shows fat
premalignant condition, it is still necrosis. This biopsy result is most consistent with
reversible. which of the following etiologies?
B. Cytoplasmic fatty metamorphosis A. Physiologic atrophy
Incorrect. Fatty change is potentially a Incorrect. At age 30 she is
reversible condition. premenopausal.
C. Nuclear pyknosis B. Breast trauma
CORRECT. The hypotension leads to CORRECT. Fat necrosis is seen with
diminished tissue perfusion with ischemic trauma to the breast, and her lawyer will
injury. Nuclear chromatin clumping is make good use of that documentation.
reversible, but nuclear pyknosis is not. The pattern of multiple injuries of differing
D. Atrophy ages at different sites suggests abuse.
Incorrect. 'Downsizing' of the cell in C. Lactation
atrophy is reversible. Incorrect. Lactation leads to a physiologic
E. Anaerobic glycolysis hyperplasia of the breast with increase in
Incorrect. A lack of sufficient oxygen may lobules.
lead to anaerobic metabolism, but this D. Radiation injury
can be temporary until the hypoxia is Incorrect. A variety of vascular and
relieved. parenchymal changes can occur with
F. Autophagocytosis radiation injury.
Incorrect. The cell 'downsizes' with E. Hypoxic injury
autophagocytosis of cytoplasmic Incorrect. The breast is not a site for
hypoxic injury.
Cellular Injury | Pathology Reviewer (Utah) | 24 / 72
A 46-year-old man has smoked 2 packs of cigarettes C. Extensive edema
per day for the past 30 years. He has had a chronic Incorrect. Edema is not likely to occur at
cough for the past 3 years, worsening over the last 2 this location. There can be pulmonary
weeks. A suspicious left pulmonary parenchymal congestion and edema from left-sided
lesion is seen on a chest x-ray. He has a bronchoscopy heart failure.
performed. A biopsy of a segmental bronchus shows D. Granulomatous inflammation
squamous metaplasia. Which of the following is the Incorrect. Granulomatous inflammation is
most appropriate interpretation of this finding? more typical for infection with
A. Ischemic tissue damage mycobacteria and fungi, not bacteria.
Incorrect. Ischemia could lead to loss of E. Dystrophic calcification
cells, but the lung is hard to damage from Incorrect. Dystrophic calcification may
ischemia because of its dual blood occur in areas of chronic tissue injury, but
supply. the time course here is short.
B. Irritant effect
CORRECT. The irritant effect, such as An 18-year old G2 P1 woman has an amniocentesis
the various chemicals in cigarette smoke, performed at 17 weeks of gestation. Fibroblasts
leads to replacement of the normal recovered from amniocentesis are grown in culture to
epithelium with another (such as assess the karyotype of fetal cells. These cells are
squamous epithelium replacing subcultured for additional experimental work, but the
respiratory epithelium). This metaplastic culture is lost after 50 doublings of the cells has
process is the first step that could lead to occurred, and the fibroblasts no longer grow. Which of
dysplasia and then to neoplasia. Note the following factors affecting these cells is most likely
that histologists call any flattened demonstrated by this phenomenon?
epithelium a 'squamous' epithelium, but A. Nutrition
pathologists refer to specific cell types, Incorrect. The state of nutrition can play
and consider a true squamous epithelium a role in survival; but does not put an
to be a stratified squamous epithelium. upper limit on doublings.
C. Early stage of cancer B. Mutation
Incorrect. Although there can be a natural Incorrect. Mutations occur over time, but
history of progression from metaplasia to overall are uncommon and have more to
dysplasia to carcinoma, most do with appearance of neoplasms.
metaplasias do not become cancers, and C. Apoptosis
metaplasia by itself is not cancer. Incorrect. Apoptosis is programmed cell
D. Viral infection death that is controlled by a variety of
Incorrect. Chronic viral infection is not factors which should not affect cells
likely to lead to metaplasia. growing in culture.
E. Congenital anomaly D. Aging
Incorrect. Metaplasias are typically not CORRECT. A number of factors play a
congenital; but acquired. Hint: the role in aging, but the lack of immortality
pediatric term 'dysplasia' refers to of individual cells is one feature. The lack
disordered development of a tissue, of telomerase activity in most somatic
including cell types that ordinarily are not cells prevents repeated division. Stem
present. In adults, dysplasia refers to an cells have greater replicative capacity.
abnormal change in an epithelium, E. Oxidation
beyond physiologic alteration, that is Incorrect. Cellular damage from
starting down the road to cancer. oxidation--such as free radical damage--
contributes to aging but does not limit the
A 44-year-old woman has had congestive heart failure number of generations.
for the past 4 years. She develops a fever that persists
for over a week. On physical examination, a heart A 54-year-old man has sudden onset of severe, sharp
murmur is present. Her temperature is 38.4°C. chest pain with diaphoresis and dyspnea. On physical
Laboratory studies show a blood culture positive for examination he has tachycardia with an irregular heart
Streptococcus, viridans group. An echocardiogram rhythm. Electrocardiographic changes suggest the
reveals a 1 cm vegetation on the superior aspect of her possibility of focal myocardial damage involving the left
mitral valve. Which of the following pathologic changes lateral ventricular wall. Which of the following
is most likely present in her mitral valve? laboratory tests on the patient's serum is most useful in
A. Passive congestion this situation?
Incorrect. The heart failure would be A. Total cholesterol
mainly left-sided, with pulmonary Incorrect. An elevated total serum
congestion as a consequence. cholesterol, particularly if it is over 200
B. Necrosis mg/dL, will suggest an increased risk for
CORRECT. Friable mitral valvular a myocardial infarction, but will not help
vegetations from infective endocarditis to diagnose an acute event.
can damage the valve. The inflammation B. Troponin I
is predominantly neutrophilic, and a CORRECT. The troponin I will be
collection of neutrophils release a variety elevated with myocardial ischemic injury
of enzymes that cause tissue damage. for up to 2 weeks. Myoglobin and creatine
kinase can become elevated within 3 to 4
hours following injury, similar to troponin
Cellular Injury | Pathology Reviewer (Utah) | 25 / 72
I, and they are sensitive, but not specific, A 56-year-old woman has smoked 2 packs of
markers for myocardial injury, because cigarettes per day for the past 35 years. She has had
they could also be released from skeletal a chronic cough for the past 8 years, but recently has
muscle. noted increased sputum production. On physical
C. Triglyceride examination she has a few crackles auscultated best
Incorrect. Hypertriglyceridemia suggests over the lung bases. Bronchoscopy with biopsy is
an increased risk for coronary performed. The biopsy reveals bronchial epithelium
atherosclerosis and its complications, but with squamous metaplasia. Which of the following
a triglyceride level will not predict an statements represents the best interpretation of these
acute event, nor change because of that findings?
event. A. Physiologic process of aging
D. Lipase Incorrect. Normal aging does not lead to
Incorrect. Lipase can be increased with metaplastic changes in epithelium.
injury to the pancreas, not cardiac Epithelia may become thinner with aging.
muscle. B. Irreversible, even if she stops smoking
E. Sedimentation rate Incorrect. Epithelial metaplasia is a
Incorrect. The sed rate is a very non- reversible process; but may progress to
specific indicator for inflammation and dysplasia.
infection. It essentially measures 'acute C. Metastases to the lung
phase proteins' such as fibrinogen that Incorrect. A metastasis would be
increase with acute tissue injuries. composed of malignant cells. Metaplasia
is one step on the way to malignancy, but
A 42-year-old previously healthy woman notes that is, by itself, not malignant.
over the past week her eyes have developed a D. Risk for infection
yellowish appearance. She has had mild nausea and CORRECT. The loss of normal
vomiting over the past week. On physical examination functioning respiratory epithelium with
she has scleral icterus. She has no other major loss of mucociliary protection means that
physical examination findings except for mild right the normal innate barrier to infection has
upper quadrant tenderness. Which of the following been weakened.
underlying conditions is most likely to contribute to E. Thromboembolism with infarction
development of her icterus? Incorrect. Pulmonary infarctions are not
A. Hypercholesterolemia accompanied by squamous metaplasia.
Incorrect. An increased serum
cholesterol leads to complications of At the end of a normal menstrual cycle, the
atherosclerosis, with the possibility of endometrium sloughs. Examination of the
infarctions in various organs, but not endometrium microscopically shows cellular
disturbances of bilirubin metabolism. fragmentation. Which of the following is most likely to
B. Thrombocytopenia trigger apoptosis in these endometrial cells?
Incorrect. A low platelet count will lead to A. Acute inflammation
a coagulopathy marked by petechiae and Incorrect. Though inflammatory cells may
purpura, but the bilirubin is not recycled respond to the degenerating
fast enough to produce jaundice. endometrium, this is not the triggering
C. Metastatic carcinoma event. Acute endometritis is a pathologic
Incorrect. Even metastases to the liver process, not a physiologic process.
are unlikely to produce obstructive B. Hypoxia
jaundice, because of the focal nature of Incorrect. Hypoxia is not the triggering
the masses. event, as the uterine arteries and
D. Viral hepatitis branches remain intact and functional.
CORRECT. Hepatitis, most often an C. p53 protein accumulation
infectious viral hepatitis, leads to liver cell Incorrect. p53 protein accumulates with
dysfunction with impaired uptake, DNA damage, as can occur with
conjugation, and excretion of bilirubin. As chemotherapy or radiation.
a result bilirubin is not cleared from the D. Decreased estrogen
blood and there is an increasing serum CORRECT. This is one form of the
bilirubin that produces the yellowish intrinsic pathway for induced apoptosis.
appearance of jaundice (icterus). Falling hormone levels late in the
E. Diabetes mellitus endometrial hormonal cycle induce
Incorrect. Diabetes mellitus has a variety menstruation.
of complications, mainly from E. Anaerobic glycolysis
accelerated atherosclerosis, but liver Incorrect. The type of metabolism of the
failure is not common. Liver has a dual endometrium does not change with the
blood supply. hormonal cycle.

An experiment is conducted to determine if cell


membrane injury is lessened by the effects of vitamin
E ingestion. Which of the following cellular components
is primarily involved in generation of lipid peroxides, the
formation of which is inhibited by the vitamin E?
Cellular Injury | Pathology Reviewer (Utah) | 26 / 72
A. Glutathione inflammatory cells are present. Which of the following
Incorrect. Glutathione is part of the descriptive terms is best applied to these Pap smear
protective antioxidant mechanism. and biopsy findings?
B. Ionized calcium A. Dysplasia
Incorrect. Ionized calcium does not CORRECT. Disordered growth of an
directly participate in lipid damage. epithelium is seen in dysplasia, which
Calcium influx into mitochondria is an can be a precursor to neoplasia. When
irreversible cellular injury. the term 'dysplasia' is used in pediatric
C. Lactate conditions, it refers to disordered
Incorrect. Under conditions of hypoxia, development of a tissue, including cell
cellular respiration becomes anaerobic, types that ordinarily are not present. In
with lactic acid production. adults, dysplasia refers to an abnormal
D. Ferric ion change in an epithelium, beyond
CORRECT. Ferric ion is needed for physiologic alteration, that is starting
reduced oxygen species to injure cells. down the road to cancer.
Hydroxyl radicals initiate lipid B. Metaplasia
peroxidation. Incorrect. Metaplasia refers to exchange
E. Lipase of one type of epithelium to another not
Incorrect. Lipase is a digestive enzyme typically found there. The endocervix,
that is formed in the pancreatic acinar which normally has columnar epithelium,
parenchyma and excreted into the may undergo squamous metaplasia with
pancreatic duct system and conveyed to persistent cervicitis.
the small intestine. C. Anaplasia
Incorrect. Anaplasia refers to such an
A 35-year-old woman has had headaches and irregular appearance of cells in a
abdominal pain worsening for 3 months. There are no neoplasm that it cannot be determined
remarkable physical examination findings. On what kind of cell is present.
radionucleide scanning of the neck, she is found to D. Hyperplasia
have a mass involving one of her parathyroid glands. Incorrect. Hyperplasia refers to an
An abdominal CT scan suggests extensive increased number of cells.
nephrocalcinosis along with urinary tract calculi. Which E. Aplasia
of the following laboratory test findings is most likely to Incorrect. Aplasia refers to absence of
accompany her disease? growth--lack of development, which is
A. CO2 of 30 mmol/L typically a congenital problem.
Incorrect. The CO2 is increased with
respiratory acidosis, but this does not A 38-year-old man has a health screening examination.
lead to precipitation of calcium. He has a routine chest x-ray that shows a 2 cm nodule
B. Phosphorus of 2.2 mg/dL in the right lower lobe. The nodule has focal
CORRECT. Metastatic calcification calcifications. A wedge resection of the nodule is done.
occurs as a result of a high serum On microscopic examination the nodule shows
calcium, which in the case of caseous necrosis and calcification. Which of the
hyperparathyroidism is accompanied by following processes explains the appearance of the
a low serum phosphorus. The high calcium deposition:
calcium often induces a fatal cardiac A. Dystrophic calcification
arrhythmia before metastatic calcification CORRECT. Calcium is deposited in and
can occur. around the granuloma as a reaction to
C. Uric acid of 15.1 mg/dL injury with necrosis. Most pulmonary
Incorrect. Uric acid may contribute to granulomas are the result of infection,
formation of uric acid calculi; but does not typically tuberculosis.
play a role in hyperparathyroidism and B. Apoptosis
hypercalcemia. Incorrect. Apoptosis refers to single cell
D. Sodium of 121 mmol/L necrosis, not a process that occur with
Incorrect. Hyponatremia could result the focal tissue destruction seen in a
from excessive antidiuretic hormone; but granuloma.
is not part of hyperparathyroidism. C. Hypercalcemia
E. Calcium of 4.5 mg/dL Incorrect. High calcium levels, such as
Incorrect. A low serum calcium may can occur with hyperparathyroidism, may
occur when the serum phosphorus rises lead to calcium deposition, even in
with renal failure, not with normal tissues, and this is called
hyperparathyroidism. metastatic calcification.
D. Metastatic calcification
A 21-year-old woman has a routine Pap smear Incorrect. Metastatic calcification can be
performed for a health screening examination. The seen with hyperparathyroidism and a
pathology report indicates that some cells are found high serum calcium. The calcification
cytologically to have larger, more irregular nuclei. A occurs in normal tissues.
follow-up cervical biopsy microscopically demonstrates E. Excessive ingestion of calcium
disordered maturation of the squamous epithelium, Incorrect. Calcium homeostasis is
with hyperchromatic and pleomorphic nuclei extending normally regulated by vitamin D and
nearly the full thickness of the epithelial surface. No parathormone and has little to do with the
Cellular Injury | Pathology Reviewer (Utah) | 27 / 72
amount of ingested calcium, unless a more a marker of obesity than a risk for
very large amount is ingested, which myocardial failure.
happens rarely. Excess calcium is D. Cholesterolosis
typically not absorbed or is excreted. Incorrect. Cholesterolosis refers to lipid
accumulation (including cholesterol
A 3-year-old child has been diagnosed with ornithine esters) in macrophages in an epithelium.
transcarbamylase deficiency and has developed The term is generally applied to the
hepatic failure. The left lobe of an adult donor liver is gallbladder.
used as an orthotopic transplant. A year later, the size E. Xanthomatosis
of each liver in donor and recipient is greater than at Incorrect. Xanthomas are accumulations
the time of transplantation. Which of the following of macrophages filled with lipid that form
cellular alterations is most likely to explain this a mass effect.
phenomenon?
A. Metaplasia A cellular mutation results in transcription with
Incorrect. Metaplasia refers to the translation of a protein that does not fold properly. The
exchange of one epithelial cell type for misfolded protein remains within the cell and is not
another. excreted. Activation of which of the following
B. Dysplasia cytoplasmic enzymes is most likely to occur?
Incorrect. Dysplasia in cellular terms A. Caspase
refers to disordered cell growth in an CORRECT. Misfolded proteins can
epithelium. trigger apoptosis with release of
C. Hyperplasia caspases that lead to nuclear and
CORRECT. After removal of part of the cytoplasmic fragmentation. Huntington
liver, remaining liver can undergo disease is one example of a genetic
hyperplasia to compensate. Hepatocytes disease with neuronal cell loss from
are stable cells that are able to re-enter protein misfolding.
the cell cycle. B. Glutathione peroxidase
D. Anaplasia Incorrect. Glutathione protects cells
Incorrect. Anaplasia refers to cells of a against free radicals.
neoplasm that are so poorly C. NADPH oxidase
differentiated that it is impossible to tell Incorrect. NADPH oxidase mutations
from what cell type they arose. underlie chronic granulomatous disease,
where phagocytic cell function is
E. Neoplasia diminished.
Incorrect. Neoplasia refers to the process D. Ribonuclease
of new, uncontrolled growth that forms a Incorrect. Ribonucleases degrade RNA,
tumor mass growing without physiologic and this is not the triggering event with
control. protein misfolding.
E. Telomerase
A morbidly obese 51-year-old woman dies from Incorrect. Telomerase is present in stem
complications of heart disease. At autopsy, her heart cells and preserves the telomeres at the
weighs 600 gm (normal up to 300 gm) with all the end of DNA strands, providing for
chambers enlarged. Microscopically, there is increased ongoing cell division.
fibrous connective tissue seen in the interstitium
between myocardial fibers. The fibers are increased in
size. Beneath the epicardium can be seen adipocytes A 53-year-old man suffers a cardiac arrest and his wife
interdigitating with the myocardial fibers. Which of the calls emergency services. The paramedics arrive a few
following terms best describes the presence of the minutes later and begin life support measures. A
adipocytes in her myocardium? regular heart rate is established after 40 minutes of
A. Steatosis resuscitative efforts as he is being transported to the
Incorrect. Steatosis refers to the hospital. A thrombolytic agent (tPA) is administered.
intracellular accumulation of lipid, either Which of the following cellular processes is most likely
as small (micro) or large (macro) droplets to occur in his myocardium following administration of
(vesicles). This process occurs in the tPA?
hepatocytes. A. Apoptosis
B. Lipid degeneration Incorrect. Apoptosis is single cell
Incorrect. Fatty, or lipid, degeneration is necrosis that is not the major mechanism
a rare phenomenon in which small lipid of cell death from ischemic injury.
droplets may accumulate within B. Free radical injury
myocardial fibers under conditions of CORRECT. The ischemia during the
severe hypoxia. cardiac arrest followed by reperfusion
C. Fatty infiltration established following successful
CORRECT. The adipocytes are normal resuscitation will generate activated
fat cells. Obese persons just have more oxygen species to produce free radical
of them, and the individual cells are larger, injury. This is the so-called 'reperfusion
making them more obvious and injury'.
prominent on examination. Their
presence in the superficial myocardium is
Cellular Injury | Pathology Reviewer (Utah) | 28 / 72
C. Heterophagocytosis bilirubin production to cause the icterus,
Incorrect. Heterophagocytosis by as the amount of bilirubin generated
macrophages is a way of clearing debris exceeds the capacity of the liver to
from dead cells following infarction. conjugate and excrete it into the bile. This
D. Squamous metaplasia also explains his anemia.
Incorrect. There is no epithelium in the E. Pulmonary squamous cell carcinoma
myocardium to undergo metaplasia, and Incorrect. Neoplasms such as
such metaplasia is not a consequence of carcinomas are unlikely to lead to
ischemic injury. hemolysis, though some (leukemias,
E. Accumulation of cytokeratins lymphomas) are associated with
Incorrect. Such cytokeratin accumulation autoimmune phenomena that include
is a long-term process, as is seen with antibodies directed at red blood cells.
neurofibrillary tangles in Alzheimer's
disease. This is not part of an ischemic A 49-year-old man with a history of alcohol abuse has
injury. increasing abdominal girth. On examination his liver
edge is firm. A liver biopsy shows cirrhosis, and
A 38-year-old woman has severe abdominal pain with individual hepatocytes contain red, globular inclusions
hypotension and shock that has developed over the positive for cytokeratin with immunohistochemical
past 36 hours. On physical examination, her abdominal staining. Which of the following structural elements are
muscles are rigid and her abdomen is extremely tender. these intracellular globules most likely to contain?
An abdominal CT scan reveals fluid density in the A. Actin and myosin
region of the pancreas, which appears to be enlarged. Incorrect. Thin actin filaments and thick
Which of the following laboratory test findings in her myosin filaments are components of
serum is most likely to be present? muscle tissue.
A. Alanine aminotransferase of 1123 U/L B. Cholesterol esters
Incorrect. An increased ALT suggests Incorrect. Cholesterol esterified to long
hepatocyte injury. chain fatty acids can be a transport form
B. Total cholesterol of 324 mg/dL of cholesterol utilized in building cell
Incorrect. Hypercholesterolemia is not membranes. Cholesterol may also collect
associated with pancreatitis; but marked to form deposits that are a component of
hypertriglyceridemia (>1g/dL) can be atheromas.
associated with pancreatitis. C. Fatty acids
C. Creatine kinase of 869 U/L Incorrect. Fatty acids can be taken up by
Incorrect. An elevated CK suggests injury a variety of cells for lipid biosynthetic
to striated muscle. pathways; they may be used in energy
D. Urea nitrogen of 110 mg/dL pathways. They may be taken up by fat
Incorrect. An elevated urea nitrogen cells for lipid storage.
suggests renal failure. D. Fibronectin
E. Lipase of 1134 U/L Incorrect. Fibronectin is a component of
CORRECT. There is evidence for acute the extracellular matrix.
pancreatitis with edema and necrosis. E. Intermediate filaments
The necrosis is typically fat necrosis with CORRECT. Intermediate filaments may
grossly visible focal chalky-white collect in the damaged hepatocytes as
deposits representing areas of the globular hyaline known as Mallory
saponification as a consequence of hyaline. It is characteristic for alcoholic
release of pancreatic enzymes (lipase) liver disease, but not pathognomic for it.
with acute inflammation. F. Microtubules
Incorrect. Microtubules are a structural
A 45-year-old man has noted scleral icterus for the past component of cells.
4 days. He has been feeling tired and 'run down' for
about a month. On physical examination, other than A 31-year-old woman has been previously diagnosed
mild tachycardia, his vital signs are normal. No other with systemic lupus erythematosus. She has had chest
physical examination findings are of noted other than pain for the past 6 months. A chest radiograph shows
scleral icterus. Laboratory studies show a hematocrit of bilateral pleural effusions. Thoracentesis on the right
25%. Which of the following conditions is most likely to yields 400 mL of clear yellow serous fluid, and a cell
account for these findings? count shows only a few cells present. What type of cell
A. Systemic hypertension is most likely to be most numerous in this fluid?
Incorrect. Hypertension leads to vascular A. Basophil
changes in arteries, but not to hemolysis Incorrect. Basophils are few in number in
with icterus. the blood and are not often seen in
B. Excessive iron absorption effusions
Incorrect. With hemolysis, the iron is B. Ciliated columnar cell
recycled and neither lost nor gained. Incorrect. Ciliated columnar cells form
C. Diabetes mellitus, type I the lining of the bronchial tree are would
Incorrect. Diabetes mellitus is not be sloughed off into the sputum, not into
associated with red blood cell destruction. the pleural cavity.
D. Intravascular hemolysis
CORRECT. The increased turnover of
red blood cells leads to increased
Cellular Injury | Pathology Reviewer (Utah) | 29 / 72
C. Eosinophil
Incorrect. Eosinophils may participate in
inflammatory reactions that are most
often allergic or parasitic.
D. Fibroblast
Incorrect. Fibroblasts are part of an
inflammatory process with healing and
collagen deposition, typical of a resolving
inflammatory process leading to pleural
adhesions.
E. Langhans giant cell
Incorrect. Langhans giant cells are
aggregated activated macrophages in a
granulomatous response. If the pleura
were involved, the cell count would be
higher.
F. Lymphocyte
CORRECT. Small numbers of
lymphocytes may be present with this
serous effusion, typical for autoimmune
diseases that include serositis.
G. Neutrophil
Incorrect. Neutrophils are characteristic
for exudates containing many cells and
protein, not transudates with few cells
and protein. Neutrophils are more
characteristic of acute inflammatory
processes, such as bacterial infections.

A 65-year-old woman has the sudden inability to move


her right arm and to speak. MR angiography shows
occlusion of a cerebral artery. She is given tissue
plasminogen activator (tPA). Over the next week she
regains some ability to move her arm and to speak.
Which of the following cellular changes most likely
subsided upon tPA therapy?
A. Cell fragmentation
Incorrect. Fragmentation of the cell is a
sign of irreversible injury. It is most
characteristic of apoptosis.
B. Cell swelling
CORRECT. Cell swelling is reversible.
The tPA provided reperfusion so that
areas of brain that were ischemic but not
infarcted could recover. However, waiting
too long to restore circulation could
increase free radical formation and
reperfusion injury.
C. Cytoplasmic eosinophilia
Incorrect. Cytoplasmic eosinophilia is a
feature of irreversible cell damage
because cell organelles have been
significantly altered.
D. Membrane disruption
Incorrect. Cell membrane or organelle
membrane disruption leads to
irreversible cell injury.
E. Nuclear karyorrhexis
Incorrect. Nuclear karyorrhexis is
fragmentation of the nucleus and
represents irreversible injury.
F. Nuclear pyknosis
Incorrect. Nuclear pyknosis is nuclear
shrinkage to the point of irreversible
injury.
IMMUNOPATHOLOGY SECTION

A 15-year-old healthy girl with no major medical multiple myeloma, and conversely just a
problems notes blotchy areas of erythema that are subset of myeloma patients develop
pruritic over the skin of her arms, legs, and trunk within amyloidosis.
an hour every time she eats seafood, followed by E. Alzheimer’s disease
diarrhea. These problems abate within 3 hours, and Incorrect. There is an amyloid angiopathy
then physical examination reveals no abnormal with Alzheimer disease, but it is found in
findings. Which of the following immunologic cortical arteries the brain, and is not
abnormalities is she most likely to have? limited to light chains.
A. Localized anaphylaxis
CORRECT. Food allergies are typically a A 22-year-old woman has experienced episodes of
form of type I hypersensitivity reaction. myalgias, chest pain, and arthralgias for the past 3
The allergens react with IgE bound to years. On examination a friction rub is audible on chest
mast cells, mainly in skin and auscultation and there is dullness to percussion at
gastrointestinal tract. More severe posterior lung bases. No joint deformity is noted. She
allergic reactions may be systemic and has continued working at her job. She has a mild
life-threatening from airway obstruction normocytic anemia. A chest x-ray shows bilateral
and circulatory collapse. pleural effusions. Which of the following laboratory
B. Cell-mediated hypersensitivity screening tests is most appropriate to begin the workup
Incorrect. Cell-mediated immune activity for her condition?
is a feature of Type IV hypersensitivity. A. CD4 lymphocyte count
C. Complement activation Incorrect. A decreased CD4 lymphocyte
Incorrect. Release of complement C3b is count is more typical of AIDS (an immune
a feature of Type III hypersensitivity. deficiency) than of an immunologically
Lack of C1 inhibitor is a rare cause for mediated disease process such as SLE.
edema, but it is not allergic. B. Blood culture
D. Hypergammaglobulinemia Incorrect. The findings given are non-
Incorrect. Plasma cells are not involved specific and do not point to a septicemia.
in this clinical setting. Increases in C. Antinuclear antibody test
immunoglobulin occur over days to CORRECT. These non-specific findings
weeks. put together suggest possible
E. Immune complex deposition autoimmune disease. An ANA is a good
Incorrect. Localized immune complexes way to begin the workup, then more
occur in an Arthus reaction with Type III specific tests can be ordered. Antigen-
hypersensitivity. Systemic immune antibody complexes of many
complex deposition can occur in autoimmune diseases can precipitate in
diseases such as SLE. a variety of locations, typically trapped in
basement membranes in sites such as
A 55-year-old man has developed progressive renal skin, synovium, pleura, pericardium, and
failure for the past 5 years. Microscopic examination of glomeruli. The complexes elicit an
a renal biopsy shows extensive glomerular and inflammatory reaction with
vascular deposition of pink amorphous material on exudates/transudates.
H&E staining. This material demonstrates apple-green D. Sedimentation rate
birefringence under polarized light after Congo red Incorrect. The sedimentation rate is a
staining. Immunohistochemical staining of these very non-specific test that is elevated in
deposits is positive for lambda light chains. Which of many inflammatory conditions. The
the following conditions is most likely to be present in history here points more to an
this man? immunologic disease.
A. Rheumatoid arthritis E. Creatine phosphokinase
Incorrect. Rheumatoid arthritis is a Incorrect. CPK is more useful for
chronic inflammatory disease that may diagnosis of an acute disease --
lead to deposition of AA amyloid. myocardial infarction. CPK can be
B. Tuberculosis increased with myositis, but with the
Incorrect. Tuberculosis is a chronic multiple manifestations in this woman, it
infectious disease that may lead to is not the best screening test.
deposition of AA amyloid.
C. Systemic lupus erythematosus A male infant is born at term. No congenital anomalies
Incorrect. SLE is a chronic inflammatory are noted on examination. A year later he now has
disease that may lead to deposition of AA failure to thrive and has been getting one bacterial
amyloid. pneumonia after another with both Hemophilus
D. Multiple myeloma influenzae and Streptococcus pneumoniae cultured
CORRECT. The light chains are from his sputum. Which of the following diseases is he
elaborated by plasma cells. Light chains most likely to have?
from a plasma cell abnormality contribute A. DiGeorge syndrome
to the formation of the amyloid. Not all Incorrect. DiGeorge syndrome from 22q-
patients with AL amyloid, however, have is predominantly manifested by loss of T-

Page 30 of 72
Immunopathology | Pathology Reviewer (Utah) | 31 / 72
cell function, leading to viral, fungal, and Incorrect. Wiskott-Aldrich syndrome has
protozoal infections. This is most atopy with eczema, thrombocytopenia,
pronounced with complete DiGeorge and immune defects in both cell
syndrome, with lesser severity in partial mediated and humoral arms. The protein
DiGeorge syndrome. There are often is involved with bridging cell surface
congenital anomalies. receptors to actin filaments.
B. Selective IgA deficiency C. Cytokine receptor common gamma chain
Incorrect. A selective IgA deficiency is Incorrect. The most common form of
seen in about 1 in 600 persons and leads severe combined immunodeficiency
to mild diarrhea and/or occasional (SCID) seen in boys is the X-linked defect
respiratory tract infections; but is not life- in the common gamma chain of the
threatening. cytokine receptor involved in signlaing for
C. Epstein-Barr virus (EBV) infection many of the interleukins.
Incorrect. EBV infection does not lead to D. CD40L
immunodeficiency states. CORRECT. He has hyper-IgM syndrome,
D. Acute leukemia an X-linked disorder. The CD40-CD40L
Incorrect. Acute leukemias can result in interaction is involved with isotype
immune deficiency, but not so selective switching from IgM production, thus
and not typically in infancy. accounting for his lack of IgG and IgA.
E. X-linked agammaglobulinemia This accounts for bacterial infections.
CORRECT. Recurrent bacterial However, some degree of cell mediate
infections suggest a lack of B-cell immune defect is present as well.
immune function with lack of gamma E. Complement C1 inhibitor
globulin production. Once protective Incorrect. Lack of C1inh leads to
maternal antibodies are gone, the hereditary angioedema.
disease is manifested more severely.
A 20-year-old woman has had symmetrical, proximal
A study is performed of outcomes involving patients muscle weakness for 6 months. On exam she has 4/5
who received renal allografts. It is noted that patients motor strength in all extremities. A deltoid biopsy
receiving allografts with matching to the donor by tissue shows a neutrophilic infiltrate with focal necrosis of the
typing for HLA-DR (Class II) antigens have a low rate muscle fibers. Her antinuclear antibody test is negative,
of complications. Which of the following immunologic but she has histidyl-tRNA synthetase (Jo-1) antibody in
abnormalities is most likely to be diminished by tissue her serum. Which of the following additional findings
typing? would be most likely be present in this patient?
A. Amyloidosis A. Heart failure
Incorrect. Amyloidosis is a complication Incorrect. The cardiac muscle is usually
from chronic inflammation over many not involved with polymyositis.
years or from multiple myeloma. B. Malabsorption
B. Cell lysis by CD8 lymphocytes Incorrect. Myositis is not typical for
Incorrect. Cytotoxic CD8 cell activation is scleroderma.
primarily a function of class I antigen C. Peripheral neuropathy
recognition. Incorrect. Polymyositis is not
C. Graft versus host disease accompanied by neuropathy. A
Incorrect. GVHD is not a consequence of neuropathy by itself would not account
renal transplantation, and few if any for inflammation in muscle.
donor lymphocytes are present in the D. Skin rash
allograft. CORRECT. Polymyositis-
D. CD4 lymphocyte activation dermatomyositis is present. The
CORRECT. Class II HLA antigens are violaceous skin rash is often quite subtle
involved in the process of CD4 and limited to the face.
lymphocyte activation. E. Renal failure
E. Serum sickness Incorrect. Renal failure is more typical of
Incorrect. Serum sickness results from systemic lupus erythematosus.
antigen-antibody complexes.
A hypersensitivity response of the immune system is
A 5-year-old boy and his 4-year-old brother have had noted to be stimulated by release of IL-4 and IL-5. This
recurrent pneumonia, meningoencephalitis, sinusitis, is accompanied by eosinophilia. This response is most
otitis, and diarrhea since infancy. Bacterial and viral likely be directed against which of the following?
agents have been implicated, as well as Pneumocystis, A. Amyloid protein
Giardia, and Cryptosporidium. Laboratory studies Incorrect. Amyloid deposition does not
show serum IgG 47 mg/dL, IgA 5 mg/dL, and IgM 671 result in an immune response.
mg/dL. Normal numbers of B and T cells are present. B. Spirochetes
These children are most likely to have a mutation Incorrect. Both cell-mediated and
involving a gene encoding for which of the following? humoral immune mechanisms play a role
A. NADPH oxidase in syphilis, but not specifically a type I
Incorrect. Abnormalities of NADPH hypersensitivity reaction.
oxidase are found with chronic
granulomatous disease.
B. Wiskott-Aldrich syndrome protein
Immunopathology | Pathology Reviewer (Utah) | 32 / 72
C. Neoplasms D. Type IV hypersensitivity
Incorrect. Natural immune mechanisms CORRECT. This is a contact dermatitis,
directed against neoplasia are not typical of exposure to a plant such as
generally effective. poison ivy.
D. Inhaled dusts
Incorrect. Some organic dusts can lead A 30-year-old woman has experienced myalgias for the
to a localized type III reaction. past 3 months. On physical examination she has 5/5
E. Liver flukes motor strength in all extremities. She has dullness to
CORRECT. Parasitic infestations with percussion at lung bases. A chest x-ray shows bilateral
organisms that invade tissues can be pleural effusions. Laboratory studies show a positive
accompanied by a type I hypersensitivity antinuclear antibody test at a titer of 1:1024. Her serum
reaction. urea nitrogen is 30 mg/dL. A renal biopsy is performed
and microscopic examination shows a granular pattern
A 54-year-old woman has noted that during the past of immunofluorescence staining with antibody to
month her fingers become pale and painful upon complement component C1q. This pattern is most
exposure to cold. On exam she has mild dyspnea, but typically produced as a consequence of which of the
no wheezing. Her blood pressure is 170/110 mm Hg. following immunologic mechanisms?
Her antinuclear antibody test is positive with a titer of A. IgE coating mast cells
1:256 and a nucleolar pattern. Her serum urea nitrogen Incorrect. This is typical for type I
is 15 mg/dL with creatinine of 1.1 mg/dL. Which of the hypersensitivity reactions, such as
following autoimmune diseases is she most likely to allergic reactions.
have? B. Antiglomerular basement membrane
A. Discoid lupus erythematosus antibody
Incorrect. DLE is characterized mainly by Incorrect. Anti-GBM is a feature of
skin findings. Goodpasture syndrome, which produces
B. Systemic sclerosis a linear pattern of immunofluorescence.
CORRECT. The nucleolar pattern C. Antigen-antibody complexes
suggests scleroderma (systemic CORRECT. A granular pattern of
sclerosis) more than the other glomerular immunofluorescence staining
autoimmune diseases, and likely diffuse is typical of immune complex deposition
scleroderma. Complications of that occurs with a type III hypersensitivity
scleroderma include hypertension, reaction. This patient's findings suggest
though signs of renal failure may not SLE.
occur unless the complication of a D. Macrophage release of lymphokines
hypertensive emergency supervenes. Incorrect. These findings do not suggest
She also has features of Raynaud cell mediated immunity with a type IV
phenomenon and pulmonary fibrosis. hypersensitivity reaction
C. Polymyositis-dermyatomyositis E. Release of prostaglandins
Incorrect. The major autoantibody Incorrect. Prostaglandin release occurs
associated with PM is Jo-1. Renal with acute inflammation and leads to
disease is not a feature of this complex. increased vasodilation.
D. Sjorgen’s syndrome
Incorrect. SS primarily affects salivary The second pregnancy for a 23-year-old woman
and lacrimal glands. appears uncomplicated until ultrasound performed at
E. Rheumatoid arthritis 19 weeks shows hydrops fetalis. The fetal organ
Incorrect. RA mainly affects small joints development is consistent for 19 weeks, and no
and is associated with an increased congenital anomalies are noted. Her first pregnancy
rheumatoid factor (RF). Renal disease is was uncomplicated and resulted in the birth of a normal
not a major feature of RA. girl at term. The current pregnancy ends with birth of a
baby boy at 32 weeks gestation. On examination the
Twelve hours after going on a hike through dense baby has marked icterus. Laboratory studies show a
foliage, a 40-year-old man notices a slightly raised and hemoglobin of 7.5 g/dL, and neonatal exchange
tender irregular reddish rash on one forearm that was transfusion is performed. Which of the following
not covered by clothing. This rash gradually increases immunologic mechanisms best explains this infant's
in intensity for 2 days and then fades away after two findings?
weeks. Which of the following forms of immunologic A. Anti-receptor antibody
hypersensitivity is most likely demonstrated in this Incorrect. In this form of type II
patient? hypersensitivity, an antibody is directed
A. Type I hypersensitivity against a cellular receptor, such as
Incorrect. Type I hypersensitivity against acetylcholine receptor in
reactions typically occur immediately myasthenia gravis.
upon contact with the antigen. B. Loss of self-tolerance
B. Type II hypersensitivity Incorrect. Loss of self-tolerance is found
Incorrect. A rash is not typical for type II in many autoimmune diseases.
reactions. C. Immune complex formation
C. Type III hypersensitivity Incorrect. Immune complexes are found
Incorrect. These findings are not typical in many autoimmune diseases and can
for antigen-antibody complex disease. produce inflammation, but not anemia.
Immunopathology | Pathology Reviewer (Utah) | 33 / 72
D. Delayed type hypersensitivity show that her rheumatoid factor titer is markedly
Incorrect. Type IV hypersensitivity is a elevated, but her antinuclear antibody test is negative.
feature of inflammation with granuloma A rectal biopsy shows submucosal deposition of pink
formation or transplant rejection. amorphous material that stains positively with Congo
E. Complement mediated cell destruction red. Which of the following precursor proteins most
CORRECT. This is immune hydrops, likely gave rise to these deposits?
probably from Rh incompatibility, A. Serum amyloid-associated protein
resulting in maternal antibody crossing CORRECT. She has rheumatoid arthritis,
the placenta and destroying baby's RBCs which is a chronic inflammatory condition
through complement mediated lysis, a that can be complicated by amyloidosis
form of type II hypersensitivity reaction. of the reactive systemic variety with SAA
protein contributing to amyloid deposition
A previously healthy 12-year-old girl develops a sore in a variety of organs
throat over the past 2 days. On physical examination B. Lambda immunoglobulin light chains
she has marked pharyngeal erythema with overlying Incorrect. Light chains are derived from
yellowish exudate. A throat culture grows group A beta- monoclonal antibodies produced from
hemolytic Streptococcus. Her pharyngitis resolves, but plasma cells. Some of these patients
3 weeks later she develops chest pain along with fever have multiple myeloma.
and malaise. The girl describes aching of her knees, C. Transthyretin
hips, shoulders, and elbows. Multiple subcutaneous Incorrect. Transthyretin protein can be
skin nodules are noted on physical examination, and the precursor for amyloid in some familial
she exhibits slight shaking movements of her head and forms of amyloidosis and in cardiac
neck. Her antistreptolysin-O and anti-DNAse B titers amyloidosis.
are elevated. Her creatine kinase MB fraction is D. Amyloid precursor protein
elevated. Her findings are most likely the result of Incorrect. APP is seen in the neuritic
which of the following immunologic mechanisms? plaques in the cerebral cortex of persons
A. Bypass of low-zone tolerance through with Alzheimer disease.
cross-reactivity E. Beta-2-microglobulin
CORRECT. Acute rheumatic fever Incorrect. Beta-2-microglobulin
follows a group A beta hemolytic contributes to amyloid deposition in
streptococcal infection. There is cross- persons on chronic hemodialysis.
reactivity of the immune response to
streptococcal M proteins, with the heart A 35-year-old man has experienced a malar skin rash,
mainly affected. The incidence of polyarthritis with swelling and warmth of his hands, and
rheumatic fever is now so low that strict sensitization to the cold for the past 6 months. On
criteria must be applied for antibiotic physical examination he has generalized
treatment of pharyngitis. lymphadenopathy and pale conjunctivae. Laboratory
B. Deposition of antigen-antibody findings include a hemoglobin of 9.5 g/dL, total WBC
complexes in multiple tissue count of 2100/microliter, total serum protein 8.8 g/dL,
Incorrect. This mechanism is an example albumin 3.6 g/dL, creatinine 1.1 mg/dL, and creatine
of an autoimmune disease such as SLE, kinase of 468 U/L. His antinuclear antibody test is
which probably results from several positive at 1:256. Which of the following additional
immunologic derangements that result in serologic tests is most likely to determine his
B-cell hyperactivity. underlying disease process?
C. Formation of an antibody to immune A. Anti-centromere
complexes Incorrect. Anti-centromere is a helpful
Incorrect. This mechanism can be seen antibody to determine a diagnosis of
with rheumatoid arthritis, which may limited scleroderma (CREST syndrome),
result from modification of self-antigen but he has more findings than accounted
(collagen) so that it is recognized as for by CREST.
foreign by T-cells. B. Anti-RNP
D. Exposure of a previously sequestered CORRECT. The anti-RNP suggests a
antigen diagnosis of mixed connective tissue
Incorrect. This mechanism can be seen disease (MCTD), which has elements of
with ocular trauma to produce SLE, rheumatoid arthritis, and
sympathetic ophthalmia in the non- scleroderma. Unlike SLE, MCTD is less
injured eye. likely to be accompanied by severe renal
E. Interaction between genetic complications, but some cases evolve to
predisposition and environmental insults SLE. Like scleroderma, MCTD is likely to
Incorrect. This is a presumed mechanism result in severe pulmonary fibrosis.
for type I diabetes mellitus with C. Rheumatoid factor
destruction of islet cells. Incorrect. Though he has arthritis, the
other findings are less typical for just
A 60-year-old woman has developed crippling arthritis rheumatoid arthritis.
over the past 20 years. On physical examination the D. Serum C1q complement level
arthritis primarily involves her hands and feet, with Incorrect. The C1q is often decreased
marked joint deformities characterized by ulnar with SLE, but it is not the most specific
deviation and swan-neck deformities of her fingers. test.
She has an irregular heart rate. Laboratory studies
Immunopathology | Pathology Reviewer (Utah) | 34 / 72
E. HIV test A 28-year-old woman is given intravenous penicillin to
Incorrect. The findings suggest an treat infective endocarditis. Within minutes of starting
autoimmune disease more than an this therapy, she begins to have severe difficulty
immunodeficiency disease. breathing with respiratory stridor and tachypnea. She
suddenly develops an erythematous skin rash over
A 48-year-old man has had a chronic cough with fever most of her body. Her symptoms are most likely to be
for 2 months. On physical examination his temperature produced by release of which of the following chemical
is 37.9°C. A chest radiograph reveals a diffuse bilateral mediators?
reticulonodular pattern. A transbronchial biopsy is A. Interleukin 1
performed and microscopic examination shows focal Incorrect. IL-1 is more involved with the
areas of inflammation containing epithelioid appearance of fever with inflammation.
macrophages, Langhans giant cells, and lymphocytes. B. Bradykinin
These findings are most typical for which of the Incorrect. Bradykinin mediates the
following immunologic responses? appearance of pain and of vascular
A. Type I hypersensitivity permeability.
Incorrect. Type I reactions are associated C. Complement C5a
with allergy and anaphylaxis. Incorrect. Complement C5a is a
B. Type II hypersensitivity chemotactic factor.
Incorrect. Type II reactions are D. Histamine
associated with complement-mediated CORRECT. She is having a type 1
immune reactions. hypersensitivity reaction with systemic
C. Graft versus host disease anaphylaxis, in part mediated by
Incorrect. GVHD does not produce a histamine released from mast cells,
granulomatous reaction. which causes vascular permeability.
D. Polyclonal B-cell activation About 20% of all persons will have some
Incorrect. Granulomatous reactions are reaction to penicillin or similar drugs, and
based mostly upon cell-mediated a few of these will be severe.
immunity. E. Thromboxane
E. Type IV hypersensitivity Incorrect. Thromboxane generated by
CORRECT. Granuloma formation with the cyclooxygenase pathway of
reactivation or reinfection tuberculosis in arachidonic acid metabolism mediates
an adult is a classic type IV platelet aggregation and vasoconstriction
hypersensitivity reaction. to promote hemostasis.

A 40-year-old woman has had increasing difficulty A 42-year-old man has lost 7 kg over the past two years.
swallowing for over the past year. She also notes that During that time, he has had pain and swelling around
it is more difficult to use the keyboard of her computer the small joints of his hands and feet. He has had a
because she finds it hard to move her fingers, though series of upper respiratory tract infections, with
the joints are not painful. She has lost the facial organisms including Staphylococcus aureus,
wrinkles that she was beginning to develop in middle Streptococcus pneumoniae, and Pseudomonas
age. A skin biopsy is performed and shows extensive aeruginosa cultured. He has a chronic diarrhea. A stool
dermal fibrosis but almost no inflammatory cell specimen is found to have Giardia lamblia cysts.
infiltrates. These findings are most typical for which of Laboratory studies show a hemoglobin of 8.8 g/dL with
the following conditions? a reticulocyte count of 3.2%. His serum urea nitrogen
A. Mixed connective tissue disease is 17 mg/dL with creatinine 1.0 mg/dL. His antinuclear
Incorrect. MCTD is not characterized by antibody test is negative. Quantitative serum
significant skin involvement. immunoglobulins show IgA 70 mg/dL, IgG 303 mg/dL,
B. Discoid lupus erythematosus and IgM 64 mg/dL. By flow cytometry there are normal
Incorrect. DLE is characterized by numbers of T cells and slightly reduced numbers of B
immune complex deposition with a rash, cells. Which of the following conditions is most likely to
not fibrosis. be present in this man?
C. Limited systemic sclerosis A. HIV infection
CORRECT. The limited form of Incorrect. The bacterial infections
scleroderma, sometimes called CREST suggest hypogammaglobulinemia, not a
syndrome, is a more benign form of primary or secondary T cell defect as
systemic sclerosis (scleroderma) in could occur with human
which fibrosis and tightening of the skin immunodeficiency virus infection.
(sclerodactyly) is common. She has B. Polymyositis
esophageal dysmotility. However, she Incorrect. Anti-Jo1 is more specific for
has not developed respiratory or renal polymyositis.
complications. C. Bruton agammaglobulinemia
D. Dermatomyositis Incorrect. Bruton's disease manifests
Incorrect. Dermatomyositis is early in childhood with a marked
characterized by inflammation and a rash, reduction in all serum globulins.
not fibrosis. D. Common variable immunodeficiency
E. Secondary amyloidosis CORRECT. He has common variable
Incorrect. Amyloid is pink, amorphous immunodeficiency (CVID), with
material, not collagen. hypogammaglobulinemia that
predisposes him to bacterial infections
Immunopathology | Pathology Reviewer (Utah) | 35 / 72
and also Giardia infection. Patients with B. Keratinocyte apoptosis
CVID can have rheumatoid arthritis and CORRECT. The rash is typical for acute
hemolytic anemia. Most patients have graft versus host disease (GVHD). The
normal to slightly reduced numbers of B donor lymphocytes engraft and attack
lymphocytes that cannot differentiate into host tissues, including epidermal
immunoglobulin-secreting plasma cells. keratinocytes, causing apoptosis. The
E. Diabetes mellitus slight antigen mismatch predisposes to
Incorrect. Though persons with diabetes GVHD, though most cases are acute and
mellitus are prone to bacterial infections, mild and can be treated. The two other
they do not get a Giardia diarrhea or tissues often affected by GVHD include
hemolytic anemia or arthritis. liver and GI tract.
C. Dry gangrenous necrosis
A 31-year-old woman has noted chest pain for the past Incorrect. Gangrenous necrosis is very
week. On physical examination she has decreased extensive and involves an entire body
breath sounds with dullness to percussion at lung part, including soft tissues.
bases. A chest radiograph shows modest bilateral D. Contact dermatitis
pleural effusions. On chest CT scan, the pleural Incorrect. Contact dermatitis is a form of
effusions, as well as a pericardial effusion, are seen. A type IV hypersensitivity and produces
thoracentesis on the left yields clear fluid with a low erythema and swelling of skin in affected
protein and cell count. She is found to have an anti- areas exposed to the antigen.
double stranded DNA titer of 1:512. If she is later found E. Abrasions from a fight in the ICU
to have a serum urea nitrogen that is 55 mg/dL, it will Incorrect. Abrasions produce a scraping
most likely be the result of which of the following type of injury.
pathologic processes?
A. Atherosclerosis A 22-year-old woman is bothered by an erythematous
Incorrect. Atherosclerosis, if anything, is rash involving her cheeks and nose every time she
probably less severe in a patient with spends more than a few minutes outdoors on a sunny
systemic lupus erythematosus. day. On physical examination between these episodes,
B. Immune complex deposition there are no abnormal findings. A biopsy of the skin
CORRECT. Systemic lupus involved with the rash shows deposition of IgG along
erythematosus manifestations are often the basement membrane by immunofluorescence
mediated through a type III microscopy, but uninvolved skin does not demonstrate
hypersensitivity reaction with antigen- this IgG deposition. Her antinuclear antibody test is
antibody complexes. These complexes negative. She has no other complaints. Her renal
can lodge in the glomeruli. The immune function tests are normal. Which of the following is the
complexes tend to to be trapped along most likely diagnosis?
basement membranes in epithelial and A. Discoid lupus erythematosus
mesothelial surfaces. CORRECT. This patient has the classic
C. Amyloid deposition malar rash of lupus, but without a positive
Incorrect. Although chronic inflammatory ANA or other findings, discoid lupus
diseases may lead to amyloidosis, this is limited to the skin is the most likely
less common than renal disease due to diagnosis. Some patients with DLE
antigen-antibody complexes with SLE. (particularly those with a positive ANA)
D. Vascular chronic inflammation may go on to develop SLE.
Incorrect. Although chronic inflammation B. Systemic lupus erythematosus
may be present with vasculitis with SLE, Incorrect. SLE would typically have a
immune deposition plays a greater role in positive ANA.
development of renal disease. C. Limited systemic sclerosis
E. Anti-glomerular basement membrane Incorrect. Scleroderma leads to dermal
antibody fibrosis, without vasculitis and without
Incorrect. Goodpasture syndrome, not erythema.
systemic lupus erythematosus, is D. Diffuse systemic sclerosis
associated with anti-GBM that leads to a Incorrect. More extensive systemic
rapidly progressive glomerulonephritis. sclerosis not only has sclerodactyly, but
also renal and lung disease.
A 28-year-old woman undergoes allogeneic bone E. Contact dermatitis
marrow transplantation for acute leukemia. There is a Incorrect. Contact dermatitis does not
5/6 HLA match. Three weeks later, she has marrow usually have a pattern of involvement
engraftment and her hemoglobin and WBC count are over the malar regions and is related to
returning to normal. However, she has the appearance antigen exposure, not sun exposure.
of a fine, scaling skin rash over her trunk and upper F. Atopic dermatitis
extremities. These findings are most consistent with Incorrect. Atopic dermatitis is likely to
which of the following complications? have a component of type I
A. Reduced number of megakaryocytes hypersensitivity.
Incorrect. Thrombocytopenia leads to
appearance of petechiae and purpuric
areas on the skin.
Immunopathology | Pathology Reviewer (Utah) | 36 / 72
A 37-year-old woman develops a malar rash when she but have their onset in late teenage years.
spends several hours outside at a Kensington Oval Giardiasis is common.
cricket match. Ordinarily, she avoids being in the sun, C. Human immunodeficiency virus infection
but a friend provided free tickets to a West Indies test Incorrect. HIV infection could lead to
match. When she sees her physician, a urinalysis is multiple opportunistic infections but
performed that shows hematuria and proteinuria. A would not explain the thymic hypoplasia.
skin biopsy shows immunofluorescence positivity D. Autoantibodies to both T and B
along the dermal-epidermal junction with antibody to lymphocytes
C1q. Which of the following additional complications is Incorrect. One mechanism for
she most likely to suffer as a consequence of her development of common variable
underlying disease? immunodeficiency is autoantibody
A. Pulmonary hemorrhage formation, with targeting of lymphocytes.
Incorrect. The anti-glomerular basement E. Failure of development of 3rd and 4th
membrane antibody of Goodpasture's pharyngeal pouches
produces a linear pattern on Incorrect. Thymic hypoplasia in
immunofluorescence in glomeruli and DiGeorge syndrome leads to T-cell
alveoli, but not skin. deficiency with symptoms related to
B. Joint ankyloses failure of cell-mediated immunity -- viral,
Incorrect. Though patients with SLE may fungal, and protozoal infections.
have arthralgias, there is no significant
erosion of joint surfaces and no bony Every springtime, a young healthy man is bothered by
anklyosis or deformity, such as may be episodes of nasal congestion accompanied by
seen with rheumatoid arthritis. sneezing and watery eyes. He has no cough. On
C. Nodular glomerulosclerosis physical examination he is afebrile. There is swelling of
Incorrect. Nodular glomerulosclerosis, his nasal passageways, but no other findings. His
typically seen with diabetes mellitus, condition improves with use of loratadine. His problems
does not have immune complex are most likely produced by release of chemical
deposition. mediators from which of the following cell types?
D. Pulmonary fibrosis A. Neutrophil
Incorrect. Significant pulmonary fibrosis Incorrect. Neutrophils release proteases
is a feature of diffuse scleroderma, not that can produce local destruction of
SLE. microbes and host cells
E. Pericarditis B. Mast cell
CORRECT. A granular pattern of staining CORRECT. The primary mediator of the
with C1q is characteristic for systemic stuffy nose with hay fever (from allergies
lupus erythematosus with immune to plant pollens) is histamine released
complex deposition with type III from mast cells. Why do you think so
hypersensitivity. Immune complexes many anti-histaminic drugs, such as the
tend to deposit where basement one he received, are sold? Up to 10% of
membranes beneath epithelial and persons have some degree of atopy.
mesothelial surfaces trap them, such as C. CD4+ cell
the epicardial / pericardial surfaces. 50 Incorrect. CD4+ and CD8+ cells are more
over 49 is too many. important in cell-mediated immune
responses.
An 11-month-old infant has had upper and lower D. NK cell
respiratory tract infections almost continuously since Incorrect. NK cells help in destruction of
the time of birth, with organisms including antibody-coated cells and in cells
Pneumocystis jiroveci and Pseudomonas aeruginosa infected with intracellular microbes such
identified. The baby also has oropharyngeal as viruses.
candidiasis. The baby succumbs to a cytomegalovirus E. Macrophage
pneumonitis. At autopsy, the thymus is markedly Incorrect. Macrophages help to clean up
hypoplastic, and lymph nodes throughout the body are an inflammatory mess; but are not part of
small, with absent germinal centers on microscopic an immediate response.
examination. Which of the following mechanisms is
most likely to explain these findings? A 28-year-old woman has noted occasional dyspnea
A. Adenosine deaminase deficiency for the past 13 months. She has also noted a non-
CORRECT. Severe combined productive cough and mild fever. Her symptoms
immunodeficiency (SCID) may be due to improve when she is on vacation. A chest radiograph
adenosine deaminase (ADA) deficiency reveals nodular infiltrates. A transbronchial lung biopsy
in about half of cases. This disorder leads reveals microscopic findings consistent with an
to malfunction of both humoral (B-cell) extrinsic allergic alveolitis with interstitial infiltrates
and cell-mediated (T-cell) immunity. Viral, composed of lymphocytes along with a few neutrophils
fungal, and bacterial infections occur and eosinophils. Which of the following is most likely to
frequently in the first year of life. explain her findings?
B. Failure of B cell maturation to plasma A. Parrots in her house
cells CORRECT. The history suggests
Incorrect. The findings with common exposure to an allergen at home to which
variable immunodeficiency resemble she is not exposed elsewhere. Bird
those of X-linked agammaglobulinemia; fancier's disease is similar to farmer's
Immunopathology | Pathology Reviewer (Utah) | 37 / 72
lung. Both are forms of extrinsic allergic A 40-year-old woman complains of dry mouth and dry
alveolitis, a form of localized antigen- eyes that have become almost constantly irritating
antibody complex disease (Arthus during the past 2 years, regardless of her fluid intake
reaction). Persistence of exposure may or the weather outside. However, her fingers do not
eventually lead to granulomatous turn blue or painful upon exposure to cold. She is found
inflammation as well. serologically to have autoantibodies to the nuclear
B. Heavy cigarette smoking antigen SS-A, along with a positive antinuclear
Incorrect. Smoking does not typically antibody test. Which of the following is most likely to be
produce an Arthus reaction, and smokers the worst long-term complication of her underlying
usually do not quit only when they are disease?
away from home A. Pneumocystis jiroveci pneumonia
C. Downwind from a lead smelter Incorrect. PCP is a complication of
Incorrect. Chemical irritants do not immune deficiency diseases such as
typically produce immune complex AIDS.
disease with extrinsic allergic alveolitis. B. Monoclonal B-lymphocyte proliferation
D. Abusing crack cocaine CORRECT. As in other autoimmune
Incorrect. Cocaine is not associated with diseases, but particularly with her
immune complex disease. Sjogren syndrome, malignancy is more
E. Systemic lupus erythematosus common, and non-Hodgkin lymphoma in
Incorrect. SLE does not come and go particular.
with vacation scheduling and generally C. Malabsorption with diarrhea
does not involve the pulmonary Incorrect. This complication is more
parenchyma in a focal fashion. typical for scleroderma.
D. Diffuse pulmonary interstitial fibrosis
A 30-year-old man has the sudden onset of fever, Incorrect. This complication is more
cough, and dyspnea. He has lost about 10% of his typical for scleroderma.
normal body weight over the past 6 months, along with E. Glomerulonephritis
a chronic, watery diarrhea. A bronchoalveolar lavage is Incorrect. Glomerulonephritis is more a
performed, and cysts of Pneumocystis jiroveci are seen feature of SLE.
in the fluid. He is treated with clotrimoxazole and
recovers. He develops pain and decreased vision on A 3-year-old boy has had numerous bacterial infections,
the right, and funduscopic examination reveals a including respiratory infections with Hemophilus
cytomegalovirus retinitis; this improves with ganciclovir influenzae, Streptococcus pneumoniae, and
therapy. He develops tan-yellow plaques on his tongue Staphylococcus aureus since infancy. He develops a
with Candida albicans. Which of the following polyarthritis that clears with immunoglobulin therapy. A
laboratory test findings is most likely to be present in lymph node biopsy is performed, and microscopically
this man? the germinal centers of his nodes are rudimentary.
A. ANA titer of 1:1024 During the 3rd decade of his life he develops systemic
Incorrect. These opportunistic infections lupus erythematosus. He demonstrates skin test
are more severe than would be expected positivity to Candida antigen. Laboratory studies show
for an autoimmune disease such as SLE. that his serum IgG is 110 mg/dL. His total WBC count
B. Platelet count of 15,000/microliter is 7650/microliter with differential count of 65
Incorrect. Though some persons with segmenters, 4 bands, 22 lymphocytes, and 9
AIDS may develop thrombocytopenia, it monocytes. Which of the following immunologic
is typically not as severe as in persons disorders best explains these findings?
with Wiskott-Aldrich syndrome with A. Adenosine deaminase deficiency
predominantly IgM deficiency. Incorrect. Adenosine deaminase
C. Serum IgA of 10 mg/dL deficiency is a cause for severe
Incorrect. Adults with a selective IgA combined immunodeficiency (SCID).
deficiency are bothered by mild B. Malformation of 3rd and 4th pharyngeal
respiratory tract infections and diarrhea, pouches
not life-threatening opportunistic Incorrect. DiGeorge syndrome has
infections. thymic hypoplasia. The thymus,
D. CD4 lymphocyte count of 100/microliter parathyroids, aorta, and heart can be
CORRECT. The opportunistic infections involved. T-cell function is deficient,
are characteristic for development of resulting in recurrent and multiple fungal,
AIDS in a person with human viral, and protozoal infections.
immunodeficiency virus (HIV) infection. C. Reduction in CD4 lymphocytes
Manifestations of clinical AIDS generally Incorrect. congenital HIV would be
occur once the CD4 count drops below associated with decreased CD4
200/microliter. lymphocytes and problems with cell-
E. cANCA titer of 1:256 mediated immunity without hypocalcemia.
Incorrect. cANCA is a feature of D. Defect in NADPH oxidase
vasculitides such as ANCA-associated Incorrect. Chronic granulomatous
granulomatous vasculitis. disease is manifested by recurrent
bacterial infections, particularly with S.
aureus, but IgG levels are normal.
Immunopathology | Pathology Reviewer (Utah) | 38 / 72
E. Failure of B cell maturation into plasma B. ANCA positive at 1:256
cells Incorrect. A positive ANCA suggests an
CORRECT. He has features of X-linked autoimmune disease with vasculitis as a
agammaglobulinemia of Bruton. In this prominent feature.
condition, B-cell maturation stops after C. Elevated rheumatoid factor
the rearrangement of heavy-chain genes, Incorrect. Her joint deformities occurred
and light chains are not produced. Thus, as a complication of rheumatoid arthritis.
complete immunoglobulin molecules with Rheumatoid nodules typically form in
both heavy and light chains are not subcutaneous locations on extensor
assembled and transported to the cell surfaces, though they may involve
membrane. The lack of immunoglobulins visceral organs such as lung less
predisposes the child to recurrent commonly.
bacterial infections. Since T-cell function D. Markedly decreased serum complement
remains intact, viral, fungal, and Incorrect. Decreased serum complement
protozoal infections are not common is typical for autoimmune diseases such
as SLE.
A bee sting is suffered by a 28-year-old man, and his E. Monoclonal gamma spike
wife searches frantically for the medical kit with the Incorrect. Amyloid deposition typically
injectable epinephrine. Which of the following does not involve bone or joints. Myeloma
immunologic mechanisms are they trying to circumvent? can produce lytic lesions of bone.
A. Immune complex
Incorrect. Local immune complexes are a A 44-year-old man has been plagued by mild chronic
feature of type III hypersensitivity with diarrhea and increased numbers of minor respiratory
farmer's lung. tract infections for most of his life. Following a motor
B. Interleukin release from macrophages vehicle accident with multiple lacerations to the lower
Incorrect. Macrophages have a major leg, he has a significant blood loss requiring transfusion
role with type IV hypersensitivity of blood products. During the transfusion, he is noted
reactions. to have an anaphylactic transfusion reaction. Which of
C. Binding of anti-receptor antibody the following diseases is he most likely to have?
Incorrect. Myasthenia gravis is an A. Selective IgA deficiency
example of an anti-receptor disease of an CORRECT. Such persons with markedly
autoimmune nature (type II decreased to absent IgA can live a
hypersensitivity). Antibody is directed normal life, but they can have antibodies
against acetylcholine receptors. Many to IgA, which set off an anaphylactic
myasthenia gravis patients have a reaction when they encounter IgA in the
thymoma or thymic hyperplasia. serum in transfused blood products.
D. Release of preformed tryptase B. Secondary amyloidosis
CORRECT. Exposure to the allergens of Incorrect. Amyloidosis is a chronic
a bee sting is a cause for systemic disease not marked by hypersensitivity
anaphylaxis for some people, and this reactions.
form of type I hypersensitivity can be life- C. Systemic lupus erythematosus
threatening. The epinephrine counteracts Incorrect. SLE is more likely to have a
the effects of the histamine release. An polyclonal gammopathy; there is no
antihistaminic H1 blocker such as tendency to type 1 hypersensitivity
diphenhydramine can be useful reactions.
treatment as well. Serum tryptase levels D. Common variable immunodeficiency
can serve as a marker for mast cell Incorrect. CVID is related to selective IgA
burden and risk for anaphylaxis. deficiency; but is more severe.
E. Complement activation E. Graft versus host disease
Incorrect. Complement activation is a Incorrect. GVHD is rarely possible from a
feature of type III hypersensitivity transfusion, but the results take weeks to
reactions. manifest.

A 35-year-old man has decreasing mobility of his spine, A 55-year-old woman has had increasing difficulty with
and he becomes more 'hunched over' during the past swallowing over the past year. On physical
5 years. He has hip pain bilaterally. On physical examination her blood pressure is 210/110 mm Hg.
examination he has loss of lumbar lordosis and Laboratory testing reveals a positive antinuclear
diminished range of motion at the hips. A radiograph of antibody test with a nucleolar pattern on
the pelvis shows ankylosis with sacroiliitis bilaterally. immunofluorescence. Her serum complement levels
No other joints appear to be affected. His antinuclear are normal. Which of the following lesions is most likely
antibody test is negative. He has no other significant to be seen in her kidneys with renal biopsy:
medical problems. Which of the following laboratory A. Renal cell carcinoma
test findings is most likely to be present in this man? Incorrect. Scleroderma is not
A. HLA B27 complicated by risk for renal cell
CORRECT. He has ankylosing carcinoma.
spondylitis. However, not every person B. Nodular glomerulosclerosis
with HLA B27 will develop ankylosing Incorrect. Nodular glomerulosclerosis is
spondylitis, one of the most often seen as a complication of long
spondyloarthropathies. term diabetes mellitus
Immunopathology | Pathology Reviewer (Utah) | 39 / 72
C. Acute glomerulonephritis E. Neutrophils emigrating from capillaries
Incorrect. Glomerulonephritis is more Incorrect. Neutrophils release proteases
typical for SLE than for other non-specifically and can be attracted with
autoimmune diseases. complement activation, but their effects
D. Hyperplastic arteriolosclerosis are not localized.
CORRECT. These findings are typical for
diffuse scleroderma, and a hypertensive A 15-year-old boy underwent bone marrow
emergency may result. transplantation for aplastic anemia, with marrow
E. Polycystic change donated from a sibling, with a 5 out of 6 antigen match.
Incorrect. Polycystic change in adults can He later developed a fine, scaling skin rash as well as
be an autosomal dominant inherited a watery diarrhea. Which of the following laboratory
condition; but is not an autoimmune test findings is most likely to accompany this patient's
phenomenon. disease?
A. Hyperamylasemia
A positive tuberculin skin test, with a firm 15 mm Incorrect. The pancreas is not typically
diameter dark red firm area of induration on the forearm, affected by GVHD.
appears 60 hours following injection of the purified B. Hyperbilirubinemia
protein derivative (PPD). This finding is most likely to CORRECT. There is cholestasis in the
be a consequence of which of the following types of liver as a consequence of his graft versus
hypersensitivity reaction? host disease (GVHD), with donor
A. Type I hypersensitivity lymphocytes attacking host tissues that
Incorrect. Allergic conditions and don't have an exact HLA antigen match.
anaphylaxis are typical for type 1 C. Hypocomplementemia
hypersensitivity reactions. Incorrect. Complement tends to be used
B. Type II hypersensitivity up with antigen-antibody reactions, not
Incorrect. Type 2 reactions are diverse GVHD.
and include antibody dependent cell- D. Neutrophilia
mediated cytotoxicity, complement Incorrect. Neutrophilia is seen with
mediated cell lysis, and anti-receptor inflammation and infection, but GVHD
diseases. tends to be mediated more by apoptosis.
C. Type III hypersensitivity E. Elevated creatine kinase
Incorrect. Type 3 reactions are antigen- Incorrect. Muscle tends not to be affected
antibody complex mediated diseases. by GVHD.
D. Type IV hypersensitivity
CORRECT. This tuberculin skin test A 50-year-old man with end stage renal disease from
relies upon an intact cell-mediated long-standing diabetes mellitus receives a cadaveric
immune response. It is positive with prior renal transplant. A month later, he has increasing
exposure to mycobacterial organisms so malaise. His urine output decreases. Laboratory
that memory T cells are present and can studies show a rising serum urea nitrogen and
be activated. creatinine. The allograft is biopsied and seen
E. Complement activation microscopically to be undergoing destruction by cells
Incorrect. Cell-mediated immune which are recognizing graft cells expressing class I
reactions do not depend upon HLA antigens. Which of the following cells is most likely
complement. to mediate this response?
A. CD4 lymphocyte
A study of immunofluorescence patterns in skin Incorrect. CD4 lymphocytes are helper
biopsies is performed. One immunofluorescence cells that are part of rejection, but they
pattern with antibody to IgG that shows a band of tend to recognize class II HLA antigens.
immunofluorescence at the dermal-epidermal junction. B. NK cell
This pattern is most likely to be based upon which of Incorrect. Natural killer cells can
the following pathologic mechanisms? recognize Fc receptors as well as lyse
A. CD4 lymphocytes infiltrating the dermis IgG coated cells as part of an antibody-
Incorrect. CD4 lymphocytes play a role in dependent cell mediated cytotoxicity
cell-mediated immune responses. reaction. NK cells have receptors that
B. Degranulation of mast cells upon recognize self class I MHC molecules.
allergen contact C. CD8 lymphocyte
Incorrect. Degranulation of mast cells CORRECT. These are cytotoxic
occurs with allergic responses, part of lymphocytes that participate in acute
type 1 hypersensitivity reactions. cellular rejection of solid organ
C. Trapping of antigen-antibody complexes transplants. Immunosuppressive agents
CORRECT. This pattern is typical for a are generally good at combating this type
type III hypersensitivity reaction. Antigen- of transplant rejection.
antibody complexes tend to be trapped D. Macrophage
along basement membranes. Incorrect. Macrophages clear up debris
D. Macrophages releasing cytokines after cell death after other inflammatory
Incorrect. Macrophages mainly take part cells have destroyed tissues.
in cell-mediate immune responses and in
some type II hypersensitivity reactions.
Immunopathology | Pathology Reviewer (Utah) | 40 / 72
E. Plasma cell acute inflammatory response dominated
Incorrect. Plasma cells make by neutrophils, which gives the sputum
immunoglobulins and have no lytic or the yellowish, purulent appearance.
phagocytic properties. C. Mast cell
Incorrect. Mast cells degranulate and
Persons infected with HIV are shown to have a period release vasoactive substances as part of
of clinical latency of infection lasting years. During this type 1 hypersensitivity reactions.
time their risk for infection by opportunistic pathogens D. Lymphocyte
is not increased. However, a reservoir of virus remains Incorrect. Lymphocytes and monocytes
within their lymphoid tissues. Which of the following and plasma cells are the 'round cells' of
cells is most important to maintain this reservoir of HIV? chronic inflammatory infiltrates.
A. CD8 lymphocyte E. Langhans giant cell
Incorrect. CD8 cells are not infected by Incorrect. Langhans giant cells are
HIV. CD4 cells are infected by HIV and typically seen with infectious granulomas.
are lysed. During the period of clinical
latency, there is continual turnover of A 31-year-old man incurs an injury in an industrial
CD4 cells. accident, with a steel H-beam striking the right side of
B. High endothelial venule cell his face, crushing the zygomatic arch and damaging
Incorrect. High endothelial venules have the globe of the right eye. He undergoes plastic repair
endothelial cells in lymph nodes that aid of bone and soft tissues and an attempt is made to
in trafficking of cells because of their salvage the eye. A month later, he has decreased
expression of carbohydrate ligands to visual acuity in both eyes, with choroidal thickening
which L-selectins of naive T cells can seen on funduscopic examination. The right eye is
bind. enucleated and shows prominent CD4 lymphocyte and
C. Macrophage macrophage infiltration of the uvea. By which of the
CORRECT. Macrophages can become following immunologic mechanisms has this condition
infected with HIV but can survive and most likely developed?
serve as a reservoir for spread to other A. Breakdown of T-cell anergy
cells. Follicular dendritic cells can also Incorrect. This mechanism of
capture HIV and spread it to other cells. autoimmunity is not part of rheumatic
The gut associated lymphoid tissue fever.
(GALT) is a large reservoir for HIV B. Polyclonal lymphocyte activation
replication. Incorrect. A hypergammaglobulinemia
D. Memory B cell can be seen in autoimmune diseases
Incorrect. Memory B cells play a role in such as SLE, suggesting that this
humoral responses. Unfortunately, mechanism is operative.
vaccines which produce antibody are not C. Release of sequestered antigens
protective of HIV. CORRECT. Sympathetic ophthalmia,
E. NK cell with released of antigens that incite a
Incorrect. NK cells do not play a major granulomatous response, even in the
role in HIV infection. opposite eye, can occur by this
F. Plasma cell mechanism.
Incorrect. Plasma cells secrete D. Cross-reactivity with myocardial antigens
antibodies, and such antibodies may be Incorrect. Streptococcal M proteins cross
directed against components of HIV, react with cardiac glycoproteins, resulting
which is useful for serologic testing for in rheumatic heart disease, a form of
HIV. autoimmunity.
E. Failure of T-cell mediated suppression
A 21-year-old man with serum IgA of 22 mg/dL, IgG of Incorrect. Derangements in T-cell
175 mg/dL, and IgM of 40 mg/dL has had a high fever regulation can lead to immunologic
with cough productive of yellowish sputum for the past disorders and neoplasia, but this is not
two days. Auscultation of his chest reveals a few part of rheumatic fever.
crackles in both lung bases. A chest radiograph shows
bilateral patchy pulmonary infiltrates. Which of the A neonate born at term developed tetany soon after
following inflammatory cell types will most likely be birth. On physical examination the infant has a heart
seen in greatly increased numbers in his sputum murmur. Laboratory studies show a serum calcium of
specimen? 6.3 mg/dL. Echocardiography reveals a membranous
A. Macrophage intraventricular septal defect. Within the next year, this
Incorrect. Macrophages are mainly part infant has bouts of Pneumocystis jiroveci pneumonia,
of the process of resolution of Aspergillus fumigatus pneumonia, and parainfluenza
inflammation. virus and herpes simplex virus upper respiratory
B. Neutrophil infections. Which of the following abnormalities most
CORRECT. The signs and symptoms are likely explains the development of this infant's findings?
suggestive of an acute bacterial A. Abnormal Wiskott-Aldrich syndrome
pneumonia. The decreased serum protein
immunoglobulins at his age suggest Incorrect. Wiskott-Aldrich syndrome has
common variable immunodeficiency, atopy with eczema, thrombocytopenia,
which puts him at risk for bacterial and immune defects in both cell-
infections. Such infections promote an mediated and humoral arms. The protein
Immunopathology | Pathology Reviewer (Utah) | 41 / 72
is involved with bridging cell surface degranulation with release of vasoactive
receptors to actin filaments. substances such as histamine to cause
B. 22q- chromosome anaphylaxis.
CORRECT. This pattern of infection E. Cross-reactivity with tissue antigens
implies a problem with cell-mediated Incorrect. Cross-reactivity is a feature of
immunity, and the hypocalcemia some infections such as those with group
suggests DiGeorge syndrome with A strep producing either a
thymic hypoplasia. The thymus, glomerulonephritis or carditis.
parathyroids, aorta, and heart can be
involved. T-cell function is deficient, A 45-year-old woman has noted episodes of pain with
resulting in recurrent and multiple fungal, swelling and warmth of her hands and feet for the past
viral, and protozoal infections. 3 weeks. During these episodes, lasting hours to days,
C. Reduction in CD4 lymphocytes it is difficult for her to walk or to prepare meals. On
Incorrect. congenital HIV would be physical examination she has painful 1 cm
associated with decreased CD4 subcutaneous nodules on the extensor surfaces of her
lymphocytes and problems with cell- elbows. Over the next 6 years, her hands become
mediated immunity without hypocalcemia. deformed so that it is difficult to perform tasks as simple
D. Defect in NADPH oxidase as opening a door or buttoning her blouse. Which of
Incorrect. Chronic granulomatous the following laboratory test findings is she most likely
disease is manifested by recurrent to have?
bacterial infections, particularly with S. A. HLA B27
aureus, but IgG levels are normal. Incorrect. Not every person with HLA B27
E. Failure of B cell maturation into plasma present will develop ankylosing
cells spondylitis, one of the
Incorrect. With X-linked spondyloarthropathies.
agammaglobulinemia of Bruton, B-cell B. ANA positive at 1:256
maturation stops after the rearrangement Incorrect. A positive ANA suggests an
of heavy-chain genes, and light chains autoimmune disease such as SLE or
are not produced. Thus, complete scleroderma.
immunoglobulin molecules with both C. Elevated rheumatoid factor
heavy and light chains are not assembled CORRECT. Her joint deformities
and transported to the cell membrane. occurred as a complication of rheumatoid
The lack of immunoglobulins arthritis. Rheumatoid nodules typically
predisposes the child to recurrent form in subcutaneous locations on
bacterial infections. Since T-cell function extensor surfaces, though they may
remains intact, viral, fungal, and involve visceral organs such as lung less
protozoal infections are not common. commonly. Another serum marker for her
disease is detection of antibodies to
A 46-year-old woman has experienced progressive, citrullinated peptide/protein (ACPA).
increasing muscular weakness, particularly toward the D. Markedly decreased serum complement
end of the day, over the past 2 months. She does not Incorrect. Decreased serum complement
have arthralgias or myalgias. On physical examination is typical for autoimmune diseases such
her motor strength goes from 5/5 to 4/5 with repetitive as SLE.
movement of extremities. A chest CT scan reveals an E. Hypogammaglobulinemia
anterior mediastinal mass. Laboratory studies show Incorrect. Most patients with rheumatoid
that her antinuclear antibody test is negative. By which arthritis have hypergammaglobulinemia.
of the following immunologic mechanisms is her Some patients with common variable
disease most likely produced? immunodeficiency with
A. Local immune complex formation hypogammaglobulinemia develop
Incorrect. Local immune complexes are a rheumatoid arthritis, but they have a
feature of type III hypersensitivity with problem with infections.
farmer's lung.
B. Interleukin release from macrophages A 7-year-old boy has had recurrent upper and lower
Incorrect. Macrophages have a major respiratory tract bacterial infections since infancy. On
role with type IV hypersensitivity examination he has eczema and areas of ecchymoses
reactions. on his skin. Laboratory studies show Hgb 11 g/dL,
C. Binding of anti-receptor antibody WBC count 7600/microliter, and platelet count
CORRECT. Myasthenia gravis is an 44,000/microliter. His brother and a paternal uncle are
example of an anti-receptor disease of an similarly affected. What is the most likely diagnosis?
autoimmune nature (type II A. Bruton agammaglobulinemia
hypersensitivity). Antibody is directed Incorrect. Though bacterial infections are
against acetylcholine receptors. Many frequent with agammaglobulinemia, and
myasthenia gravis patients have either a it is X-linked, there is no eczema or
thymoma or thymic hyperplasia. thrombocytopenia.
D. Mast cell degranulation B. Complement deficiency
Incorrect. Exposure to the allergens of a Incorrect. A variety of complement
bee sting is a cause for systemic deficiencies may predispose to infections,
anaphylaxis for some people. Antigen there is no thrombocytopemia.
linking to IgE coating mast cells causes
Immunopathology | Pathology Reviewer (Utah) | 42 / 72
C. DiGeorge syndrome autoimmune condition increases the risk
Incorrect. DiGeorge syndrome involves for others, and she likely has rheumatoid
cell-mediated immunity. arthritis.
D. Severe combined immunodeficiency
Incorrect. SCID invoves both humoral Human immunodeficiency virus crosses the mucosa of
and cell-mediated immunity. There is no the vagina into the extracellular matrix. Within the
thrombocytopenia. epithelium the virus encounters a cell which lacks CD4
E. Wiskott-Aldrich syndrome receptors. However, this cell transports the virus to a
CORRECT. WAS results from deficiency regional lymph node. What is this transporting cell most
of a protein involved with cytokeletal likely to be?
functions, including movement of actin A. B lymphocyte
filaments. There is eczema, Incorrect. B-lymphocytes participate in
thrombocytopenia, and recurrent the humoral immune process and can
infection. It has an X-linked inheritance terminally differentiate into antibody-
pattern. secreting plasma cells.
B. Dendritic cell
A study of persons with inflammatory bowel diseases CORRECT. Epithelia contain dendritic
shows that their course of illnesses extends over cells called Langerhans cells which serve
decades. During this time there are bowel lesions to collect antigens and transport them via
marked by ulceration of the mucosa. Inflammatory cells lymphatics and present them to
are present that participate in a TH17 immune response. lymphocytes. The other major type of
Which of the following cell types is mediating the tissue antigen presenting cell is the
injury in these persons? macrophage, but in the case of HIV,
A. B lymphocytes macrophages become infected. Within
Incorrect. B cells are primarily involved in lymph nodes is a population of follicular
humoral responses with immunoglobulin dendritic cells that can serve to capture
production. and present antigen in follicles.
B. Dendritic cells C. CD8 lymphocyte
Incorrect. Dendritic cells are antigen Incorrect. CD8 lymphocytes have a
presenting cells. cytotoxic function, not antigen
C. NK cells presentation.
Incorrect. NK cells respond when MHC D. Neutrophil
antigens are not being presented. Incorrect. Neutrophils are short-lived and
D. Neutrophils ineffective in combating viruses.
Incorrect. Neutrophils are predominantly E. NK cell
an innate immune response in acute Incorrect. NK cells are part of innate
inflammatory processes. immunity and can react to cells not
E. T lymphocytes displaying MHC antigen.
CORRECT. This is a form of T cell
mediated immune response, a form of A 48-year-old man is experiencing marked pain in his
type IV hypersensitivity. left first metatarsophalangeal joint. On examination this
joint is swollen and markedly tender. Joint aspiration
A 55-year-old woman is bothered by dry mouth and eye yields cloudy fluid that on microscopic examination has
discomfort with blurry vision for 2 years. On many neutrophils and needle-shaped crystals. His
examination she has xerophthalmia and xerostomia. acute response is most likely mediated through crystal-
There is bilateral diffuse painless parotid gland induced NOD-like receptor signaling of which of the
enlargement. A year later she develops bilateral following?
swelling, warmth, and pain on movement of her hands. A. Antigen-antibody complex
Which of the following autoantibodies is most likely to Incorrect. Antigen-antibody complexes
be detected in this woman? are characteristic for a type III
A. Centromere hypersensitivity reaction to endogenous
Incorrect. Anti-centromeric antibody is antigens and microbial agents.
more specific for limited systemic B. Complement membrane attack complex
sclerosis. Incorrect. Complement activation can be
B. Double stranded DNA part of an acute inflammatory response,
Incorrect. Double stranded DNA but the membrane attack complex is
antibodies are more specific for SLE. mainly aimed at killing of microbes or
C. Histone lysis of targeted cells.
Incorrect. Anti-histone antibody is more C. Inflammasome
specific for drug-induced SLE. CORRECT. The features of the disease
D. RNP in this man are characteristic for acute
Incorrect. Anti-RNP is more gouty arthritis, and the needle-shaped
characteristic for mixed connective tissue crystals are sodium urate. NOD-like
disease. receptors can recognize many
E. SS-A substances and signal inflammasomes to
CORRECT. The SS-A is characteristic activate caspase which cleaves IL-1 to
for Sjogren syndrome involving salivary an active form promoting inflammation.
glands (major and minor) as well as
lacrimal glands. The presence of one
Immunopathology | Pathology Reviewer (Utah) | 43 / 72
D. Mast cell
Incorrect. Mast cells release biogenic
amines as part of a type I hypersensitivity
response.
E. Toll-like receptor
Incorrect. Toll-like receptors (TLRs) react
to non-self-microbial molecules.

A 54-year-old man has chronic hepatitis C infection


with an elevated serum alanine aminotransferase. His
liver architecture now includes bridging fibrosis with
hepatocyte regeneration. A clone of hepatocytes has
an acquired mutation leading to loss of growth control.
These cells also have poor MHC class I expression.
This abnormal clone of hepatocytes is most likely to be
destroyed via which of the following cells?
A. B lymphocyte
Incorrect. B cells produce antibodies that
are generally ineffective in fighting
intracellular viral infections and in fighting
neoplastic cells.
B. CD4 lymphocyte
Incorrect. CD4 cells are helper cells and
do not directly kill microbes or cells.
C. CD8 lymphocyte
Incorrect. CD8 cells are cytotoxic but
require MHC recognition for targeting.
D. Macrophage
Incorrect. Macrophages may engulf cell
debris but do not directly kill cells.
E. Neutrophil
Incorrect. Neutrophils are part of an
acute inflammatory response that is
ineffective in fighting intracellular viruses
or exerting an anti-neoplastic effect.
F. NK cell
CORRECT. NK cells do not express T
cell receptors, but they are programmed
to respond and destroy cells that fail to
properly display MHC class I antigen. If a
virus downregulates MHC expression, or
a neoplastic cell functions abnormally in
MHC expression, then such abnormal
cells are attached by the NK cells. Thus,
NK cells play a role in clearing viral
infections and in tumor immune
surveillance.
INFLAMMATION SECTION

A 22-year-old man develops marked right lower E. Plasma cell


quadrant abdominal pain over the past day. On Incorrect. Plasma cells are more typical
physical examination there is rebound tenderness on for chronic inflammation.
palpation over the right lower quadrant. Laparoscopic
surgery is performed, and the appendix is swollen, A 39-year-old man incurs a burn injury to his hands and
erythematous, and partly covered by a yellowish arms while working on a propane furnace. Over the
exudate. It is removed, and a microscopic section next 3 weeks, the burned skin heals without the need
shows infiltration with numerous neutrophils. The pain for skin grafting. Which of the following is the most
experienced by this patient is predominantly the result critical factor in determining whether the skin in the
of which of the following two chemical mediators? region of the burn will regenerate?
A. Complement C3b and IgG A. Good cardiac output with tissue perfusion
Incorrect. These two compounds can act Incorrect. Cardiac output may determine
as opsonins. if the patient survives, but not whether the
B. Interleukin-1 and tumor necrosis factor skin regenerates. Tissue perfusion is
Incorrect. Il-1 and TNF act to produce needed to maintain all tissues, not just
fever. the skin.
C. Histamine and serotonin B. Persistence of skin appendages
Incorrect. These are vasodilators. CORRECT. The skin appendages have
D. Prostaglandin and bradykinin epithelium from which the surface
CORRECT. The findings are those of epidermis can regenerate. A partial
acute appendicitis. The acute thickness burn retains epithelial elements
inflammation is marked by neutrophil in the skin. This is why a full thickness
exudation. Release of a variety of burn is much worse and requires grafting.
chemical mediators results in the findings C. Maintenance of underlying connective
associated with inflammation: calor tissue
(warmth), rubor (erythema), tumor Incorrect. Even if collagen is present, the
(swelling), and dolor (pain). The skin may still not regenerate.
analgesic aspirin, by inhibiting the D. Diminished edema and erythema
cyclooxygenase pathway, can decrease Incorrect. These features by themselves
prostaglandin synthesis. give no indication of the prognosis.
E. Leukotriene and HPETE E. Granulation tissue formation
Incorrect. These products of the Incorrect. Regeneration is dependent
lipooxygenase pathway promote upon the presence of remaining
chemotaxis of inflammatory cells such as epithelium.
the neutrophils.
A 58-year-old woman has had a cough with fever for 3
A 40-year-old woman had laparoscopic surgery 3 days. A chest radiograph reveals infiltrates in the right
months ago. Now she has a small 0.5 cm nodule lower lobe. A sputum culture grows Streptococcus
beneath the skin at the incision site that was sutured. pneumoniae. The clearance of these organisms from
Which of the following cell types is most likely to be the lung parenchyma would be most effectively
most characteristic of the inflammatory response in this accomplished through generation of which of the
situation? following substances by the major inflammatory cell
A. Mast cell type responding to this infection?
Incorrect. Mast cells are not numerous in A. Platelet activating factor
tissues and do not proliferate significantly Incorrect. Platelet activating factor leads
as part of a foreign body inflammatory to leukocyte aggregation, adhesion, and
response. chemotaxis, but does not directly
B. Eosinophil facilitate bacterial destruction.
Incorrect. Eosinophils are typical of B. Prostaglandin E2
parasitic and allergic inflammatory Incorrect. Prostaglandins potentiate
responses, not of inflammatory vasodilation, but not neutrophil function.
responses to foreign bodies. C. Kallikrein
C. Giant cell Incorrect. Kallikrein is released from
CORRECT. The suture is a foreign neutrophil lysosomes and promotes
material and produces a foreign body bradykinin formation, leading to
reaction. Persistence of the stimulus vasodilation.
recruits macrophages that are activated D. Leukotriene B4
and transformed to giant cells, in this Incorrect. Leukotrienes promote vascular
case foreign body giant cells. permeability, but not leukocyte function.
D. Neutrophil E. Hydrogen peroxide
Incorrect. A few neutrophils can be CORRECT. Hydrogen peroxide is
present, but giant cells are more specific reduced by myeloperoxidase to a
for a foreign body inflammatory response. powerful oxidant that kills bacteria. She
has an acute inflammatory response to a
bacterial organism, and the majority of

Page 44 of 72
Inflammation | Pathology Reviewer (Utah) | 45 / 72
inflammatory cells responding will be C. Plasma cell synthesis of
neutrophils. immunoglobulins
Incorrect. Plasma cell proliferation is not
A clinical study is performed of patients with a key feature of silicosis or other
pharyngeal infections. The most typical clinical course pneumoconioses.
averages 3 days from the time of onset until the patient D. Mast cell histamine release
sees the physician. Most of these patients experience Incorrect. Mast cells release histamine as
fever and chills. On physical examination, the most an acute inflammatory response, not as
common findings include swelling, erythema, and a chronic process.
pharyngeal purulent exudate. Which of the following E. Macrophage elaboration of cytokines
types of inflammation did these patients most likely CORRECT. Cytokines including growth
have? factors are released from macrophages
A. Granulomatous that have ingested silica crystals, and
Incorrect. Granulomatous inflammation these factors stimulate collagen
typically pursues a course over months to production by fibroblasts, producing
years, and the most important interstitial fibrosis and nodule formation
inflammatory cell is an activated that reduces and restricts the amount of
macrophage. normal lung parenchyma and makes
B. Acute breathing more difficult.
CORRECT. The short course of days
and the purulent exudate are typical A 22-year-old woman has premature labor with
features of acute inflammation with a premature rupture of fetal membranes at 20 weeks
neutrophilic response, most often caused gestation. Prior to that time, the pregnancy had been
by bacterial organisms such as group A proceeding normally. A stillbirth occurs two days later.
Streptococcus. Lacking evidence for an Microscopic examination of the normal-sized placenta
acute bacterial infection, avoid use of reveals numerous neutrophils in the amnion and
antibiotics. chorion, but no villitis. The premature labor was most
C. Gangrenous likely mediated by the effects from release of which of
Incorrect. Gangrene refers to extensive the following substances?
inflammation with necrosis involving a A. Immunoglobulin
whole organ or body part. Incorrect. Increased immunoglobulin
D. Resolving production from plasma cells takes time
Incorrect. With complete resolution, the and is a feature of chronic inflammatory
purulent exudate should be removed, processes.
and the course to this point would B. Prostaglandin
probably be longer. The process may CORRECT. Prostaglandins are
resolve to an abscess, become more mediators of acute inflammation that
chronic, or lead to scarring. promote labor through effects on smooth
E. Chronic muscle and blood vessels.
Incorrect. Chronic inflammation generally C. Complement
pursues a course of weeks. Purulent Incorrect. Complement components are
exudate is not typical for chronic not as important as the prostaglandins in
inflammation, where the predominant cell this setting.
types are mononuclears: lymphocytes, D. Fibrinogen
monocyte-macrophages, plasma cells. Incorrect. Fibrinogen is converted to
fibrin during coagulation.
A 56-year-old man has had increasing dyspnea for 6 E. Lymphokines
years. He has no cough or fever. He had chronic Incorrect. Lymphokines such as TNF and
exposure to inhalation of silica dust for many years in interleukins have a greater effect in
his job. A chest x-ray now shows increased interstitial producing fever.
markings and parenchymal 1 to 3 cm solid nodules. His
pulmonary problems are most likely to be mediated After two weeks in the hospital following a fall in which
through which of the following inflammatory processes? she incurred a fracture of her left femoral trochanter, a
A. Neutrophilic infiltrates producing 76-year-old woman now has a left leg that is swollen,
leukotrienes particularly her lower leg below the knee. She
Incorrect. Leukotrienes are more experiences pain on movement of this leg, and there is
important mediators in acute tenderness to palpation. Which of the following
inflammation, leading to chemotaxis and complications is most likely to occur next after these
increased vascular permeability, among events?
other functions. A. Gangrenous necrosis of the foot
B. Foreign body giant cell formation Incorrect. Obstruction of the arterial
Incorrect. Foreign body giant cells are not supply of blood to the foot would be
a significant feature of silicosis. Even necessary for necrosis to occur, and this
though dust particles are technically patient has evidence for venous
'foreign' material, they do not elicit thrombosis. Necrosis from venous stasis
granuloma formation in the manner of is rare.
infectious agents. However, the cell- B. Hematoma of the thigh
mediated immune response is similar. Incorrect. She has thrombosis, but this
does not typically lead to vascular rupture.
Inflammation | Pathology Reviewer (Utah) | 46 / 72
C. Disseminated intravascular coagulation basement membrane in damaged
Incorrect. DIC is unlikely to occur with tissues.
large venous vessel thrombosis. D. Bradykinin
D. Pulmonary thromboembolism Incorrect. Bradykinin, a product of the
CORRECT. She has deep and kinin system and derived from high
superficial venous thrombosis as a molecular weight kininogen, causes pain
consequence of venous stasis from and promotes vasodilation and vascular
immobilization following the accident. permeability.
The large deep venous thrombi can E. Complement
embolize to the lungs and lead to death. CORRECT. The C5a component of
E. Soft tissue sarcoma complement, along with TNF,
Incorrect. Thrombosis does not leukotrienes, and bacterial products, is
predispose to neoplasia. chemotactic for neutrophils.

A 43-year-old woman has had a chronic cough with An episode of marked chest pain lasting 4 hours brings
fever and weight loss for the past month. A chest a 51-year-old man to the emergency room. He is found
radiograph reveals multiple nodules from 1 to 4 cm in to have an elevated serum creatine kinase. An
size, some of which demonstrate cavitation in the angiogram reveals a complete blockage of the left
upper lobes. A sputum sample reveals the presence of circumflex artery 2 cm from its origin. Which of the
acid fast bacilli. Which of the following cells is the most following substances would you most expect to be
important in the development her lung lesions? elaborated around the region of tissue damage in the
A. Macrophage next 3 days as an initial response to promote healing?
CORRECT. Epithelioid cells and giant A. Histamine
cells are derived from activated Incorrect. Histamine produces
macrophages. Tissue macrophages are vasodilation as part of an immediate
derived from blood monocytes. These inflammatory response. It is not part of
cells are important in the development of the healing process.
granulomatous inflammation in the B. Immunoglobulin G
adaptive immune response to Incorrect. Immunoglobulins are secreted
tuberculosis. by plasma cells as part of a response to
B. Fibroblast inflammation, particularly with infectious
Incorrect. Although collagen is laid down agents.
around a granuloma as part of the C. Complement component C3b
response, it is not the major component Incorrect. C3b is an opsonin.
involved in granuloma formation. D. Leukotriene B4
C. Neutrophil Incorrect. Leukotrienes are best known
Incorrect. Although some neutrophils as acute inflammatory mediators
may be present in a granuloma, they are generated by the cyclo-oxygenase
not the major contributors to granuloma pathway of arachidonic acid metabolism.
formation.
D. Mast cell E. Vascular endothelial growth factor
Incorrect. Mast cells are few in number. CORRECT. The VEGF stimulates
They are involved in release of mediators angiogenesis to aid in establishing
primarily in acute inflammatory collateral circulation and restoring blood
responses. flow to the myocardium in this patient with
E. Platelet an acute myocardial infarction.
Incorrect. Platelets are mainly involved in
coagulation. A 94-year-old woman has developed a fever and cough
over the past 2 days. Staphylococcus aureus is
A 20-year-old man has experienced painful urination cultured from her sputum. She receives a course of
for 4 days. A urethritis is suspected, and Neisseria antibiotic therapy. Two weeks later she no longer has
gonorrheae is cultured. Numerous neutrophils are a productive cough, but she still has a fever. A chest
present in a smear of the exudate from the penile radiograph reveals a 3 cm rounded density in the right
urethra. These neutrophils undergo diapedesis to lower lobe whose liquefied contents form a central air-
reach the organisms. Release of which of the following fluid level. There are no surrounding infiltrates. Which
chemical mediators is most likely to drive neutrophil of the following is the best description for this outcome
exudation? of her pneumonia?
A. Histamine A. Hypertrophic scar
Incorrect. Histamine increases vascular Incorrect. A hypertrophic scar is an
permeability and promotes vasodilation. exaggerated healing response with
B. Prostaglandin extensive collagen deposition that is
Incorrect. Prostaglandin is more most likely to occur with injury to the skin.
important as a mediator of inflammation B. Abscess formation
in producing pain, vascular permeability, CORRECT. The formation of a fluid filled
and vasodilation. cavity following an infection with S.
C. Hageman factor aureus suggests that liquefactive
Incorrect. Hageman factor is Factor XII of necrosis has occurred. The cavity is filled
the coagulation sequence and is with tissue debris and viable and dead
activated by contact with collagen and neutrophils (pus). Localized, pus filled
Inflammation | Pathology Reviewer (Utah) | 47 / 72
cavities are called abscesses. The A small sliver of wood becomes embedded in the finger
liquified contents seek a level (air-fluid of a 25-year-old man. He does not remove it, and over
level). the next 3 days the area around the sliver becomes red,
C. Regeneration swollen, and tender. Neutrophils migrate into the
Incorrect. If lung tissue is destroyed, it injured tissue. Expression of which of the following
does not regenerate. However, there is substances on endothelial cells is most instrumental in
considerable pulmonary reserve capacity, promoting this inflammatory reaction?
so pulmonary failure is not an outcome of A. Interferon gamma
a resolved pneumonia. Incorrect. Interferons are produced by a
D. Bronchogenic carcinoma variety of cells and are mainly part of
Incorrect. Neoplasia is not a inflammatory reactions involving
consequence of acute inflammatory lymphocytes.
processes. B. Hageman factor
E. Chronic inflammation Incorrect. Hageman factor is involved in
Incorrect. Pneumonias caused by coagulation, but not through platelets.
bacterial organisms tend to resolve C. Lysozyme
without becoming chronic. Incorrect. Lysozyme within the
F. Granulomatous cavitation neutrophils aids in bacterial killing.
Incorrect. Granulomas can become large D. E-selectin
and cavitate from central caseous E. CORRECT. Selectins are adhesive
necrosis. However, this process takes molecules expressed on endothelial cell
months. In this case, acute inflammation surfaces and attract leukocytes, mainly
was complicated by abscess formation neutrophils. P-selectin is expressed
with liquefactive necrosis. The most rapidly, while E-selectin is expressed
common organism causing pulmonary within several hours.
granulomas is Mycobacterium F. Prostacyclin
tuberculosis. Incorrect. Prostacyclin is produced by
endothelium to reduce platelet adhesion.
A 36-year-old woman has been taking acetylsalicylic
acid (aspirin) for arthritis for the past 4 years. Her joint An inflammatory process that has continued for 3
pain is temporarily reduced via this therapy. However, months includes the transformation of tissue
she now has occult blood identified in her stool. Which macrophages to epithelioid cells. There are also
of the following substances is most likely inhibited by lymphocytes present. Over time, fibroblasts lay down
aspirin to cause this complication? collagen as the focus of inflammation heals. These
A. Leukotriene B4 events are most likely to occur as an inflammatory
Incorrect. LTB4 is generated through the response to which of the following infectious agents?
lipooxygenase pathway of arachidonic A. Mycobacterium tuberculosis
acid metabolites, which is not affected by CORRECT. Mycobacterial organisms
aspirin. Leukotrienes can affect are difficult to kill, because of their lipid
neutrophil chemotaxis and do not play a coat of mycolic acid. Hence a neutrophilic
major role in production of pain. response has little effect. This persistent
B. Interleukin-1 infection requires macrophage activation
Incorrect. IL-1 is a cytokine that is not part that leads to granuloma formation.
of the arachidonic acid metabolic B. Pseudomonas aeruginosa
pathway. It is not generally associated Incorrect. Bacteria tend to elicit an acute
with pain. inflammatory response.
C. Thromboxane C. Cytomegalovirus
CORRECT. Asprin inhibits thromboxane Incorrect. Viral infections tend to produce
A2 production by platelets, which chronic inflammatory responses.
reduces the ability of platelets to plug D. Giardia lamblia
small vascular defects. Thromboxane is Incorrect. This intestinal parasite
in the cyclooxygenase arm of the produces little inflammation.
arachidonic acid pathway. E. Treponema pallidum
D. Bradykinin Incorrect. Syphilis can produce
Incorrect. Aspirin has no direct effect gummatous necrosis, which is a form of
upon bradykinin generation, even though granulomatous response, but neutrophils
bradykinin has role to play in appearance predominate.
of pain.
E. Hageman factor
Incorrect. Aspirin does not affect
Hageman factor. The anticoagulant
effect of aspirin occurs mainly through its
action on platelets.
Inflammation | Pathology Reviewer (Utah) | 48 / 72
A 37-year-old man has had nausea and vomiting for 5 B. Hemorrhage
weeks. He experienced an episode of hematemesis Incorrect. Complement is present in
yesterday. On physical examination he has no serum and can be activated in
abnormal findings. Upper GI endoscopy is performed, inflammatory processes, but vascular
and there is a 1.5 cm diameter lesion in the gastric leakage with hemorrhage is an
antrum with loss of the epithelial surface. These undesirable effect.
findings are most typical for which of the following
pathologic processes? C. Edema
A. Abscess Incorrect. Vascular dilation with
Incorrect. An abscess is a localized endothelial cell retraction allows
collection of pus. Ulceration may occur intravascular fluid to leak out, causing the
secondarily over an abscess. 'tumor' of swelling in an inflammatory
B. Serositis reaction.
Incorrect. Serositis is inflammation of a D. Hemolysis
mesothelial surface, such as the pleura Incorrect. Hemolysis is a feature of direct
or peritoneum red blood cell damage, not focal
C. Granuloma inflammation.
Incorrect. A granuloma is a localized area E. Vasodilation
of inflammation including epithelioid CORRECT. The inflammatory reaction of
macrophages. sunburn involves vasodilation and
D. Gangrene leakage of fluid. Vasodilation imparts a
Incorrect. Gangrenous necrosis includes red color, which can fade quickly.
coagulative and liquefactive necrosis Prostaglandins are generated through
involving a body part. the cyclooxygenase pathway of
E. Ulcer arachidonic acid metabolism and induce
CORRECT. The loss of an epithelial vasodilation. Histamine contributes to
surface with inflammation is called an vasodilation. Sunburn is typically
ulceration. The presence of hydrochloric superficial, and keratinocytes of the
acid in the stomach promotes this epidermis may undergo apoptosis and
process when epithelial injury occurs. slough off.

A 17-year-old truck driver is involved in a collision. He A 45-year-old woman has had a chronic, non-
incurs blunt force abdominal trauma. In response to productive cough for 3 months, along with intermittent
this injury, cells in tissues of the abdomen are fever. She has a chest radiograph that reveals multiple
stimulated to enter the G1 phase of the cell cycle from small parenchymal nodules along with hilar and
the G0 phase. Which of the following cell types is most cervical lymphadenopathy. A cervical lymph node
likely to remain in G0 following this injury? biopsy is performed. Microscopic examination of the
A. Smooth muscle biopsy shows noncaseating granulomatous
Incorrect. Smooth muscle cells may inflammation. Cultures for bacterial, fungal, and
proliferate in response to injury. mycobacterial organisms are negative. Which of the
B. Endothelium following chemical mediators is most important in the
Incorrect. Endothelium proliferates and development of her inflammatory response?
helps to revascularize injured areas. A. Interferon gamma
C. Skeletal muscle CORRECT. This non-caseating
CORRECT. Skeletal and cardiac muscle granulomatous inflammation is typical for
cells do not typically proliferate in sarcoidosis, a non-infectious process.
response to injury, and necrotic cells are Granulomas are a form of type IV
replaced by connective tissue. hypersensitivity reaction. There are
Remaining striated muscle cells, cytokines such as interferon gamma
however, can undergo hypertrophy. produced by lymphocytes that recruit
D. Fibroblast blood monocytes and stimulate
Incorrect. Fibroblasts form an important macrophages to develop a
part of the response to injury. granulomatous response.
E. Hepatocyte B. Bradykinin
Incorrect. Hepatocytes can proliferate in Incorrect. Bradykinin is mainly a product
response to injury. of acute inflammatory responses and
results in pain.
A 19-year-old woman who works indoors spends a day C. Complement C5a
outside gardening. She does not wear a hat or Incorrect. Complement C5a is a
sunscreen. That evening her partner remarks that her chemotactic factor for neutrophils in
face appears red. Which of the following dermal acute inflammatory reactions.
changes most likely accounts for her red appearance? D. Histamine
A. Neutrophil aggregation Incorrect. Histamine release results in
Incorrect. Leukotriene-induced vasodilation leading to edema with acute
neutrophil aggregation is a function of the inflammation.
lipooxygenase pathway. There will be E. Prostaglandin E2
inflammation, and a collection of Incorrect. Prostaglandins participate in
neutrophils imparts a yellow color as an acute inflammatory reactions.
exudate.
Inflammation | Pathology Reviewer (Utah) | 49 / 72
A 55-year-old man has a history of A 45-year-old man has been working hard all day long
hypercholesterolemia with coronary artery disease and carrying loads of bricks to build a wall. He takes a non-
suffered a myocardial infarction 2 years ago. He now steroidal anti-inflammatory drug (ibuprofen). Which of
presents with crushing substernal chest pain. Which of the following processes is this drug most likely to
the following laboratory tests is most useful in diminish in his arms?
diagnosing the cause of his chest pain? A. Thrombosis
A. Increased white blood cell count Incorrect. However, aspirin is felt to
Incorrect. Although the WBC count may diminish platelet function and may have
increase with inflammation and infarction, a longer term benefit in prevention of
it is non-specific. thrombosis.
B. Elevated sedimentation rate B. Pain
Incorrect. The sedimentation rate is CORRECT. Aspirin, indomethacin, and
elevated in a variety of inflammatory NSAIDS block the cyclo-oxygenase arm
conditions. of the arachidonic acid pathway, which
C. Decreased serum complement generates the prostaglandins that play a
Incorrect. Complement decrease is more role in development of pain as a
typical for inflammatory conditions with consequence of the inflammatory
immunologic mechanisms. process. The inflammation here is
D. Increased serum troponin minimal but widespread as a
CORRECT. Troponins are released from consequence of the overuse of the
the damaged striated muscle. Cell connective tissues in the arms.
damage leads to leakage of intracellular C. Necrosis
contents, such as enzymes or other Incorrect. Necrosis results in cell death.
proteins. Some of these are more Normal activities generally do not
specific for certain tissues to aid in produce significant tissue damage. It
diagnosis. generally takes a disease process, such
E. Decreased platelet count as infection producing inflammation, or
Incorrect. Reduction of the platelet count tissue trauma, or ischemia from loss of
suggests a coagulopathy or hematologic blood supply, to produce necrosis.
disorder. D. Fibrinolysis
F. Increased serum cholesterol Incorrect. The fibrinolytic mechanisms
Incorrect. We already know that his are not affected.
cholesterol has been elevated, and this is E. Scar formation
a measure of his risk for coronary artery Incorrect. Collagen deposition is not
disease; but does not help diagnose an affected.
acute coronary event.
Within minutes following a bee sting, a 37-year-old man
A 15-year-old girl has had episodes of sneezing with develops marked respiratory stridor with dyspnea and
watery eyes and runny nose for the past 2 weeks. On wheezing. He also develops swelling and erythema
physical examination she has red, swollen nasal seen in his arms and legs. An injection of epinephrine
mucosal surfaces. She has had similar episodes each helps to reverse these events and he recovers within
Spring and Summer when the amount of pollen in the minutes. Which of the following chemical mediators is
air is high. Her symptoms are most likely to be most important in the pathogenesis of this man's
mediated by the release of which of the following condition?
chemical mediators? A. Bradykinin
A. Complement C3b Incorrect. Bradykinin can be released
Incorrect. C3b acts as an opsonin. with acute inflammatory reactions, but it
B. Platelet activating factor (PAF) mainly produces pain, and it is not
Incorrect. Although one of the actions of released by mast cells with allergic
PAF is to increase vascular permeability, responses.
PAF is not as important as histamine in B. Complement C5a
this setting. Incorrect. Complement C5a is a
C. Tumor necrosis factor (TNF) chemoattractant for neutrophils, but the
Incorrect. TNF is involved in chemotaxis response in this case is more immediate.
and in producing fever. C. Nitric oxide
D. Histamine Incorrect. Nitric oxide can be released by
CORRECT. Histamine, mainly released macrophages and endothelial cells and is
from mast cells in perivascular locations not a major part of an allergic response.
in the submucosa beneath the nasal D. Tumor necrosis factor
respiratory epithelium, leads to Incorrect. TNF can be released by
vasodilation and fluid exudation. That is activated macrophages and is not a
why people with allergies take significant part of an allergic response.
antihistamines. E. Histamine
E. Immunoglobulin G CORRECT. This is systemic anaphylaxis,
Incorrect. IgG is directed against specific a type I hypersensitivity reaction in which
antigens. It is not as important as IgE in the antigen contacts preformed IgE
an allergic response. antibody attached to mast cells, leading
to mast cell degranulation with release of
bioactive compounds such as histamine
Inflammation | Pathology Reviewer (Utah) | 50 / 72
and production of symptoms. The initial A 43-year-old woman has had nausea with vomiting
response typically occurs within minutes. persisting for the past 5 weeks. On physical
examination there are no abnormal findings. She
A 72-year-old woman did not get a 'flu' shot in the fall undergoes an upper GI endoscopy and gastric
as recommended for older persons. In the wintertime, biopsies are obtained. The microscopic appearance of
she became ill, as many people in her community did, these biopsies shows mucosal infiltration by
with a respiratory illness that lasted for 3 weeks. During lymphocytes, macrophages, and plasma cells. Which
this illness, she had a fever with a non-productive of the following most likely caused her findings?
cough, mild chest pain, myalgias, and headache. What A. Staphylococcus aureus septicemia
was her chest radiograph most likely to have shown Incorrect. Bacterial infections are
during this illness? typically accompanied by acute
A. Hilar mass inflammation, including abscesses in the
Incorrect. Mass lesions are more typical case of septicemia with dissemination of
for neoplasms or granulomatous infection.
diseases. B. Ingestion of chili peppers
B. Interstitial infiltrates Incorrect. Local irritants such as the
CORRECT. Viral illnesses, including capsaicin in the chilis typically produce a
those caused by common viral short-lived acute inflammatory response.
respiratory pathogens such as influenza, C. Diabetes mellitus
often produce chronic inflammation that Incorrect. Diabetes mellitus is associated
is mainly interstitial in the lungs. The lack with an increased risk for acute infections,
of an alveolar filling process results in the but the stomach is not often involved.
lack of a productive cough. D. Tuberculosis
C. Hilar lymphadenopathy Incorrect. Tuberculosis produces a
Incorrect. Lymphadenopathy is more granulomatous inflammatory response
typical for acute inflammatory processes, and rarely involves the stomach.
granulomatous diseases, and metastatic E. Infection with Helicobacter pylori
disease. CORRECT. This is a chronic
D. Lobar consolidation inflammatory process, based upon the
Incorrect. Consolidation is a more typical mononuclear inflammatory cell
feature for acute infectious processes in components present. H. pylori organisms
the lung caused by bacterial organisms. live in the mucus above the gastric
E. Pleural effusions epithelium and release chemical
Incorrect. Pleural effusion is most often a substances that alter epithelial function
feature of right heart failure, neoplasia, or and defenses, leading to gastritis. This
empyema. organism does not directly infect or
damage the epithelium, and its
In an experiment, Enterobacter cloacae organisms are persistence leads to the chronic
added to a solution containing leukocytes and blood inflammation.
plasma. Engulfment and phagocytosis of the microbes
is observed to occur. Next a substance is added which In an experiment, lymphatic channels are observed in
enhances engulfment, and more bacteria are normal soft tissue preparations. Staphylococcus
destroyed. Which of the following substances in the aureus organisms are innoculated into the tissues and
plasma is most likely to produce this effect? the immunologic response observed over the next 24
A. Complement C3b hours. Which of the following functions is most likely to
CORRECT. The C3b fragment be served by these lymphatics to produce a specific
generated from the complement cascade immune response to these organisms?
serves as an effective opsonin, attaching A. Carry lymphocytes to peripheral tissue
to the bacteria so that phagocytes will sites
find and engulf them more easily. Incorrect. There are valves in the
B. Glutathione peroxidase lymphatics (just as in veins) which result
Incorrect. Glutathione peroxidase does in a one-way flow of lymph from
not act as an opsonin; but is involved in peripheral tissues to lymph nodes.
scavenging free radicals. B. Remove extravascular tissue fluid
C. Immunoglobulin M Incorrect. Excess tissue fluid is drained
Incorrect. IgM does not act as an opsonin, by lymphatics, but this does not
though IgG does. specifically serve an immune function,
D. P-selectin though antigens may be contained within
Incorrect. Selectins aid in initial binding of the lymphatic fluid.
leukocytes to endothelial surfaces so that C. Transport antigen presenting cells
the leukocytes slow down and attach in CORRECT. Antigen presenting cells
preparation for their exit into tissues. such as dendritic cells and macrophages
E. NADPH oxidase which have trapped antigen in peripheral
Incorrect. NADPH oxidase in leukocytes tissues can be transported to lymph
aids in the respiratory burst that leads to nodes where they encounter specific
killing of phagocytized microbes. clones of memory cells capable of
responding to those specific antigens,
and this typically happens within a day.
Inflammation | Pathology Reviewer (Utah) | 51 / 72
D. Serve as a route for dissemination of In a clinical study, patients undergoing laparoscopic
infection cholecystectomy are followed to document the post-
Incorrect. Infectious agents can gain surgical wound healing process. The small incisions
access to lymphatics and go to lymph are closed with sutures. Over the 4 weeks following
nodes to produce lymphadenitis. This surgery, the wounds are observed to regain tensile
spread of infection is not advantageous strength and there is re-epithelialization. Of the
to the host. following substances, which is most likely found to
E. Provide an emigration point for function intracellularly in cells involved in this wound
neutrophils healing process?
Incorrect. Most neutrophils marginate, A. Fibronectin
roll, adhere, and diapedese through Incorrect. Fibronectin acts in the
venules and capillaries. extracellular matrix to bind
macromolecules (such as proteoglycans)
In an experiment, surgical wound sites are observed via integrin receptors to aid attachment
following suturing. An ingrowth of new capillaries is and migration of cells.
observed to occur within the first week. A substance B. Laminin
elaborated by macrophages is found at the wound site Incorrect. Laminin is an extracellular
to stimulate this capillary proliferation. Which of the matrix component that is abundant in
following substances is most likely to have this function? basement membranes.
A. Platelet-derived growth factor C. Tyrosine kinase
Incorrect. PDGF is released from CORRECT. Cell surface growth factor
macrophages as well as activated receptors recruit intracellular protein
platelets and can stimulate fibroblast kinases that begin a sequence of events
growth for collagen synthesis. leading to cell division and growth.
B. Phospholipase C-gamma D. Hyaluronic acid
Incorrect. Phospholipases aid in Incorrect. Hyaluronic acid is one of the
stimulation of intracellular protein kinases proteoglycans in the extracellular matrix.
that promote protein phosphorylation. E. Collagen
C. Fibronectin Incorrect. Collagen fibers are part of the
Incorrect. Fibronectin is a component of extracellular matrix that gives strength
the extracellular matrix that helps to link and stability to connective tissues.
cells together.
D. Fibroblast growth factor A 31-year-old woman has a laparotomy performed for
CORRECT. FGF can stimulate all removal of an ovarian cyst. She recovers uneventfully,
aspects of angiogenesis. with no complications. At the time of surgery, a 12 cm
E. Epidermal growth factor long midline abdominal incision was made. The tensile
Incorrect. EGF is a stimulator of cell strength in the surgical scar will increase so her normal
growth through activation of tyrosine activities can be resumed. Most of the tensile strength
kinases in the cells; it can also stimulate will likely be achieved in which of the following time
angiogenesis to a degree. periods?
A. One week
A 55-year-old man with a history of ischemic heart Incorrect. The wound site has granulation
disease has worsening congestive heart failure. He tissue in this first week, but little collagen
has noted increasing dyspnea and orthopnea for the to provide tensile strength at this point.
past 2 months. On physical examination there is B. One month
dullness to percussion at lung bases. A chest x-ray Incorrect. Collagen is being laid down
shows bilateral pleural effusions. A left thoracentesis is during this month, but a plateau phase
performed, and 500 mL of fluid is obtained. Which of has not been reached, and wound
the following characteristics of this fluid would most strength is still increasing.
likely indicate that it is a transudate? C. Three months
A. Cloudy appearance CORRECT. About 70 to 80% of the
Incorrect. A cloudy appearance suggests tensile strength in the scar, compared to
the presence of increased cells, non-wounded skin, is reached at 3
characteristic of an exudate. months, which is about as much as will
B. High protein content be obtained.
Incorrect. Increased protein is D. Six months
characteristic of an exudate. Incorrect. After 3 months, there will be
C. <3 lymphocytes/microliter little additional gain in tensile strength, so
CORRECT. A transudate resembles a little more tensile strength will be
filtrate of plasma and has few cells with achieved in another 3 months.
very little protein and has a clear E. One year
appearance. Incorrect. After 3 months, there will be
D. Presence of fibrin little additional gain in tensile strength, so
Incorrect. Fibrin comes from exuded little more tensile strength will be
fibrinogen, a blood protein. achieved in another 9 months.
E. Large size of the effusion
Incorrect. The size of an effusion does
not define exudate or transudate.
Inflammation | Pathology Reviewer (Utah) | 52 / 72
A 9-year-old girl sustains a small 0.5 cm long laceration D. Prostacyclin
to her right index finger while playing 'Queen of Swords' Incorrect. Prostacyclin is generated by
with a letter opener. Which of the following substances, the arachidonic acid pathway and it
on contact with injured vascular basement membrane, promotes vasodilation.
activates both the coagulation sequence and the kinin E. Complement C3a
system as an initial response to this injury? Incorrect. C3a can promote vasodilation
A. Thromboxane and increase vascular permeability to
Incorrect. Thromboxane, a product of the promote vascular collapse, but
cyclooxygenase pathway, is produced by complement is found in plasma, not cells.
endothelial cells and promotes
vasoconstriction and platelet aggregation. A 20-year-old woman sustains an injury to her right calf
B. Plasmin in a mountain biking accident. On physical examination
Incorrect. Plasmin, derived from she has a 5 cm long laceration on the right lateral
plasminogen, is one of the aspect of her lower leg. This wound is closed with
anticoagulants generated to break down sutures. Wound healing proceeds over the next week.
thrombi. Which of the following factors will be most likely to aid
C. Platelet activating factor and not inhibit wound healing in this patient?
Incorrect. PAF promotes platelet A. Commensal bacteria
aggregation and release. It is also Incorrect. The commensal population of
chemotactic to neutrophils and causes organisms on the skin ordinarily does not
them also to aggregate and adhere. gain access to underlying tissues to
D. Hageman factor establish infection. A wound may provide
CORRECT. Hageman factor (factor XII a route for infection, a serious
as measured in the intrinsic coagulation postoperative complication.
pathway) becomes activated upon B. Decreased tissue perfusion
contact with injured, exposed vascular Incorrect. Diabetics and persons with
basement membrane. severe atherosclerosis have notoriously
E. Histamine poor wound healing.
Incorrect. Histamine functions as a C. Presence of sutures
vasodilator of arterioles and increases CORRECT. Although any foreign body
vascular permeability in venules. will delay wound healing to some degree,
the net effect of having sutures is to aid
A 65-year-old woman has had a fever for the past day. the process of healing more than to
On physical examination her temperature is 39°C and hinder it, since the sutures close the
blood pressure 90/50 mm Hg with heart rate of wound, so there is less space to fill in by
106/minute. Laboratory studies show a WBC count of granulation tissue, and less distance for
12,510/microliter and WBC differential count of 78 segs, cells to move.
8 bands, 11 lymphs, and 3 monos. A blood culture is D. Corticosteroid therapy
positive for Escherichia coli. Her central venous Incorrect. Corticosteroids dampen the
pressure falls markedly. She goes into hypovolemic inflammatory response that contributes
shock as a result of the widespread inappropriate to the healing process.
release of a chemical mediator derived from E. Hypoalbuminemia
macrophages. She develops multiple organ failure. Incorrect. Poor nutrition is a detriment to
Which of the following mediators is most likely to wound healing.
produce these findings?
A. Nitric oxide A 24-year-old primigravida is late in the second
CORRECT. Nitric oxide (NO) acts as a trimester of pregnancy. She experiences the sudden
vasodilator. Small amounts released onset of some cramping lower abdominal pain. This is
from endothelial cells causes immediately followed by passage of some fluid per
vasodilation and prevents thrombosis. vagina along with a foul-smelling discharge. The fetus
Macrophages release NO to kill bacteria, is stillborn two days later. Examination of the placenta
but large amounts released body-wide demonstrates extensive neutrophilic infiltrates in the
can cause hypotension and shock. chorion and amnion. Which of the following organisms
B. Bradykinin is most likely to be responsible for these findings?
Incorrect. Bradykinin produces pain. It is A. Mycobacterium tuberculosis
generated via the kinin system from Incorrect. Tuberculosis produces a
substances in plasma activated during granulomatous pattern of infection that
acute inflammation. develops over weeks to months.
C. Histamine B. Herpes simplex virus
Incorrect. Histamine can contribute to Incorrect. Viral infections typically do not
vasodilation, edema, and vascular cause acute inflammation with
collapse, but much of it is preformed and neutrophils.
released from mast cells or basophils. C. Escherichia coli
Widespread histamine release can be CORRECT. She developed an acute
part of anaphylaxis with type I chorioamnionitis, and bacterial infections
hypersensitivity, but infections typically that precede or follow premature rupture
do not produce this effect. of membranes are most often to blame.
Bacterial infections tend to elicit
neutrophilic inflammatory responses.
Inflammation | Pathology Reviewer (Utah) | 53 / 72
D. Treponema pallidum D. Chronic inflammation
Incorrect. Syphilis is not known for Incorrect. Chronic inflammation has a
producing acute infection with preponderance of mononuclear cells, not
neutrophilic infiltrates. neutrophils.
E. Toxoplasma gondii E. Granulomatous inflammation
Incorrect. This parasitic organism Incorrect. A granulomatous response is
produces a congenital infection over characterized by mononuclear cells.
weeks' time.
A 52-year-old woman with no major medical problems
A 19-year-old man incurs a stab wound to the chest. takes a long airplane flight across the Pacific Ocean.
The wound is treated in the emergency room. Two Upon arrival at Sydney's Kingsford Smith airport
months later there is a firm, 3 x 2 cm nodular mass with following the flight from Los Angeles, she cannot put
intact overlying epithelium in the region of the wound. her shoes back on. There is no pain or tenderness.
On examination the scar is firm, but not tender, with no Which of the following is the most likely explanation for
erythema. This mass is excised and microscopically this phenomenon?
shows fibroblasts with abundant collagen. Which of the A. Activation of Hageman factor has led to
following mechanisms has most likely produced this bradykinin production.
series of events? Incorrect. There is no tissue damage in
A. Keloid formation this situation to activate Hageman factor.
CORRECT. Some persons have an B. A lot of drinks were served in the first-
exaggerated, inappropriate wound class section.
healing response with excessive Incorrect. Normal renal function will
collagenization. maintain water and electrolyte balance,
B. Development of a fibrosarcoma along with trips to the rest room.
Incorrect. Trauma does not lead to C. Femoral vein thrombosis developed.
neoplasia. Incorrect. Though swelling could come
C. Poor wound healing from diabetes from venous obstruction with thrombosis,
mellitus there would likely be pain and tenderness
Incorrect. Diabetes mellitus leads to with thrombosis, and swelling in more
severe peripheral vascular disease that than just her feet. However, the
delays wound healing, but the wound immobilization with venous stasis from
would not have excessive collagen. such a long flight is a risk for thrombosis.
D. Foreign body response from suturing D. A cellulitis developed in her legs.
Incorrect. Sutures can produce small Incorrect. She would be unlikely to
foreign body granulomas that are develop a cellulitis over this short a time.
typically not visible. E. Venous hydrostatic pressure became
E. Staphyloccocal wound infection increased.
Incorrect. A wound infection will produce CORRECT. The longest non-stop flight in
dehiscence and abscess formation that the world has its drawbacks. Extracellular
delays or disrupts collagenization. fluid builds up when hydrostatic pressure
is increased over a long time. Remaining
A 45-year-old man has had a fever and dry cough for 3 upright and stationary in a cramped
days, and now has difficulty breathing and a cough airplane predisposes to reduced venous
productive of sputum. On physical examination his and lymphatic return.
temperature is 38.5°C. Diffuse rales are auscultated
over lower lung fields. A chest radiograph shows a In an experiment, a lung tissue preparation is exposed
right pleural effusion. A right thoracentesis is performed. to Mycobacterium tuberculosis organisms. Over the
The fluid obtained has a cloudy appearance with a cell next week, it is observed that granulomas form in the
count showing 15,500 leukocytes per microliter, 98% lung. Within the granuloma are found inflammatory
of which are neutrophils. Which of the following terms cells expressing class II MHC antigens. These cells
best describes his pleural process? elaborate cytokines that promote fibroblastic
A. Serous inflammation production of collagen within the granulomas. From
Incorrect. A transudate in a serous which of the following peripheral blood leukocytes are
effusion has few cells. these cells bearing class II antigen most likely to be
B. Purulent inflammation derived?
CORRECT. The neutrophils suggest an A. Neutrophils
acute inflammatory process; the fluid is Incorrect. Neutrophils participate in acute
characteristic for an exudate. Such a inflammatory processes and release
large amount of purulent exudate in the enzymes to cause non-specific
pleural space can be termed an destruction of anything nearby.
empyema. B. B cells
C. Fibrinous inflammation Incorrect. B cells are activated to become
Incorrect. Fibrin can often accompany plasma cells that secrete
acute inflammatory processes, but a immunoglobulins.
process with so many neutrophils is best C. Monocytes
characterized as a purulent exudate. CORRECT. Blood monocytes can
migrate into tissues and become
macrophages. Macrophages play an
Inflammation | Pathology Reviewer (Utah) | 54 / 72
important role in delayed hypersensitivity
reactions with cell-mediated immunity.
D. NK cells
Incorrect. Natural killer cells selectively
bind to a target site to destroy it.
E. Basophils
Incorrect. Basophils play a role in allergic
responses.

A 56-year-old man has had increasing difficulty


breathing for the past week. On physical examination
he is afebrile. Auscultation of his chest reveals
diminished breath sounds and dullness to percussion
bilaterally. There is 2+ pitting edema present to the
level of his thighs. A chest radiograph reveals bilateral
pleural effusions. Which of the following laboratory test
findings is he most likely to have?
A. Hypoalbuminemia
CORRECT. The decrease in oncotic
pressure from decreased serum albumin,
the blood protein that accounts for most
of the oncotic pressure, can be significant.
This can be a cause for edema and fluid
transudates. Too little circulating protein
doesn't keep in or draw water into the
vasculature.
B. Glucosuria
Incorrect. Glucosuria with diabetes
mellitus can explain loss of free water
with dehydration, not edema.

C. Neutrophilia
Incorrect. Neutrophilia suggests an acute
inflammatory response, which can
produce localized edema in the area of
inflammation.
D. Anemia
Incorrect. Anemia reduces oxygen
carrying capacity; if severe, it could
eventually lead to a high output
congestive heart failure that would
initially involve mainly the left heart, with
consequent pulmonary congestion and
edema.
E. Hypernatremia
Incorrect. An increased serum sodium
suggests loss of free water and
dehydration, not edema.
NEOPLASIA SECTION

A 44-year-old woman notes a lump in her left breast C. Stool culture with Shigella flexneri
while taking a shower. The nurse practitioner palpates Incorrect. Bacterial infections do not tend
a 3 cm firm, irregular, non-movable mass in the upper to increase the risk for adenocarcinoma.
outer quadrant of her left breast on physical D. Presence of HIV-1 RNA
examination. A fine needle aspiration of this mass is Incorrect. Adenocarcinomas of the colon
performed, and cytologically, the cells are consistent are generally not seen with significantly
with infiltrating ductal carcinoma. The mass is removed increased frequency in persons with
with lumpectomy along with an axillary lymph node immunodeficiency diseases.
dissection. Which of the following findings will best E. DNA topoisomerase I autoantibody
predict a better prognosis for this patient? Incorrect. Adenocarcinomas of the colon
A. Tumor cells strongly estrogen receptor are generally not seen with significantly
positive increased frequency in persons with
Incorrect. Many breast cancers are collagen vascular diseases.
estrogen receptor positive, which
suggests that hormonal therapy will be An experiment is conducted in which proliferating cells
helpful. are subjected to ionizing radiation. The ionizing
B. No metastases in the sampled lymph radiation leads to arrest in a checkpoint that monitors
nodes completion of DNA replication. It is observed that there
CORRECT. The lack of metastases are increased numbers of chromosomal abnormalities
suggests a lower stage and a better in these cells. Which of the following is the checkpoint
prognosis. In general, stage is a better affected by the ionizing radiation?
indicator of prognosis than grade. In A. G0/G1
general, low stage cancers are more Incorrect. This represents entry into the
amenable to surgical resection and cell cycle from the resting, non-
chance for cure. proliferating state. Connective tissue
C. Flow cytometric analysis with aneuploidy cells and hepatocytes can be stimulated
and a high S-phase to enter the cell cycle, but others such as
Incorrect. Both aneuploidy and a high S- neurons are terminally differentiated and
phase are characteristic for malignancies cannot.
and suggest a worse prognosis. B. G1/S
D. One relative who had a similar type of Incorrect. This is the cell cycle checkpoint
breast cancer at which defects in DNA are detected to
Incorrect. A history of breast cancer prevent replication with mutations or
suggests a greater risk for breast cancer; chromosomal breaks.
but does not predict prognosis. C. S/G2
E. High cytologic tumor grade Incorrect. This is the synthesis phase in
Incorrect. A higher grade suggests a which there is no checkpoint.
worse prognosis. D. G2/M
CORRECT. This is the second
A change in bowel habits prompts a 53-year-old checkpoint in the cell cycle; ionizing
woman to see her physician. On physical examination radiation activates this checkpoint,
there are no lesions noted on digital rectal examination, resulting in chromosomal abnormalities
but her stool is positive for occult blood. A colonoscopy in mitosis.
is performed and reveals a 6 cm friable exophytyic E. M/G0
mass in the cecum. A biopsy of this mass is performed Incorrect. This is the point at which a cell
and microscopic examination shows a moderately goes back to a resting state.
differentiated adenocarcinoma. Which of the following
laboratory findings is most likely to be present in this A clinical study is performed to determine the incidence
patient? of cancers in different countries. The data show that
A. K-RAS mutation in the neoplastic cells persons born in Japan and continuing to reside there
CORRECT. Many human carcinomas have an increased risk for cancer. Which of the
are associated with RAS mutations that following cancers is most likely seen with increased
contribute to oncogenesis. Many types of frequency in this population?
mutations can be present, and in general A. Breast
multiple mutations are present. Sporadic Incorrect. Breast cancer incidence is
cancers in adults can have many of the highest for women from Europe and
same mutations, such as APC and North America.
mismatch repair, that are found in familial B. Colon
cancers, but they are acquired and not Incorrect. Colon cancers are most
inherited. prevalent in developed nations and when
B. Neoplastic cells positive for vimentin diets are low in fiber.
Incorrect. Vimentin is a more typically C. Lung
marker for soft tissue malignancies Incorrect. Lung cancer is most related to
(sarcomas). Carcinomas are positive for smoking.
cytokeratin by immunohistochemistry.

Page 55 of 72
Neoplasia | Pathology Reviewer (Utah) | 56 / 72
D. Stomach C. Invasion
CORRECT. Gastric cancers are more CORRECT. Metastasis would be an
prevalent in Japan. Is this genetics or even better indicator, but invasion of
environment? The risk goes down with adjacent tissues suggests malignancy
emigration and subsequent generations more than the other items listed here.
living outside of Japan. The prevlance of Size increasing beyond 1 cm is also a risk
H. pylori infection may play a role. for malignancy in a colonic polypoid
Incidence of gastric cancer has lesion.
diminished in the U.S. The health care D. Increased nuclear/cytoplasmic ratio
system in Japan has effective screening Incorrect. A high N/C ratio is one feature
programs that diagnose many cases at of malignancy; but is not the best
the curable early gastric cancer stage. indicator.
E. Uterine E. Necrosis
Incorrect. Uterine cancers include Incorrect. Necrosis may occur for many
endometrium and cervix. The latter is reasons. Even a benign tumor could
related to HPV infection, and Pap smear cause pressure necrosis or outgrow its
screening has markedly reduced the risk blood supply.
for development of fatal cervical cancers.
Review of a series of surgical pathology reports
A 48-year-old woman has a routine physical indicates that a certain type of neoplasm is diagnosed
examination. A 4 cm diameter non-tender mass is as grade I on a scale of I to IV. Clinically, some of the
palpated in her right breast. The mass appears fixed to patients with this neoplasm are found to have stage I
the chest wall. Another 2 cm non-tender mass is disease. Which of the following is the best
palpable in the left axilla. A chest radiograph reveals interpretation of a neoplasm with these designations?
multiple 0.5 to 2 cm nodules in both lungs. Which of the A. Unlikely to be malignant
following TNM classifications best indicates the stage Incorrect. Criteria for malignancy must be
of her disease? satisfied first, then grading and staging
A. T1 N1 M0 follow. Benign neoplasms are by
Incorrect. This classification is for a small definition well-differentiated and localized.
primary cancer with nodal metastases B. Arising from epithelium
but no distant metastases. Incorrect. Grading and staging are most
B. T1 N0 M1 useful for epithelial malignancies; but are
Incorrect. This classification is for a small not reserved specifically for them.
primary cancer with no lymph node C. May spread via lymphatics and
metastases but with distant metastases. bloodstream
C. T2 N1 M0 Incorrect. A malignant neoplasm may
Incorrect. This classification is for a larger indeed spread to lymph nodes,
primary cancer with nodal metastases particularly if it is a carcinoma, or
but no distant metastases. hematogenously to distant sites, but is
D. T3 N0 M0 less likely to do so if it has a low grade
Incorrect. This classification is for a larger and it remains small and localized.
primary cancer with no metastases to D. Has an in situ component
either lymph nodes or to distant sites. Incorrect. It may have an in situ
E. T4 N1 M1 component, but the behavior of most
CORRECT. She has a large invasive neoplasms is judged by the worst part of
(high T) primary tumor mass with axillary it, and stage I puts it beyond in situ. The
node (N > 0) and lung (distant) designation Tis may be used for
metastases (M1). carcinoma in situ of epithelial neoplasms.
E. Well-differentiated and localized
A study is performed to analyze characteristics of CORRECT. A well-differentiated (low
malignant neoplasms in biopsy specimens. The grade) and localized (lower stage)
biopsies were performed on patients who had palpable malignant neoplasm often has both a low
mass lesions on digital rectal examination. Of the grade and low stage. In such cases
following microscopic findings, which is most likely to surgery to excise the lesion is more likely
indicate that the neoplasm is malignant? to be curative.
A. Pleomorphism
Incorrect. While cellular pleomorphism A 51-year-old man has worked for 10 years in a factory
(variation in shape and size) is more producing plastic pipe but not following safety
prominent in malignant neoplasms, it standards. He has noted weight loss, nausea, and
may be present in benign neoplasms, too. vomiting worsening over the past 5 months. On
B. Atypia examination he is afebrile. There is generalized muscle
Incorrect. Atypia can be part of benign or wasting. Laboratory studies show the serum alkaline
dysplastic processes that are not phosphatase is 405 U/L with AST 47 U/L, ALT 35 U/L,
malignant. Some mild atypia may and total bilirubin 1.2 mg/dL. An abdominal CT scan
accompany inflammatory processes. reveals a 12 cm right liver lobe mass. Liver biopsy
Dysplasias may be present in the reveals a neoplasm composed of spindle cells forming
pathway of progression to carcinomas. irregular vascular channels. With
immunohistochemical staining the cells demonstrate
vimentin positivity and cytokeratin negativity. Exposure
Neoplasia | Pathology Reviewer (Utah) | 57 / 72
to which of the following substances most likely led to A 22-year-old woman goes to her physician for a
development of this neoplasm? routine examination. A palpable nodule is found in the
A. Benzene right lobe of her thyroid gland. No lymphadenopathy is
Incorrect. Hydrocarbon exposure can noted. A chest x-ray shows no masses. A fine needle
predispose to development of leukemias. aspirate of the nodule is performed and cytologic
B. Radon examination reveals cells present consistent with a
Incorrect. Radon gas exposure can papillary carcinoma of the thyroid. There are no other
increase the risk for lung cancer. family members affected by this disorder. She works as
C. Cyclophosphamide an administrator for an accounting firm part time and is
Incorrect. Cyclophosphamide is an earning a college degree. Which of the following
alkylating agent used for chemotherapy, findings would you consider most relevant in her past
but this drug can promote leukemia, history to indicate a risk factor for this neoplasm?
lymphoma, and carcinoma. A. Chronic alcoholism
D. Asbestos Incorrect. Chronic alcoholism can lead to
Incorrect. Asbestos exposure can micronodular cirrhosis, and in this setting
increase the risk for lung cancer hepatocellular carcinomas can arise.
particularly in persons who smoke. It is B. Radiation therapy in childhood
also associated with the rare neoplasm CORRECT. Radiation is oncogenic.
known as mesothelioma. Cancers of thyroid and bone often
E. Vinyl chloride develop in children with radiation
CORRECT. This is a rare cause of exposure. Leukemias can occur as well.
cancer. However, this causal relationship C. Ataxia telangiectasia
is easy to demonstrate, because hepatic Incorrect. This is an inherited disorder of
angiosarcoma is a rare neoplasm. What DNA instability in which leukemias,
else could happen in the future? At least lymphomas, and breast cancers may
a hundred new chemical compounds are arise.
dumped into the environment each year. D. Blunt trauma from a fall
The elevated alkaline phosphatase is Incorrect. Trauma is not a risk for
characteristic for hepatic metastases as neoplasia.
well as bone metastases. E. Occupational exposure
Incorrect. Her jobs do not include any
A child is born with a single functional allele of a tumor typical exposure to carcinogens. An
suppressor gene. At the age of five the remaining example of an environmental exposure
normal allele is lost through a point mutation. As a would be arsenic, to which exposure can
result, the ability to inhibit cell cycle progression until increase the risk for skin cancer. For
the cell is ready to divide is lost. Which of the following years, pressure-treated lumber to
neoplasms is most likely to arise via this mechanism? prevent wood rot has relied upon use of
A. Breast ductal carcinoma arsenical compounds. There is a lot of
Incorrect. Most breast carcinomas are that treated wood out in the environment.
sporadic, though some are related to
BRCA1 and BRCA2 or other gene A 50-year-old man has felt vague abdominal discomfort
mutations which can be inherited, though for the past 4 months. On physical examination he has
the onset of disease occurs in young no lymphadenopathy, and no abdominal masses or
adults. organomegaly can be palpated. Bowel sounds are
B. Pulmonary small cell anaplastic present. An abdominal CT scan shows a 20 cm
carcinoma retroperitoneal soft tissue mass obscuring the left
Incorrect. Most small cell carcinomas of psoas muscle. A stool specimen tested for occult blood
the lung are related to smoking. is negative. Which of the following neoplasms is this
C. Ocular retinoblastoma man most likely to have?
CORRECT. The RB gene is the classic A. Melanoma
example of the 'two hit' mechanism for Incorrect. Melanomas arise on the skin in
loss of tumor suppression. About 60% of most cases, related to ultraviolet light
these tumors are sporadic, while others exposure, and are rarely visceral or in
are familial, and there is inheritance of a soft tissue.
mutated copy of the RB gene. Loss of the B. Hamartoma
second copy in retinoblasts leads to the Incorrect. A hamartoma is a peculiar
occurrence of retinoblastoma in small benign neoplasm composed of
childhood. tissues normal to a site, but just in a
D. Cerebral astrocytoma jumbled mass. A pulmonary hamartoma
Incorrect. Gliomas of the brain are mostly is the most common of these.
sporadic. C. Adenocarcinoma
E. Chronic myeloid leukemia Incorrect. Adenocarcinomas do not arise
Incorrect. CML is most often associated in soft tissues. Sarcomas arise in soft
with the 'Philadelphia chromosome' that tissues.
is acquired in adulthood. D. Lymphoma
Incorrect. It is unlikely that matted nodes
with lymphoma would reach this size.
Neoplasia | Pathology Reviewer (Utah) | 58 / 72
E. Liposarcoma that increases the risk for subsequent
CORRECT. Sarcomas are big and bad. mutations that drive carcinogenesis.
They arise in soft tissues derived from D. Prostatic adenocarcinoma
mesenchymal (connective) tissues. Incorrect. Adenocarcinomas of the
Retroperitoneum is a typical location for prostate are common; but are not
a sarcoma arising in an adult. Sarcomas associated with preceding viral infection.
are much less common than carcinomas E. Hepatic angiosarcoma
in adults. Incorrect. Hepatic angiosarcoma is
causally related to vinyl chloride
A 52-year-old man has had increasing fatigue for the exposure.
past 6 months. On physical examination he has a
palpable spleen tip. Laboratory studies show a WBC An experiment is designed to study the genetics of
count of 189,000/microliter. The peripheral blood cancer. The study will link the appearance of cancer to
smear shows many mature and immature myeloid cells specific gene abnormalities. Which of the following
present. Cytogenetic analysis of cells obtained via forms of cell molecular analysis is most useful to
bone marrow aspiration reveals a t(9:22) translocation. identify gene alterations involved in carcinogenesis?
This translocation leads to formation of a hybrid gene A. Florescence in situ hybridization
that greatly increases tyrosine kinase activity. Which of Incorrect. FISH is useful to identify
the following genes is most likely translocated to cause abnormal expression of known genes,
these findings because a specific marker is needed.
A. p53 The best example is detection of
Incorrect. p53 is a tumor suppressor HER2/neu amplification in breast
gene, and mutations can be found in a cancers.
variety of human carcinomas. B. Flow cytometry
B. RB Incorrect. Flow cytometry analyzes DNA
Incorrect. RB is a tumor suppressor gene content and can use markers for known
best known for association with components of cells.
retinoblastomas of the eye. C. Immunohistochemistry
C. ABL Incorrect. Immunohistochemistry
CORRECT. This is the 'Philadelphia identifies protein products of gene
chromosome' of chronic myelogenous expression; and is most useful to
leukemia (CML) and an example of an determine the cell of origin of a cancer, or
abnormal fusion gene driving cell identify a characteristic of that cancer,
proliferation. Therapy with a biologic such as estrogen receptor expression.
agent such as a tyrosine kinase inhibitor D. Single nucleotide polymorphisms
(TKI), a monoclonal antibody that targets CORRECT. SNPs can be analyzed to
the abnormal protein produced by the find alterations present in cancer cells.
BCR-ABL fusion gene is often effective. One could analyze multiple histologically
First generation TKIs include imatinib, similar cancers to determine what gene(s)
and more effective 2nd generation TKIs alteration(s) such as deletions and
include dasatinib, nilotinib, and bosutinib. amplifications may underlie
D. NF-1 carcinogenesis. Using millions of SNPs
Incorrect NF-1 is a tumor suppressor can produce a virtual karyotype down to
gene associated with neurofibromatosis. the gene level.
E. RAS E. Tumor marker expression
Incorrect. RAS is an oncogene found in Incorrect. Serum tumor markers, such as
many human carcinomas. prostate specific antigen, utilize known
characteristics of cancer protein
A clinical study is performed of oncogenesis in human expression. Tumor markers tend to be
neoplasms. It is observed that some neoplasms appear insensitive for early detection of cancer.
to develop from viral oncogenesis, with serologic
confirmation of past viral infection. Which of the A 29-year-old woman with a history of multiple sexual
following neoplasms is most likely to arise in this partners over the last 15 years has a routine physical
manner? examination with no abnormal findings. On pelvic
A. Retinoblastoma examination, the cervix shows no abnormalities, but a
Incorrect. Retinoblastomas occur with Pap smear is taken and dysplastic cells are reported to
loss of alleles for a suppressor oncogene be present. A cervical biopsy is performed and shows
on chromosome 13. a low grade squamous intraepithelial lesion (LSIL), also
B. Small cell anaplastic carcinoma known as cervical intraepithelial neoplasia 1 (CIN-1).
Incorrect. Most of these cancers are Which of the following is the most appropriate
related to smoking. therapeutic strategy for this lesion?
C. T-cell leukemia A. No further treatment
CORRECT. Human T-lymphocytotropic Incorrect. Dysplasias should not be
virus (HTLV) infection can lead to T-cell ignored, because they can progress to
leukemia. However, few human cancers carcinomas.
arise as a result of viral oncogenesis. B. Antibiotic therapy
Some viruses, such as hepatitis viruses Incorrect. Cervical dysplasias are
B and C, result in cellular proliferation associated with human papillomavirus
infection, but HPV cannot be treated with
Neoplasia | Pathology Reviewer (Utah) | 59 / 72
antibiotic therapy. In general, most E. Decreased lactate dehydrogenase
women will clear the virus. However, it is Incorrect. The LDH is elevated in a
a sexually transmitted disease, and variety of disease conditions and never
further infections can occur. decreased. In this case, metastatic
C. Chemotherapy disease and tissue necrosis may have
Incorrect. Dysplasias are confined to the elevated the LDH.
epithelium, so there is no benefit to
chemotherapy. A 14-year-old healthy girl has a 0.3 cm reddish, slightly
D. Colposcopy raised nodule on the skin of the upper part of her chest
CORRECT. Cervical dysplasia is a pre- found on a routine physical examination. She states
neoplastic condition that has the potential that this lesion has been present for years and has not
to develop into a carcinoma if not treated. appreciably changed in size or color. Which of the
Epithelia subjected to initiators and following neoplasms is this nodule most likely to be?
promoters of carcinogenesis tend to A. Hemangioma
undergo progression from metaplasia to CORRECT. Hemangiomas, teratomas,
dysplasia to carcinoma. Even a and fibromatoses are common pediatric
carcinoma is essentially 100% curable neoplasms that are benign. Benign
when it is limited to the epithelium (in situ). neoplasms tend to grow slowly and
That is why you try to diagnose the lesion remain localized. They do not invade or
as early as possible. Some LSILs on Pap metastasize.
smear have a more advanced lesion B. Melanoma
present with colposcopic screening and Incorrect. Malignant melanomas are
warrants resection. Just LSIL may be unlikely to appear in the first two decades
followed with Pap smears, as some of life. They are malignancies that are
women with HPV infection will clear the characterized by change-growth and
virus. High-grade squamous differences in coloration.
intraepithelial lesions (HSIL) correlate C. Carcinoma
with CIN 2 or 3, which are less likely to Incorrect. Carcinomas are generally
regress and are therefore resected. uncommon in children. Carcinomas tend
E. Radiation therapy to enlarge and invade over time.
Incorrect. Dysplasias are confined to the D. Lymphoma
epithelium, and there is no benefit to Incorrect. Lymphomas are never benign.
radiation therapy. Leukemias and lymphomas are
malignancies that can occur in childhood,
A 64-year-old man has noted a 5 kg weight loss along but they rarely involve skin.
with increasing fatigue over the past year. He has E. Glioma
experienced dull abdominal pain for the past week. He Incorrect. Gliomas are not benign. They
has developed abdominal distention with lack of stools are one of the common childhood
in the past two days. On physical examination, bowel malignancies and occur in the posterior
sounds are reduced. An abdominal CT scan reveals a fossa of the brain most often.
mass involving the descending colon. At laparotomy, a F. Sarcoma
partial resection of the left colon is performed, with Incorrect. Sarcomas can arise in soft
removal of an encircling mass. Microscopically, the tissues such as the dermis, but this is not
mass is found to be a moderately differentiated a common site for such a neoplasm in
adenocarcinoma. Which of the following laboratory test children. Sarcomas comprise a larger
findings is most likely to be present in this man? percentage of malignancies in children
A. Microcytic hypochromic anemia than in adults, but only because
CORRECT. He has an adenocarcinoma carcinomas are far less common in
of the colon. Such lesions are often children than adults. Cancers in general
associated with blood loss from mucosal are not common in children, but the ones
erosion and necrosis. Cancer cells are that do occur can be aggressive.
less differentiated and less functional
than their normal cell of origin and thus A 56-year-old man has had a chronic cough for the past
less functional. With poor construction year. He is a non-smoker. He had an episode of
come many problems. hemoptysis 3 days ago. No abnormal findings are
B. Positive antinuclear antibody test noted on physical examination. A chest x-ray
Incorrect. Colonic adenocarcinoma is not demonstrates a 6 cm perihilar mass. A sputum sample
typically associated with autoimmune is collected, and the sputum cytology report reads
diseases. 'Atypical cells present suggestive of squamous cell
C. Hyperglycemia carcinoma.' Which of the following environmental
Incorrect. Hyperglycemia is unlikely, exposures is most likely to be associated with these
though inanition with hypoglycemia is findings?
possible. A. Asbestos
D. Elevated alpha-fetoprotein Incorrect. Asbestos exposure is a risk for
Incorrect. Elevation of the AFP in bronchogenic carcinoma in persons who
association with neoplasia most often are smokers. It is rare.
occurs with testicular and liver cancers.
Neoplasia | Pathology Reviewer (Utah) | 60 / 72
B. Radon A 55-year-old man has had malaise and a 4 kg weight
CORRECT. Radon gas is considered to loss over the past 6 months. On physical examination
be the second most common cause for his stool is positive for occult blood. An abdominal CT
lung cancer following tobacco smoking. scan shows his liver contains multiple tumor masses
However, most persons with radon from 2 to 5 cm in size with central necrosis. The
exposure who develop lung cancer are surrounding hepatic parenchyma appears normal.
also smokers. A small amount of radon Which of the following characteristics of neoplasia is
gas naturally seeps up from the soil and best illustrated by these findings?
may collect in houses. Commercial A. Multicentric origin
buildings tend to be larger and well Incorrect. Necrosis is unlikely to be a
ventilated so little radon collects in them. feature of a benign neoplasm. The liver is
C. Silica not a common site for multiple benign
Incorrect. The risk for lung cancer is only masses. A primary hepatocellular
slightly increased with silica dust carcinoma could be multicentric; but
exposure. Silica is more likely to produce arises in the context of chronic
a restrictive lung disease. hepatitis/cirrhosis, not normal liver.
D. Benzene B. High tumor grade
Incorrect. Benzene, as are other Incorrect. The grade of a tumor is a
hydrocarbon compounds, is more likely microscopic diagnosis and indicates the
to affect the bone marrow. degree of differentiation.
E. Formaldehyde C. Primary neoplasm in the stomach
Incorrect. Formaldehyde is unpleasant Incorrect. The primary neoplasm could
but not a major risk for lung cancer. be in the stomach, but more likely the
colon, and any number of primary sites
A 60-year-old man who has a 90-pack year history of could exist for such an appearance.
cigarette smoking has had a chronic cough for the past D. Exposure to a carcinogen
10 years. He has begun to lose weight (3 kg) during the Incorrect. It is not possible to state this
past year. No abnormal findings are noted on physical with certainty from the radiographic
examination. He has a chest radiograph that reveals a appearance alone.
right hilar mass. A sputum cytology shows atypical, E. Advanced stage
hyperchromatic squamous cells. What is the most CORRECT. The most likely possibility is
common initial pathway for metastases from this lesion? that these masses represent metastases
A. Bloodstream from a primary site elsewhere in the body,
Incorrect. Epithelial malignancies spread likely the colon because of blood in the
late via the bloodstream. Sarcomas stool. Thus, this is a malignant neoplasm
spread early via bloodstream. that has an advanced stage (M1). The
B. Pleural cavity central necrosis is the result of tumor
Incorrect. Seeding through body cavities, necrosis with collapse of a tumor nodule.
such as pleural cavities or peritoneal
cavity, is possible for a cancer, but more A 59-year-old man has had a worsening cough with
of a late finding for a squamous cell chest pain for the past 6 months. On physical
carcinoma. examination he has no remarkable findings. A chest x-
C. Contiguous spread to chest wall ray shows a 3 cm left lung mass. A sputum cytology
Incorrect. Direct spread to chest wall is a specimen yields cells diagnosed as a squamous cell
late finding with a central squamous cell carcinoma. A mediastinoscopy is performed and
carcinoma. Contiguous spread is a form reveals metastases in a lymph node. He is given
of invasion, which is technically not a radiation therapy, and the mass diminishes in size.
metastasis, but still bad. Which of the following cellular mechanisms is most
D. Lymphatics likely to account for this tumor response?
CORRECT. Epithelial malignancies such A. Point mutations in DNA
as a squamous cell carcinoma of the lung Incorrect. Mutations occurring in some
often have non-contiguous (metastatic) irradiated cells would have no significant
spread initially to regional lymph nodes. immediate effect. Exposure to
This is important to determine for staging, environmental radiation has this effect,
because the treatment and prognosis which increases the risk for cancer.
depend upon accurate staging. Lymph B. Generation of free radicals
node metastases or hematogenous CORRECT. The purpose of therapeutic
metastases imply the malignancy is radiotherapy is to try and kill more
systemic and local excision of the lung neoplastic cells than normal cells. This is
mass will not be curative. mediated mainly through direct damage
E. Bronchi to the cells and one mechanism is
Incorrect. This is a very unlikely mode of generation of oxidant free radicals.
spread for a squamous cell carcinoma; Cancer cells are more vulnerable than
however, the cells can be shed into the normal cells. Techniques are available to
bronchial tree and that is how they are focus the beam of radiation energy on the
picked up in a sputum specimen. cancer and reduce collateral damage to
normal tissues.
Neoplasia | Pathology Reviewer (Utah) | 61 / 72
C. Loss of the blood supply C. Hemangioma
Incorrect. Vascular changes do occur Incorrect. This is an unusual place for a
following radiation; but are not a desired hemangioma, formed of small vascular
effect. channels.
D. Secondary inflammation D. Leiomyoma
Incorrect. Necrosis with inflammation will CORRECT. The slow growth of the
occur following radiation, but the desired nodule, the circumscribed nature of the
effect is to kill neoplastic cells directly nodule, and the continuing lack of
Inflammation by itself does not have symptoms suggests a benign course.
diagnostic or prognostic value in Leiomyomas of the uterus are quite
neoplasia. The immune system may play common (up to 50% of women have at
a role in tumor surveillance to prevent least one).
malignancies from occurring, but once a E. Metastasis
malignancy is established, the immune Incorrect. Solitary slow-growing
system is unlikely to eliminate it unaided. metastases are quite rare.
E. Adenosine triphosphate depletion
Incorrect. ATP depletion may occur as A 15-year-old boy has felt lumps in his right neck for
the neoplastic cells die, but this is not the the past 5 months. On physical examination there is
main effect of radiotherapy. painless lymphadenopathy in the right cervical region.
One of the lymph nodes is biopsied and on microscopic
A 61-year-old woman has a firm mass with irregular examination shows effacement of the nodal
borders felt in her left breast on a routine physical architecture by many lymphocytes that are large, with
examination. A fine needle aspiration is performed and clumped chromatin and occasional mitoses. The
microscopic examination shows malignant cells. A left characterization of this population of lymphocytes as a
mastectomy with axillary lymph node dissection is neoplasm is best accomplished by which of the
performed. A tissue sample of this neoplasm is following methods?
submitted for analysis by flow cytometry. Which of the A. Immunohistochemical stain for leukocyte
following does flow cytometric analysis most likely common antigen
provide? Incorrect. Staining with LCA will only tell
A. Analysis of the karyotype you that cells are of leukocyte origin, not
Incorrect. Flow cytometry does not whether they are part of a neoplastic
distinguish individual chromosomes or proliferation.
count them. B. Gene rearrangement studies
B. Detection of gene mutations demonstrating clonality
Incorrect. Flow cytometry cannot be used CORRECT. Clonality is a key
to analyze individual genes. characteristic of neoplastic cellular
C. Determination of aneuploidy proliferations that distinguishes them
CORRECT. Flow cytometry can detect from reactive proliferations (such as
the degree of aneuploidy from the DNA inflammation).
content. Aneuploidy implies that the C. Flow cytometry indicating high S-phase
neoplasm is worse (more aggressive) Incorrect. Flow cytometry cannot tell you
than if it were not aneuploid. with certainty that a neoplasm is present.
D. Distinguishing carcinoma from sarcoma D. Differential white blood count showing a
Incorrect. Flow cytometry cannot lymphocytosis
distinguish the cell of origin of neoplasms. Incorrect. Inflammation could lead to
E. Quantitation of RNA content lymphocytosis. Lymphomas are unlikely
Incorrect. Flow cytometry is used to to have a lymphocytosis.
measure DNA content, not RNA content. E. Light microscopy demonstrating nodal
Incorrect. Fibrosis is a feature of
A 35-year-old healthy woman had a firm nodule inflammation as well as neoplasia.
palpable on the dome of the uterus six years ago
recorded on routine physical examination. The nodule A previously healthy 42-year-old man has a skin nodule
has slowly increased in size and now appears to be on his right hand that has become larger and darker
about twice the size it was when first discovered. By with more irregular outlines over the past 3 months. On
ultrasound scan it is solid and circumscribed. She physical examination this lesion is 1.2 cm diameter,
remains asymptomatic. Which of the following darkly pigmented, and a slightly raised nodule on the
neoplasms is she most likely to have? dorsum of his right hand. No other skin lesions are
A. Adenocarcinoma noted. Three non-tender enlarged lymph nodes are
Incorrect. Adenocarcinomas of the uterus palpable in the right axilla. The lesion is excised and
would arise in the endomtrium and are microscopic examination shows a neoplasm
often accompanied by hemorrhage. composed of darkly pigmented polygonal and spindle
B. Leiomyosarcoma cells. Which of the following risk factors is most
Incorrect. The slow growth and her important for development of this neoplasm?
asymptomatic condition suggest a A. Cigarette smoking
benign neoplasm. Leiomyosarcomas of Incorrect. Smoking is associated with
the uterus do occur; but are not common. many malignancies, including those of
lung and the urinary tract, but it is not a
risk for skin cancers.
Neoplasia | Pathology Reviewer (Utah) | 62 / 72
B. Allergy to latex gloves glucose is 157 mg/dL. She has been a commercial sex
Incorrect. Allergies do not give rise to worker in the past. Which of the following is the most
malignancies. likely risk factor for her cervical carcinoma?
C. Inheritance of a faulty RB gene A. Human papillomavirus infection
Incorrect. The Rb gene is a tumor CORRECT. HPV infection, a sexually
suppressor gene that can be involved transmitted disease (STD), is highly
with retinoblastomas and osteosarcomas associated with cervical dysplasias and
when a mutation is inherited. malignancies, particularly high-risk
D. Prior job-related handling of asbestos subtypes 16 and 18 that are now covered
Incorrect. Asbestos exposure can by the HPV vaccine. Syphilis is not a risk
increase the risk for bronchogenic for epithelial dysplasias or carcinomas,
carcinomas, mesothelioma, and but if one STD is present, others can be
gastrointestinal adenocarcinomas. present as well.
E. Chronic exposure to ultraviolet radiation B. Diabetes mellitus, type II
CORRECT. Worldwide, more and more Incorrect. Diabetes mellitus increases the
skin cancers, including melanoma, are risk for bacterial infections; but does not
occurring because of sun exposure. The increase cancer risk.
UV light damages the skin and damages C. Heavy cigarette smoking
cellular DNA, leading to mutations that Incorrect. Cigarette smoking is a risk for
escape cellular repair mechanisms. many cancers, particularly of the lung,
Depletion of the earth's protective upper but not for cervical carcinoma.
atmospheric ozone layer has not helped, D. Pelvic inflammatory disease
either. The ABC's of malignant Incorrect. PID is most often the result of
melanoma: Asymmetrical shape, gonorrheal or chlamydial infections and
irregular Border, abnormal Color. is not a risk for cancer.
E. Previous cancer chemotherapy
A 27-year-old woman in excellent health has a routine Incorrect. Previous chemotherapy can
health maintenance examination. A 2 cm firm, rounded increase the risk for malignancies in
mass is palpable beneath the skin of the left forearm. general, but not significantly for cervical
She has no difficulty using the arm and there is no carcinoma.
associated pain with the mass, either in movement or
on palpation. The overlying skin appears normal. The A 62-year-old man with an 80-pack year history of
mass does not change in size over the next year. smoking experiences an episode of hemoptysis. On
Which of the following neoplasms is she most likely to physical examination he has puffiness as well as
have? plethora of the face, pedal edema, bruises of the skin,
A. Metastatic carcinoma and a blood pressure of 165/100 mm Hg. A chest
Incorrect. A patient with widely radiograph reveals a 5 cm right upper lobe lung mass.
disseminated cancer would not be in A fine needle aspirate of this mass yields cells
excellent health, and there would likely consistent with small cell anaplastic lung carcinoma. A
be many other masses. bone scan shows no metastases.
B. Melanoma C Immunohistochemical staining of the tumor cells is
Incorrect. Melanomas arise in epidermis likely to be positive for which of the following hormones?
and are typically visible on the skin A. Parathormone related peptide
surface. They may infiltrate downward. Incorrect. Prp can be secreted as part of
C. Rhabdomyosarcoma a paraneoplastic syndrome and leads to
Incorrect. Such a small, unchanging hypercalcemia. Of lung neoplasms, the
mass in a young person is unlikely to be one most likely to do this is a squamous
a sarcoma. cell carcinoma.
D. Lipoma B. Erythropoietin
CORRECT. The small size of the lesion, Incorrect. Erythropoietin secretion leads
the relatively young age of the patient, to increased production of red blood cells.
and the lack of other findings suggests a Renal cell carcinomas can do this.
benign process. Lipoma is the most C. ACTH
common soft tissue neoplasm. Most of CORRECT. He has findings of Cushing
them never reach a size sufficient to syndrome from ectopic ACTH production
produce problems. by the lung tumor, a form of
E. Leiomyoma paraneoplastic syndrome common to
Incorrect. Leiomyomas can theoretically small cell carcinomas of the lung, which
arise in any location with smooth muscle, have neuroendocrine derivation.
but in a woman they are most likely to be D. Insulin
found in the myometrium. Incorrect. Insulin can be secreted by
neuroendocrine neoplasms, such as
A 41-year-old woman has noted a foul-smelling vaginal pancreatic neuroendocrine (islet cell) and
discharge for 3 weeks. On physical examination there gastrointestinal neuroendocrine
is an exophytic 3 cm mass involving the ectocervix. (carcinoid) tumors.
Pap smear testing is performed; she has never had a E. Gastrin
previous Pap smear. Cytologic changes are present Incorrect. Gastrin can be secreted by
consistent with squamous cell carcinoma. She is found neuroendocrine neoplasms, such as
to have a positive serologic test for syphilis. Her serum pancreatic neuroendocrine (islet cell) and
Neoplasia | Pathology Reviewer (Utah) | 63 / 72
gastrointestinal neuroendocrine C. JAK/STAT
(carcinoid) tumors. Incorrect. The JAK-STAT pathway
promotes neoplastic cell proliferation, but
A 45-year-old woman has noted a lump on her left is not directly targeted by this agent.
shoulder that has enlarged over the past 4 months. On D. TGF-beta
physical examination there is a palpable non-tender Incorrect. TGF-beta signaling represses
supraclavicular lymph node. A biopsy of the node is several cyclins and CDKs, as well as C-
performed and on microscopic examination there is a MYC, to reduce cell proliferation.
metastatic neoplasm. Which of the following is the most E. WNT
likely primary for this neoplasm? Incorrect. The WNT signaling pathway is
A. Cerebral glioma involved in cell proliferation and adhesion,
Incorrect. Gliomas are primary malignant but is not targeted by this agent.
brain neoplasms that rarely metastasize
outside the cranial cavity. Columnar epithelial cells from the colonic mucosa are
B. Adenocarcinoma of the stomach studied to identify abnormalities in cell signaling
CORRECT. Lymph node metastases are pathways. Abnormal epithelial cells from colonic
typical of spread from carcinomas. The adenocarcinoma are shown to have a mutation that
first stop along the way is regional nodes; blocks hydrolysis of GTP-bound active RAS. Normal
but spread to more distant lymphatic columnar cells have active RAS protein that undergoes
sites is possible. The right hydrolysis to the inactive GDP-bound form. Which of
supraclavicular node receives drainage the following signaling pathways is most likely
from mid-section of the chest, esophagus, abnormally stimulated in the carcinoma cells?
and lungs. Described here is classic A. ADP
'Virchow node' that involves left Incorrect. ADP is released from platelets
supraclavicular nodal metastasis from to signal activation and aggregation of
drainage via the thoracic duct, the surrounding platelets.
abdomen, and thorax. B. BCR-ABL
C. Fibroadenoma of the breast Incorrect. The BCR-ABL fusion gene
Incorrect. A fibroadenoma is a benign product is the 'Philadelphia chromosome'
lesion that does not metastasize. of chronic myelogenous leukemia that
D. Liposarcoma of the retroperitoneum has tyrosine kinase activity.
Incorrect. Sarcomas typically do not C. Cyclic AMP
spread via lymphatics. Sarcomas are far Incorrect. Cyclic AMP is not generally
less common than carcinomas. considered part of neoplastic processes.
E. Laryngeal papilloma cAMP is a second messenger from
Incorrect. A papilloma is a benign lesion. activation of G-protein coupled cell
Laryngeal papillomas may recur locally, surface receptors.
but do not spread elsewhere. D. Cyclin D1
Incorrect. Cyclin D appears in the mid G1
A 59-year-old man has had fatigue for the past year. phase of the cell cycle and is part of the
The only abnormal physical examination finding is a cell cycle machinery that is influenced by
positive stool guaiac test. Laboratory studies show a oncogenes.
CBC with Hgb 10.0 g/dL, Hct 29.8&, MCV 73 fL, E. MAP kinase
platelet count 300,000/microliter, and WBC count CORRECT. The mechanism described is
8700/microliter. He is found on colonoscopy to have a that of RAS oncogene activation in which
4 cm mass lesion arising on the mucosal surface of the the active RAS is permanently turned on
transverse colon. An abdominal CT scan shows and through the MAP kinase pathway
hepatic masses. He receives therapy with cetuximab activates transcription that drives
directed at vascular endothelial growth factor (VEGF). abnormal, uncontrolled cellular
Which of the following cell signaling pathways is most proliferation that leads to neoplasia.
affected by this therapy? Many carcinomas, and
A. Hedgehog adenocardinomas in particular, have
Incorrect. The PTCH genes are involved RAS mutations.
in encoding cell membrane 'patched' F. p53
protein functioning as a receptor for Incorrect. The p53 gene product is a
hedgehog proteins involved in cell tumor suppressor, and loss of both p53
proliferation. Many basal cell carcinomas alleles results in loss of control over cell
of skin have PTCH mutations. proliferation when DNA damage occurs.
B. Notch
CORRECT. Cancers must elaborate A 52-year-old woman feels a lump in her right breast.
growth factors in order to produce an On physical examination there is a 3 cm right breast
environment for continued growth. VEGF mass fixed to the chest wall. This mass is biopsied and
promotes angiogenesis that will keep the on microscopic examination shows nests of cells with
neoplasm supplied with blood. VEGF marked hyperchromatism and pleomorphism. These
increases the expression of ligands that cells are estrogen receptor positive. Flow cytometry is
active the Notch signaling pathway. performed. Compared with surrounding non-neoplastic
Biotherapies with monoclonal antibodies stromal cells, the neoplastic cells are more likely to be
can specifically target protein in which of the following phases of the cell cycle?
components of cancers.
Neoplasia | Pathology Reviewer (Utah) | 64 / 72
A. G0 In an experiment, it is observed that chronic, increased
Incorrect. G0 cells are quiescent and not exposure to ionizing radiation results in damage to
participating in the cell cycle. This cellular DNA. As a consequence, a protein is now
includes most cells in the body. absent that would arrest the cell in the G1 phase of the
B. G1 cell cycle. Subsequent to this, the cell is transformed to
Incorrect. G1 cells have just entered into acquire the property of unregulated growth. The absent
the cell cycle. protein is most likely the product of which of the
C. S following genes?
CORRECT. A high S-phase, as well as A. RAS
aneuploidy, are features often seen in Incorrect. The RAS oncogene 'turns on'
cancers. The hyperchromatism typical for cells to promote carcinogenesis.
malignant cells indicates active DNA B. TP53
synthesis. The estrogen receptor CORRECT. Abnormal TP53 protein has
positivity suggests that this breast cancer been identified in many carcinomas. This
will respond to hormonal therapy with loss of normal wild type TP53 with tumor
letrozole or tamoxifen. suppression leads to loss of tumor
D. G2 suppression driving development of
Incorrect. G2 is the part of the cell cycle carcinomas.
just before division. C. MYC
E. M Incorrect. The MYC oncogene 'turns on'
Incorrect. M is the part of the cell cycle in cells to promote carcinogenesis.
which cells are dividing. Although more D. ABL
mitoses can be seen in cancers than in Incorrect. The BCR-ABL fusion gene
normal tissues, the number of cells in results from a 9:22 translocation
mitosis is still not high. (Philadelphia chromosome of chronic
myelogenous leukemia) that promotes
A 73-year-old man has an episode of hematemesis. cellular transformation via protein
Upper GI endoscopy reveals an irregular 4 cm gastric kinases.
antral ulceration. Biopsies are performed and E. BCL-2
microscopically reveal crowded irregular gland-like Incorrect. The BCL-2 gene acts by
collections of cells with hyperchromatism and decreasing apoptosis (cell death) to
pleomorphism. Molecular analysis shows DNA cause accumulation of cells, as with
hypermethylation of the CDKN2 complex. Through follicular lymphomas.
which of the following mechanisms has this abnormal
gene expression most likely occurred? A 50-year-old woman notes a lump in her breast. Her
A. Amplification physician's assistant palpates a 2 cm firm mass in her
Incorrect. Amplification of gene left breast. A fine needle aspiration biopsy is performed,
expression typically occurs when there is and on microscopic examination a carcinoma is
reduplication of DNA sequences, often present. Molecular analysis shows HER2 positivity but
with hundreds of copies, greatly estrogen receptor negativity in these malignant cells.
increasing the amount of protein Through which of the following mechanisms has this
produced, which drives cellular abnormal expression most likely occurred?
proliferation. A. Amplification
B. Epigenetic alteration CORRECT. Amplification of gene
CORRECT. Epigenetic changes are expression typically occurs when there is
common in cancers, and gastric reduplication of DNA sequences, often
adenocarcinoma is described here. with hundreds of copies, greatly
Though the DNA may not be altered, increasing the amount of protein
methylation can occur to silence DNA produced, which drives cellular
expression. If selective hypermethylation proliferation. The mutation in HER2
of a DNA sequence encoding a tumor results in amplification of the epithelial
suppressor occurred, then the result growth factor receptor, as identified by
would be similar to loss of the tumor presence of multiple copies by FISH
suppressor gene itself. Genomic analysis.
imprinting works the same way. B. Epigenetic alteration
C. Growth factor binding Incorrect. Epigenetic changes are
Incorrect. Binding of growth factors is one common in cancers. The DNA is not
way to control cell signaling, but this does altered, but methylation occurs to silence
not affect DNA methylation directly. DNA expression. If selective
D. Point mutation hypermethylation of a DNA sequence
Incorrect. Point mutations alter the DNA encoding a tumor suppressor occurred,
directly; methylation affects expression of then the result would be similar to loss of
normal DNA. the tumor suppressor gene itself.
E. Reduced miRNA expression Genomic imprinting works the same way.
Incorrect. Micro RNAs often act to inhibit C. Growth factor binding
oncogene translation. Hence, reduced Incorrect. Binding of growth factors is one
miRNA expression has the same effect way to control cell signaling, but this does
as loss of a tumor suppressor gene. not affect DNA methylation directly.
Neoplasia | Pathology Reviewer (Utah) | 65 / 72
D. Point mutation D. Pseudomonas aeruginosa
Incorrect. Point mutations alter the DNA Incorrect. Bacterial infections do not
directly; methylation affects expression of predispose to neoplasia, but persons
normal DNA. with neoplasms may have more
E. Reduced miRNA expression infections.
Incorrect. Micro RNAs often act to inhibit E. Pneumocystis jiroveci
oncogene translation. Hence, reduced Incorrect. Pneumocystis jiroveci can be
miRNA expression has the same effect seen in persons with AIDS, but it is not
as loss of a tumor suppressor gene. related to the AIDS-associated
malignancies.
An epidemiologic study is performed to find risk factors
for development of malignant neoplasms. A statistical A 66-year-old woman has noted increasing abdominal
analysis of pre-existing medical conditions is done. girth for 3 months. On examination there is an
Some pre-existing chronic medical conditions are abdominal fluid wave. Paracentesis yields cloudy fluid
observed to precede development of malignant that cytologically contains clusters of malignant cells.
neoplasms, while others do not. Which of the following An abdominal CT scan shows bilateral cystic and solid
conditions is most likely to be statistically related to pelvic masses and multiple peritoneal nodules. She is
development of a malignancy? treated with chemotherapy with an initial response, but
A. Essential hypertension a year later there is still residual tumor. Which of the
Incorrect. Systemic hypertension is a following parameters of the neoplastic cells best
common condition; but is not predicts her diminished chemotherapy response?
premalignant. Hypertension promotes A. Doubling time
atherogenesis. Incorrect. The doubling time indicates
B. Coronary artery disease how quickly the neoplastic cells are
Incorrect. Coronary artery dividing, and if no other factors affected
atherosclerosis leads to ischemic heart growth, then a palpable 1 cm mass of a
disease, but this is not premalignant. billion cells would be present in just 90
C. Chronic bronchitis days (30 doublings), but in reality, there
Incorrect. Chronic bronchitis itself is not are limits to cell growth as well as cell
premalignant. However, cases may loss, so this generally takes years. In
occur in conjunction with inhaled general, a rapidly proliferating mass will
carcinogens such as cigarette smoke. have a fast doubling time and be more
D. Ulcerative colitis responsive to chemotherapy, but this
CORRECT. Adenocarcinoma of the does not give the best indication of why
colon is more frequent in persons with chemotherapy effectiveness diminishes.
chronic ulcerative colitis, an inflammatory B. Extent of apoptosis
bowel disease that leads to proliferative Incorrect. Apoptosis determines cell loss.
changes in which mutations may occur In general, apoptosis is slowed in
that contribute to colonic carcinogenesis. cancers. If one could turn on apoptosis in
E. Uterine leiomyomas cancer, then the cancer would self-
Incorrect. Many women have multiple destruct. However, this does not predict
uterine leiomyomas. Leiomyosarcomas diminished response to chemotherapy.
of the myometrium are rare and develop C. Gene amplification
de novo. Incorrect. Gene amplification is one way
where cell proliferation is promoted for
A 35-year-old man has noted several 1 to 2 cm reddish carcinogenesis, but this does not predict
purple, nodular lesions present on the skin of his right diminished response to chemotherapy.
arm which have increased in size and number over the D. Growth fraction
past 3 months. The lesions do not itch and are not CORRECT. The proportion of cancer
painful. He has had a watery diarrhea for the past cells that are actively proliferating will
month. On physical examination he has generalized predict response to chemotherapy. Cells
lymphadenopathy and oral thrush. Which of the not actively proliferating are generally
following infections is most likely to be related to the unresponsive to chemotherapeutic
appearance of these skin lesions? agents. As cancers increase in size,
A. Candida albicans more cancer cells exit the replicative pool.
Incorrect. Candida infection is not a risk During the course of chemotherapy, the
for malignancy, but persons with cells in replication are selectively killed,
malignancies treated with chemotherapy leaving behind the cells out of the pool
are more likely to get candidal infections. that can continue on. If one were to
B. Human herpesvirus 8 completely surgically excise the mass,
CORRECT. These plaque-like reddish then that would solve this issue, but even
lesions are characteristic for Kaposi if the mass cannot be removed, then at
sarcoma. HHV8 (KSHV) is the agent that least one can debulk it, or use radiation
promotes angiogenesis from which the therapy to kill more cells. Chemotherapy
KS arises in persons with HIV infection. protocols can be designed to include
This is an example of viral oncogenesis. more agents, thereby reducing cancer
C. Mycobacterium tuberculosis cell resistance, or by giving cycles of
Incorrect. Mycobacterial infections are chemotherapy that get more cancer cells
not related to malignancies. into the replicative pool. Ovarian
Neoplasia | Pathology Reviewer (Utah) | 66 / 72
carcinomas often seed to the peritoneal increased glucose uptake to identify the
cavity. cancer cells.
E. Telomerase expression D. Amplification of HER2
Incorrect. Cancer cells tend to express Incorrect. The amplification of epithelial
telomerase that makes them survive growth factor receptor HER2 is a
longer, but this does not predict characteristic of breast cancer cells, not
diminishing response to chemotherapy. normal cells.
E. Decreased doubling time
A 49-year-old man complains of pain in his left thigh for Incorrect. Neoplasms that have a shorter
3 months. On physical examination his thigh is doubling time are growing faster, which
increased in size, compared to the right. A plain film implies a worse prognosis.
radiograph reveals the presence of a 15 cm solid mass F. Decreased cell cycle S phase
that does not appear to arise from bone, but it does CORRECT. An increased S (DNA
have infiltrative margins. A biopsy of this mass is taken, synthetic) phase is a feature of
and on microscopic examination the mass is composed malignancy, indicating loss of growth
of highly pleomorphic spindle cells. Which of the control with increased cell proliferation.
following immunohistochemical markers is most likely
to be demonstrated in the cells of this mass? A 66-year-old man has noted darker urine for the past
A. Cytokeratin 2 weeks. A urinalysis shows hematuria. Cystoscopy is
Incorrect. The history suggests a performed and there is a 3 cm mass in the dome of the
sarcoma arising in soft tissue. bladder. Biopsies of the mass are taken and on
Cytokeratin is a marker for epithelial microscopic examination show a urothelial carcinoma.
malignancies (carcinomas). Cells of this neoplasm demonstrate a single mutation
B. Factor VIII causing cellular inability to hydrolyze GTP, thus
Incorrect. Factor VIII as a marker is found resulting in cellular transformation. Which of the
in endothelium. It is possible that this following oncogenes is most likely implicated in this
mass is an angiosarcoma, but other case?
sarcomas at this site are more common. A. ABL
C. Alpha fetoprotein Incorrect. This is part of the BCR-ABL
Incorrect. Alpha fetoprotein is a marker fusion gene of the 'Philadelphia
that is found in hepatocellular chromosome' in which the oncogene BL
carcinomas and in some testicular is spliced onto the BCR region of
carcinomas. chromosome 22, resulting in increased
D. Lambda light chain tyrosine kinase activity. It is associated
Incorrect. This component of with chronic myelogenous leukemia.
immunoglobulin would be a marker for B. ERBB2
multiple myeloma. Incorrect. This ERBB2 oncogene (also
E. Vimentin known as HER2/neu) is one of many
CORRECT. The location and size and epidermal growth factor (EGF) receptors
histologic characteristics suggest a and appears to be activated by
sarcoma. Vimentin is an intermediate overexpression and is best associated
filament often found in neoplasms of with breast cancers.
mesenchymal (connective tissue) origin. C. SIS
Incorrect. Overexpression appears to be
A clinical study is performed with patients who had a the mechanism behind the action of this
diagnosis of breast cancer. Characteristics of the grade, oncogene, one form of platelet-derived
stage, molecular biology, and histologic type are growth factor (PBDGF) beta chain
analyzed. Of the following characteristics, which is receptor, associated with brain and bone
most likely to be associated with the best prognosis for tumors.
these patients? D. RAS
A. Increased laminin receptor expression CORRECT. Through mutation the RAS
Incorrect. The expression of laminin gene can be turned on, typically by a
receptors is a characteristic of neoplasia point mutation, to make it an oncogene.
that implies the tumor can more readily Such a mutation can occur in a wide
attach to basement membranes and variety of carcinomas.
invade. E. N-MYC
B. Increased cathepsin expression Incorrect. This oncogene appears to be
Incorrect. Increased cathepsin, an activated by amplification and it has been
enzyme that aids in matrix degradation, associated with neuroblastomas.
implies a worse prognosis because the
neoplasm can more easily spread. A 44-year-old woman who has had multiple sexual
C. Increased aerobic glycolysis partners for the past 30 years has an abnormal Pap
Incorrect. Neoplastic cells can switch smear with cytologic changes suggesting human
their metabolism to aerobic glycolysis papillomavirus (HPV) infection. Without treatment, she
(the Warburg effect) to give them a is most likely to develop which of the following lesions?
selective advantage when actively A. Squamous cell carcinoma
proliferating in the poorly vascularized CORRECT. HPV infection leads to
environment. Positron emission squamous epithelial dysplasias and
tomography (PET) scans make use of carcinomas involving the uterine cervix.
Neoplasia | Pathology Reviewer (Utah) | 67 / 72
HPV is a sexually transmitted infection. A clinical trial of a new chemotherapy protocol is
HPV produces an E7 protein that binds to performed involving patients with lung carcinoma. It is
the RB gene product that normally observed that a response occurs that leads to
suppresses cell proliferation. It produces neoplastic cell DNA strand breakage and cell death.
an E6 protein that binds to the p53 The primary carcinomas are initially observed to
protein to degrade it, so that the cell cycle decrease in size following therapy. However, after 3
is not stopped. Neoplasia can arise in the months, the carcinoma is no longer regressing with this
setting of abnormal cell proliferation. therapy. Altered expression of which of the following by
B. Non-Hodgkin's lymphoma the neoplastic cells has most likely resulted in this
Incorrect. Lymphomas are malignant reduced response?
proliferations of lymphoid cells. They are A. Vimentin
rare in the female genital tract. Incorrect. The presence of the
C. Kaposi sarcoma intermediate filament vimentin in a
Incorrect. Kaposi sarcoma is a rare neoplasm suggests a mesenchymal
condition outside of HIV infection. It is origin -- a sarcoma.
associated with human herpesvirus 8 B. Leukocyte common antigen
infection. Incorrect. LCA identified in a neoplasm
D. Adenocarcinoma suggests a leukocytic differentiation --
Incorrect. Adenocarcinomas may arise in such as a lymphoma.
the endocervix, but they are not C. Laminin receptor
associated with HPV infection. Incorrect. Laminin receptors allow
E. Leiomyoma neoplastic cells to attach more readily to
Incorrect. Leiomyomas are benign basement membranes.
smooth muscle neoplasms. They are D. Actin
most frequent in the uterine myometrium. Incorrect. Actin is a 'thin' filament found
not only in muscle, but a wide variety of
In an epidemiologic study, 'fallout' from nuclear bomb other cell types.
testing and from nuclear power plant disasters is E. DNA topoisomerase II
observed to have released radioactive isotopes into the CORRECT. This topo II is a nuclear
environment. One of these isotopes is found to be enzyme which untangles DNA at the end
strontium-90. It is observed that the strontium-90 in of mitosis. The drugs that target
soils goes into the food chain and eventually topoisomerase II freeze the enzyme as it
contaminates milk products. This contamination is is cutting DNA strands. This leads to DNA
most likely to increase the risk for which of the following strand breakage and cell death.
neoplasms? Resistant cells in culture show alterations
A. Kaposi sarcoma of skin in the topoisomerase II protein.
Incorrect. The epidemic form of this
malignancy is nowadays most often seen In an experiment, biologic characteristics of neoplastic
in homosexual men with HIV infection and non-neoplastic cells are analyzed in culture. It is
and AIDS. observed that cell division in cells derived from
B. Small cell anaplastic carcinoma of lung malignant neoplasms, but not in normal cells, is aided
Incorrect. This lung cancer is nearly by the presence of an enzyme which repairs
always seen in smokers. progressive chromosomal shortening. The lack of
C. Osteosarcoma of bone chromosomal shortening allows the malignant cells to
CORRECT. The fallout includes undergo many more divisions than the normal cells.
strontium-90, an isotope that can be Which of the following enzymes is most likely to have
taken up into bone just like calcium; but this effect?
promotes neoplastic transformation. A. Reverse transcriptase
Science fiction often serves as a vehicle Incorrect. This is the enzyme used by
for social commentary, and the creator of retroviruses, such as human
'Star Trek' had an alien remark, 'They immunodeficiency virus, to transcribe
irradiated their own planet?' in disbelief their RNA into proviral DNA that is
humans would be so stupid to pollute incorporated into the host cell.
their own planet. B. DNA polymerase
D. Nasopharyngeal carcinoma Incorrect. DNA polymerase generates
Incorrect. Epstein-Barr virus (EBV) copies of DNA strands, and it is found in
infection is most often linked to normal cells.
nasopharyngeal carcinoma and to C. Telomerase
Burkitt's lymphoma. CORRECT. The telomerase synthesizes
E. Endometrial carcinoma telomeric chromosomal ends. The so-
Incorrect. Endometrial adenocarcinoma called 'immortal' neoplastic cell lines may
is increased with prolonged estrogen result from the presence of this enzyme.
hormonal imbalance. Interfering with it may promise a new
cancer therapy. Of normal human cells,
stem cells have significant telomerase
activity, but differentiated somatic cells
do not.
Neoplasia | Pathology Reviewer (Utah) | 68 / 72
D. Protein kinase B. Hamartoma
Incorrect. Protein kinases are often the Incorrect. A hamartoma is a mass
'second messengers' in cells that control composed of tissues that are normally
cell growth, and their function may be found at the site in which it is located, but
altered by oncogenic activity. the tissues do not form normal structure,
E. Topoisomerase such as a pulmonary hamartoma.
Incorrect. DNA topoisomerase is an C. Myxoma
enzyme that untangles DNA at the end of Incorrect. A myxoma is a mass
mitosis. Antineoplastic drugs that target composed of stellate cells in a gelatinous
topoisomerase interfere with this enzyme (myxomatous) matrix of connective
to produce DNA strand breakage and cell tissue, such as a left atrial myxoma.
death. D. Teratoma
CORRECT. A teratoma is a neoplasm
A 62-year-old man has complained of pain on urination derived from totipotential germ cells that
for the past week. He is afebrile. On cystoscopy, a differentiate into tissues that represent all
slightly erythematous 1 cm diameter area is seen on three germ layers: ectoderm, endoderm,
the bladder mucosa. This area is biopsied and on and mesoderm. When the elements are
microscopic examination shows cells with marked all well differentiated, the neoplasm is
hyperchromatism and increased nuclear/cytoplasmic 'mature' (benign).
ratio involving the full thickness of the epithelium. E. Mesothelioma
However, these changes are confined to the epithelium Incorrect. A mesothelioma is a neoplasm
above the basement membrane. Which of the following that arises from a mesothelial surface,
terms best describes these biopsy findings? such as a pleural mesothelioma.
A. Metaplasia
Incorrect. A change of one type of A 54-year-old woman has an episode of coughing with
epithelium for another, called metaplasia, blood-streaked sputum. On physical examination she
may be the first step in the process appears cachectic. A chest x-ray shows a mass lesion
leading to neoplasia, but it is reversible. with indistinct borders in the right lung. A transbronchial
B. Minimal dysplasia biopsy is done and on microscopic examination shows
Incorrect. Minimal dysplasia involves small cell anaplastic lung carcinoma (SCLC). She is
only a portion of the epithelium, not the treated with a multidrug chemotherapy regimen and
full thickness. has a partial response. However, metastatic disease
C. Microinvasion develops eight months later. A second chemotherapy
Incorrect. A microinvasive neoplasm protocol is tried, but no response is noted. Which of the
penetrates only a short distance (a few following neoplastic cell characteristics is the most
millimeters) beneath the basement likely reason for failure of the second protocol to elicit
membrane. a response?
D. Hyperplasia A. Abnormal p53 protein
Incorrect. Hyperplastic epithelium is Incorrect. p53 that is mutated may not
increased in thickness, and atypical function effectively in suppressing the
hyperplasia may be a precursor to a appearance of neoplasms by inhibiting
neoplastic process, but it is reversible. the cell cycle. It is the single most
E. Carcinoma in situ common genetic alteration in cancers.
CORRECT. Malignancy confined to the B. Viral infection
epithelium is CIS, and such lesions when Incorrect. Viral oncogenesis, such as the
excised have essentially a 100% cure relationship of human papillomavirus
rate. Unfortunately, urothelial carcinomas infection and squamous cell carcinomas,
have a propensity to be multifocal. does not predict resistance to
chemotherapy.
A 32-year-old woman has noted dull pelvic pain for the C. RAS oncogene activation
last two months. On physical examination there is a Incorrect. The RAS oncogene, activated
mass palpated in the right lower quadrant. An by a point mutation, is common in
abdominal ultrasound reveals an 8 cm mass involving carcinomas, playing a role in
the right ovary. The mass is surgically excised. On development of neoplasia, but does not
gross inspection, the surface of the mass is smooth affect resistance to chemotherapy.
and is not adherent to surrounding pelvic structure. On D. P-glycoprotein expression
sectioning it is cystic and filled with hair. On CORRECT. The increase in P-
microscopic examination there is squamous epithelium, glycoprotein allows the cells to pump out
tall columnar glandular epithelium, cartilage, and any toxin, including any chemotherapy
fibrous connective tissue. Which of the following agent.
neoplasms is she most likely to have? E. Second primary lung cancer
A. Choristoma Incorrect. Though a second, separate
Incorrect. A choristoma is a mass primary may be possible, it is not the
composed of tissues that appear most likely reason for this phenomenon
histologically normal, but not found at the occurring in just a few months. Overall,
site in which it is located, such as a about 10% of persons surviving one
salivary gland choristoma of the middle malignancy will develop another.
ear.
Neoplasia | Pathology Reviewer (Utah) | 69 / 72
A previously healthy 67-year-old man has been feeling C. Neuroblastoma
tired for 5 months. He goes to his physician, who Incorrect. Neuroblastomas are childhood
performs a complete physical examination, including tumors but typically arise in the
stool guaiac, which is positive. A colonoscopy is retroperitoneum in adrenal gland.
performed, and a large, sessile 4.5 cm mass with D. Ewing sarcoma
surface ulceration is found in the cecum. A biopsy of Incorrect. Ewing sarcomas are childhood
this mass microscopically shows irregular glands with neoplasms arising in bone.
crowded, tall columnar cells having marked nuclear E. Malignant lymphoma
hyperchromatism. Which of the following gene Incorrect. A malignant lymphoma is likely
alterations is he most likely to have? to arise within a lymph node in children,
A. Inherited mutant APC gene rarely reaching this size. There should be
Incorrect. Though a sporadic APC CD45 positivity.
mutation may be present at his age,
persons with an inherited faulty APC A 53-year-old woman feels a lump in her right breast.
gene present with polyposis (hundreds of Her nurse practitioner palpates an irregular 3 cm mass
polyps) by the second decade, with high that is not movable because it appears fixed to the
risk for malignant transformation. overlying skin, which is retracted. A mastectomy is
B. Chromosome translocation with BCR performed and the pathologist on sectioning the breast
gene finds a 3 x 3.5 cm ovoid mass that does not have
Incorrect. The t(9;22) producing the discrete borders; but appears to infiltrate into the
BCR-ABL fusion gene is seen with surrounding fibrofatty breast stroma. The mass is firm,
chronic myelogenous leukemia. white, and has a fibrous consistency. Which of the
C. Acquired TP53 gene mutation following features is most likely demonstrated by the
CORRECT. He has an adenocarcinoma gross appearance of this mass?
of the colon. At his age, a sporadic colon A. Anaplasia
cancer is more likely, and inherited APC Incorrect. Anaplasia is a characteristic of
and HNPCC are very unlikely. It takes malignant neoplasms in which there is no
many years for multiple sporadic differentiation to suggest a particular cell
mutations to have occurred that type.
contribute to development of most B. Aplasia
malignancies. p53 mutations are found in Incorrect. Aplasia refers to the absence
many carcinomas. of growth or development.
D. Abnormal germline mismatch repair gene C. Desmoplasia
Incorrect. HNPCC is a condition with CORRECT. Neoplasms, and malignant
many polyps that typically presents in neoplasms in particular, can form a
young to middle age adults. There are fibrous stroma that gives the tumor a
faulty DNA mismatch repair genes. characteristic firm or hard feel on
E. EBV induction of LMP-1 gene palpation. This connective tissue can fix
Incorrect. EBV infection can be the tumor to surrounding structures. An
associated with a variety of neoplasms infiltrating ductal carcinoma of the breast
including lymphomas and will often have this appearance.
nasopharyngeal carcinomas. It is not D. Dysplasia
typically associated with colon cancers. Incorrect. Dysplasia refers to disordered
growth of cells in epithelia. It is
An 8-year-old boy has complained of difficulty considered a premalignant change.
swallowing for the past month. On physical E. Metaplasia
examination he has a palpable, firm mass in the right Incorrect. Metaplasia refers to the
neck. A head CT scan reveals a 7 cm solid soft tissue exchange of one epithelial cell type for
mass lateral to the esophagus on the right. A biopsy of another, typically in response to some
this mass demonstrates pleomorphic spindle cells that form of injury. It may be the first step
are cytokeratin negative, CD45 negative, and vimentin toward neoplasia.
positive. Which of the following neoplasms is he most An 81-year-old man has a routine physical examination
likely to have? and a stool sample is positive for occult blood. He
A. Wilms tumor undergoes colonoscopy and a 5 cm sessile mass is
Incorrect. A Wilms tumor, a common present in the sigmoid colon. Biopsy of the mass yields
childhood neoplasm, arises in the kidney a diagnosis of adenocarcinoma. A chest x-ray shows
B. Rhabdomyosarcoma multiple 1 to 3 cm nodules in both lungs. An alteration
CORRECT. This is one of the more in which of the following molecular components in the
common childhood tumors of soft tissue. neoplastic cells is most likely to explain the formation
Sarcomas tend to be vimentin positive, of lung nodules?
while carcinomas are cytokeratin positive A. Vimentin
and lymphomas CD45 positive. Overall, Incorrect. The presence of the
childhood malignancies are not frequent, intermediate filament vimentin in a
compared with adults, but sarcomas neoplasm suggests a mesenchymal
make up a significant number of those origin -- a sarcoma.
that do occur. B. Leukocyte common antigen
Incorrect. LCA identified in a neoplasm
suggests a leukocytic differentiation --
such as a lymphoma.
Neoplasia | Pathology Reviewer (Utah) | 70 / 72
C. Beta-catenin neoplasms in adulthood. Which of the following
CORRECT. Beta-catenin can locate to neoplasms is most likely to be found in these patients?
the nucleus to drive cellular proliferation, A. Kaposi sarcoma of skin
Mutations of the beta-catenin gene, or in Incorrect. The epidemic form of this
genes such as APC that produce malignancy is nowadays most often seen
proteins that downregulate beta-catenin, in homosexual men with HIV infection
will favor carcinogenesis. In addition, and AIDS.
there is loss of normal cell surface e- B. Small cell anaplastic carcinoma of lung
cadherin expression that maintains Incorrect. This lung cancer is nearly
attachment of cells to each other. Loss of always seen in smokers.
contact inhibition allows neoplastic cells C. Osteosarcoma of bone
to invade and metastasize. Incorrect. Radioactive fallout includes
D. Telomerase Strontium-90, an isotope that can be
Incorrect. Telomerase expressed by taken up into bone just like calcium; but
neoplastic cells allows them to continue promotes neoplastic transformation.
division and growth. D. Nasopharyngeal carcinoma
E. DNA topoisomerase II CORRECT. EBV infection is most often
Incorrect. This is a nuclear enzyme which linked to nasopharyngeal carcinoma in
untangles DNA at the end of mitosis. The Asia and to Burkitt lymphoma in Africa.
drugs that target topoisomerase II freeze E. Endometrial carcinoma
the enzyme as it is cutting DNA strands. Incorrect. Endometrial adenocarcinoma
This leads to DNA strand breakage and is increased with prolonged estrogen
cell death. Resistant cells in culture show hormonal imbalance.
alterations in the topoisomerase II protein.
This may explain cancer chemotherapy A clinical study is performed involving children who
resistance. have developed skin cancers, including squamous cell
carcinomas and basal cell carcinomas. Molecular
A healthy 22-year-old woman undergoes a routine analysis of their cancer cells shows defective DNA
physical examination. A discrete, firm, rubbery, repair from loss of nucleotide excision repair gene
movable mass is found in the left breast. She has no expression. Which of the following genes is most likely
axillary lymphadenopathy. The skin overlying the mutated in these children?
breast and the nipple appear normal. Which of the A. APC
following neoplasms is most likely to be present in this Incorrect. The APC gene is associated
woman? with familial polyposis of the colon that
A. Lipoma can lead to colon carcinomas at a young
Incorrect. Lipomas can be found in a age, sometimes by the 2nd decade.
variety of locations, but in the breast, B. BRCA2
tumors arising from ductular structures Incorrect. BRCA1 and BRCA2 mutations
are more common. are associated with breast cancers, and
B. Intraductal carcinoma other cancers including those in ovary.
Incorrect. Ductal carcinomas, either Inherited mutations can lead to breast
confined to the duct or invasive, are the cancers in young adulthood.
most common primary breast cancer. C. MSH2
They are quite uncommon at age 22. Incorrect. MSH2 mutations are
They are unlikely to be such a discrete a associated with the hereditary
mass. nonpolyposis colon cancer (HNPCC)
C. Malignant lymphoma syndrome, with the appearance of colon
Incorrect. Lymphomas are malignant cancers from abnormal DNA mismatch
proliferations of lymphoid cells. They are repair in young adulthood.
quite rare in the breast. D. NF1
D. Fibroadenoma Incorrect. NF1 mutations are associated
CORRECT. The description is that of a with neurofibromatosis type I, with the
benign neoplasm. Fibroadenoma is the appearance of many neural tumors,
most common benign neoplasm of the particularly neurofibromas.
breast. As the name implies, there are E. TP53
glandular and stromal elements in this Incorrect. p53 mutations are best known
neoplasm. Benign neoplasms are slow as part of carcinogenesis in many adult
growing, discrete masses that do not sporadic cancers. The rare LiFraumeni
invade or metastasize. syndrome results from inheritance of a
E. Leiomyoma faulty p53 gene.
Incorrect. Leiomyomas are benign F. XPA
smooth muscle neoplasms. They are CORRECT. There are multiple
most frequent in the uterine myometrium. xeroderma pigmentosa (XP) genes, from
XPA to XPG and XPV. Sunlight exposure
with ultraviolet radiation induces
An epidemiologic study is performed involving patients pyrimidine dimer formation that is
of East Asian ancestry with long-standing Epstein-Barr ordinarily countered through the action of
virus (EBV) infection. It is observed that these patients excision repair gene products. Mutation
have an increased risk for development of malignant greatly increases the risk for skin cancer
Neoplasia | Pathology Reviewer (Utah) | 71 / 72
formation; and affected persons must A. Beta-catenin
avoid sunlight. CORRECT. Beta-catenin binds to the
G. WT1 intracellular portion of cadherins, which
Incorrect. The WT1 gene is associated anchor cells together, and the loss of this
with Wilms tumor in children. function results in less cell adhesiveness
that favors tumor cell infiltration and
A 29-year-old woman feels a nodule in her neck. On metastases. Beta-catenin also binds to a
examination there is a firm 2 cm mass in the right lobe nuclear transcription factor that increases
of her thyroid. A fine needle aspiration shows cells cell proliferation, and loss of normal beta-
consistent with papillary carcinoma. 2 years ago, she catenin function leads to cell proliferation.
had a lobular carcinoma excised from her left breast. B. BRCA-2
Her 31-year-old sister was recently diagnosed with Incorrect. BRCA-2 and BRCA-1 act as
endometrial carcinoma. A 34-year-old cousin has had tumor suppressors.
colon cancer. Over 3 generations, 6 of 24 close C. Cyclin D
relatives have had a malignancy, most diagnosed Incorrect. Cyclin D operates within the
before age 35. Which of the following most likely cell cycle machinery.
explains the increased risk for cancer in these people? D. NF-1
A. Autosomal dominant cancer syndrome Incorrect. NF-1 codes for the protein
Incorrect. The pattern here is not that of neurofibromin, which acts in the RAS
the 50% recurrence risk for autosomal pathway and promotes GTPase
dominant conditions. Most autosomal activating factors which facilitate
dominant cancer syndromes involve one conversion of RAS from active to inactive.
or two specific tissue sites. The loss of normal NF-1 leads to loss of
B. Defective DNA repair syndrome cell control.
Incorrect. Defective DNA repair E. RAS
syndromes may be inherited in an Incorrect. RAS oncogene mutations
autosomal dominant fashion. The most result in loss of control of cell proliferation;
common is the hereditary non-polyposis there is no direct effect upon cell surface
colon cancer (HNPCC) syndrome, which proteins.
involves colon. F. RB
C. Germline mosaicism Incorrect. RB acts as a tumor suppressor
Incorrect. Germline mosaicism can gene.
account for a cluster of findings within G. RET
siblings of a single generation. Cancer Incorrect. RET codes for a protein that
syndromes are generally not inherited in acts as a cell surface receptor for growth
this fashion. factors. Mutations lead to loss of cell
D. Multifactorial inheritance control because the receptor remains
CORRECT. Cancer that runs in families active even without binding of the growth
often does not have a clearly defined factor.
pattern of inheritance and does not have
a specific genetic marker. Familial A 66-year-old woman has a chronic cough for 3 months.
cancers should be suspected when A chest radiograph shows a left lung 4 cm peripheral
cancers occur at a younger age, multiple mass. Fine needle aspiration biopsy is performed and
family members are involved, and cytologic examination shows non-small cell carcinoma
multiple sites are involved. The sites of (NSCLC). Molecular analysis of these cells shows
involvement (breast, colon, thyroid, upregulation of a pathway form suppression of anti-
kidney, ovary) are similar to those for tumor lymphocytes. Biotherapy with monoclonal
sporadic cancers. antibody to which of the following is most likely to be
E. Teratogen exposure effective in treating this woman's malignancy?
Incorrect. Teratogens produce congenital A. BRAF
anomalies via their effects upon the Incorrect. BRAF is more likely to be a
developing embryo. Cancers are unlikely target in malignant melanoma. BRAF is
to occur in this manner. a serine/threonine protein kinase and
F. Viral infection mutations lead to stimulation of
Incorrect. Few viruses are associated downstream kinases.
with human malignancies. Some B. MYC
persistent viruses such as hepatitis B Incorrect. MYC gene product is part of
virus, may lead to precancerous transcriptional regulation and may target
conditions such as hepatic cirrhosis in genes such as D cyclins.
which neoplasia is more likely to occur. C. PD-1
CORRECT. The programmed cell death-
A study of malignant neoplasms reveals that some of 1 (PD-1) pathway can enable carcinomas
them have a greater potential for invasion and to suppress lymphocyte function;
metastases. Analysis of the cells of cancers that have antibody blockade of PD-1 with its ligand
metastasized reveals a mutation which results in PD-L1 have shown effectiveness in
decreased cell surface expression of E-cadherin. treating in NSCLC.
Which of the following genes is most likely mutated to D. RAS
produce these findings? Incorrect. RAS proteins stimulate the
receptor tyrosine kinase signaling
Neoplasia | Pathology Reviewer (Utah) | 72 / 72
pathway. RAS proteins are members of In an experiment carcinoma cells are observed to
a family of membrane-associated small evade immune destruction by lymphocytes which have
G proteins that normally flip back and been stimulated to proliferate. Which of the following
forth between an excited signal- mechanisms is most likely to provide the tumor cells
transmitting state in which they are with the capacity to evade immune destruction by
bound to GTP and a quiescent state in lymphocytes?
which they are bound to GDP. A. Downregulation of MHC Class I
Stimulation of receptor tyrosine kinases expression
by growth factors leads to exchange of CORRECT. Tumor cells may not express
GDP for GTP. normal levels of MHC class I molecules
E. TP53 to reduce attack by cytotoxic T cells.
Incorrect. TP53 is a cell cycle regulator However, NK cells may attack tumor cells
and functions in the manner of a tumor expressing ligands for NK cell activating
suppressor gene. receptors.
B. Expression of oncofetal antigens
A 59-year-old woman has a screening mammogram Incorrect. Expression of oncofetal
that identifies a suspicious 3 cm mass in her right antigens is a characteristic for cancer
breast. Fine needle aspiration biopsy is performed and cells and may help in recognizing 'tumor
malignant cells are present. Excisional biopsy is markers' for diagnosis.
performed and molecular analysis of the malignant C. Formation of blocking antibodies
cells shows that they are estrogen receptor positive. In Incorrect. Tumor cells do not produce
addition to anti-estrogen therapy, a biotherapy with antibodies (unless they are myeloma
monoclonal antibody targeting the G1 phase of the cell cells, but such antibodies would not block
cycle involves which of the following targets and may immune attack).
be useful in treating this woman? D. Inhibition of regulatory T cells (Tregs)
A. CDK4 Incorrect. Tumor cells may upregulate
CORRECT. Cyclin-dependent kinases Tregs which keep the immune response
(CDKs) promote transition through the in check.
cell cycle; and may neoplastic are E. Upregulation of caspase production
dependent upon the G1 phase of the cell Incorrect. Caspases may be produced in
cycle. Many events impact neoplastic cell apoptosis, and it would be useful to
proliferation via their influence on CDK4 develop a tumor therapy where apoptosis
or CDK6 complexes in the G1 phase of was increased. Tumor cells may survive
the cell cycle. Multiple genes encode for by reducing apoptotic pathways.
the CDKs, part of a family of
serine/threonine protein kinases. ER+
breast cancers respond to estrogen
signaling for proliferation and survival.
ER blockers diminish tumor cell viability
and lead to cell cycle arrest in the G1
phase, but not all breast cancers respond
to anti-hormonal therapy.
B. BCR-ABL
Incorrect. The t9;22 translocation creates
the fusion gene BCR-ABL with tyrosine
kinase activity. This is characteristic for
chronic myelogenous leukemia.
C. HER2
Incorrect. HER2 is an epithelial growth
factor receptor that is amplified in some
breast carcinomas and is targeted by
trastuzumab.
D. TP53
Incorrect. TP53 is a cell cycle regulator
and functions in the manner of a tumor
suppressor gene.
E. WNT
Incorrect. The WNT signaling pathway is
involved in controlling cell fate, adhesion,
and cell polarity during embryonic
development. WNT signals through a
family of cell surface receptors and
stimulates pathways including beta-
catenin and APC.

You might also like